Allowed Publications
Slot System
Featured Buckets
Featured Buckets Admin

Clinical Progress Note: Rhythm Control for Patients With Atrial Fibrillation

Article Type
Changed
Wed, 12/15/2021 - 09:25

It has been 19 years since the publication of the landmark AFFIRM trial.1 At the time of publication, a “rhythm control” strategy was the preferred therapy, with a rate control approach an accepted alternative. AFFIRM showed no mortality benefit of rhythm control over rate control, and its result dramatically shifted the paradigm of atrial fibrillation (AF) management. However, the high crossover rate between treatment arms may have biased the study toward the null hypothesis. Post hoc analyses of AFFIRM and other observational studies indicate that sinus rhythm was associated with a lower risk of death.2 Since AFFIRM, technical advances and procedural experience have improved the safety and efficacy of catheter ablation (CA), and recently published randomized trials have shown improved outcomes with rhythm control. This Progress Note summarizes the recent evidence, updating hospitalists on the management of AF, including inpatient cardioversion, patient selection for CA, use of antiarrhythmic drugs (AADs), and lifestyle modifications associated with maintenance of sinus rhythm.

Search Strategy

A PubMed search for recent publications using combined the MeSH terms “atrial fibrillation” with “catheter ablation,” “antiarrhythmic drugs,” and “lifestyle modifications.” Our review filtered for randomized trials, guidelines, and selected reviews. 

Should I pursue inpatient cardioversion for my patient?

Urgent cardioversion is recommended for those with hemodynamic instability, AF associated ischemia, or acute heart failure.3 Whether to perform elective cardioversion depends on AF duration, symptoms, and the initial evaluation for structural heart disease or reversible causes of AF. Evaluation for new-onset AF includes eliciting a history of AF-associated comorbidities (hypertension, alcohol use, obstructive sleep apnea) and an echocardiogram and thyroid, renal, and liver function tests.3 Stable patients with AF precipitated by high-catecholamine states (eg, postoperative AF, sepsis, hyperthyroidism, pulmonary embolism, substance use) require management of the underlying condition before considering rhythm control. Inpatient electrical or pharmacologic cardioversion may be considered for patients with stable, new-onset AF sufficiently symptomatic to require hospitalization. Pre-procedure anticoagulation and a transesophageal echocardiogram to rule out left atrial thrombus before cardioversion is preferred for a first episode of AF suspected of lasting longer than 48 hours but requires anesthesia and considerable resources. In resource-constrained settings, patients asymptomatic once rate controlled may be safely discharged with a referral for outpatient cardioversion.

For patients with structural heart disease (left atrial dilation), previously failed cardioversion, or recurrent AF, initiating AADs (eg, ibutilide, amiodarone) before electrical cardioversion can improve the success rate of cardioversion.3 Ibutilide infusion requires cardiology consultation and postinfusion hemodynamic and QTc monitoring. Defer immediate cardioversion among stable patients unable to continue a minimum of 4 weeks of anticoagulation or with comorbidities for which risks of cardioversion outweigh benefits.

 

 

Is a rhythm control strategy best for my patient?

Successful maintenance of sinus rhythm is associated with reduced symptom burden and improved quality of life and is recommended for patients with persistent symptoms, failure of rate control, younger age, first episode of AF, or patient preference for rhythm control.3 Since AF progression results in irreversible cardiac remodeling, earlier rhythm control may prevent further atrial remodeling and atrial myopathy.

The EAST-AFNET 4 trial evaluated a rhythm-control strategy in patients with AF duration <12 months and who met two of the following: age > 65 years, female sex, heart failure, hypertension, diabetes, coronary artery disease, and chronic kidney disease.4 Maintenance of sinus rhythm was associated with a lower composite outcome of adverse cardiovascular outcomes and death from cardiovascular causes over 5 years compared to rate control (3.9/100 person-years vs 5.0/100 person-years, P = .005). Interestingly, roughly 20% of patients underwent CA and the remainder received AADs. The large proportion of patients treated with AADs raises the question of why the results differed from AFFIRM. There are four primary differences between these trials to consider. First, EAST-AFNET 4 used an early rhythm-control strategy (<12 months). Second, nearly all patients in EAST-AFNET 4 continued guideline-recommend anticoagulation compared to 70% receiving rhythm control in AFFIRM. Third, in AFFIRM, 62.8% of patients received amiodarone, which has significant long-term adverse effects compared to 11.8% by the end of EAST-AFNET 4. Finally, increased use of CA in EAST-AFNET 4 may have contributed to the success of rhythm control. In patients with cardiovascular disease or cardiovascular risk factors, a rhythm-control strategy will be best if implemented early (<12 months), before the development of long-standing persistent AF, and if clinicians adhere to anticoagulation recommendations.

Should my patient receive antiarrhythmics, catheter ablation, or both?

Antiarrhythmic Drugs

Antiarrhythmic drug use prior to CA remains the cornerstone of a rhythm-control strategy for patients meeting EAST-AFNET 4 trial criteria or patient preference for medical management. Hospitalists’ knowledge of key differences between AADs used in EAST-AFNET 4 and AFFIRM as well as American Heart Association/American College of Cardiology/Heart Rhythm Society (AHA/ACC/HRS) guideline recommendations help avoid harmful AAD prescribing. Notably, 21.9% of patients in AFFIRM received AADs no longer recommended to maintain sinus rhythm in the AHA/ACC/HRS guidelines (quinidine, disopyramide, procainamide, moricizine).3 For patients without structural heart disease, flecainide, propafenone, sotalol, or dronedarone are preferred. Dronedarone and sotalol remain an option for those with coronary artery disease. For patients with heart failure with reduced ejection fraction (HFrEF), amiodarone and dofetilide are preferred (Table).3

jhmvol16no12_patel06711215e_t1.jpg

Catheter Ablation

The AHA/ACC/HRS guidelines offer a Ia recommendation for CA in patients with recurrent, symptomatic AF who failed AAD therapy. Initial CA is a IIa recommendation and is increasingly common for patients with paroxysmal AF who prefer this strategy to long-term AAD use.3 Recent trials evaluated CA as a primary treatment modality in patients with heart failure and as initial management before AADs.

Initial Catheter Ablation

The CABANA trial compared CA with AADs as an initial approach for maintaining sinus rhythm.5 In the intention-to-treat analysis, there was no difference in all death or disabling stroke between AAD therapy and CA at 5-year follow-up. The results are limited by a 27.5% crossover rate from drug therapy to CA. The per-protocol analysis based on the treatment received favored CA for the primary composite outcome of death, disabling stroke, serious bleeding, or cardiac arrest at 12 months. The STOP-AF and EARLY-AF trials found that initial CA was more successful in maintaining freedom from atrial arrhythmias (74.6% vs 45.0%, P < .001)6 and fewer symptomatic atrial arrhythmias among patients with paroxysmal AF compared to AADs, without significant CA-associated adverse events.6,7

 

 

While hospitalists should interpret the per-protocol analysis cautiously when determining the clinical benefit, these trials indicate initial CA is as safe as AADs and improves freedom from AF. Duration of AF, knowledge of periprocedural anticoagulation recommendations, and CA procedural complications are important when recommending CA (Figure). Efficacy of CA is approximately 70% in paroxysmal AF and decreases for persistent and long-standing AF.6 Complications of CA include venous access site hematoma, cardiac tamponade, phrenic nerve injury, pulmonary vein stenosis, atrial-esophageal fistula, left atrial flutter, and stroke due to endothelial injury and intraprocedural thrombosis.3 Therapeutic anticoagulation is required before CA and for at least 2 months post ablation regardless of the CHA2DS2-VASc score.3

jhmvol16no12_patel06711215e_f1.jpg

Catheter Ablation Plus Antiarrhythmics

Ongoing AADs following CA may suppress AF triggers, especially in patients with persistent AF or high-risk for recurrence post ablation (left atrial dilation). The AMIO-CAT trial found that 4 weeks of amiodarone after ablation reduced early AF recurrence at 3 months (34% vs 53%, P = .006), arrhythmia-related hospitalizations, and need for cardioversion in patients with paroxysmal and persistent AF.8 However, amiodarone did not reduce recurrent atrial tachyarrhythmias at 6 months. The POWDER-AF trial evaluated AAD use for 1 year after CA in patients with drug-refractory paroxysmal AF.9 Continuation of class IC (eg, flecainide) and III (eg, amiodarone) AADs resulted in a near 20% absolute risk reduction in recurrent atrial arrhythmias and reduced the need for repeat CA. These trials suggest that discharging patients on adjunctive AADs decreases early recurrence of AF and arrhythmia-related hospitalizations; however, studies evaluating additional clinical outcomes are needed.

Heart Failure

The AATAC trial found CA was superior to amiodarone therapy at maintaining freedom from AF and reducing unplanned hospitalizations and mortality among patients with persistent AF and HFrEF.10 The larger CASTLE-AF trial randomized patients with an ejection fraction below 35% and NYHA class II or greater symptoms with symptomatic paroxysmal AF or persistent AF in whom AAD therapy failed to CA or medical therapy.11 The CA group experienced lower cardiovascular mortality (11.2% vs 22.3%, P = .009) and fewer heart failure hospitalizations (20.7% vs 35.9%, P = .004). The subsequent AMICA trial did not find a benefit of CA in patients with HFrEF and persistent or long-standing persistent AF; however, this trial was limited to 12 months, whereas the benefit of CA in CASTLE-AF was observed after 12 months.12 Also, AMICA enrolled patients with higher NYHA class. Therefore, hospitalists should refer AF patients with left ventricular systolic dysfunction and NYHA II or III symptoms for CA. Comparing AMICA and CASTLE-AF suggests earlier referral for CA, prior to the development of worsening heart failure symptoms, may improve outcomes.

Data for patients with heart failure with preserved EF (HFpEF) is limited. One small trial showed reduced heart failure hospitalizations in HFpEF patients treated with CA compared to AADs or beta-blockers.13 It is reasonable to refer HFpEF patients with persisting symptoms or reduced quality of life for CA.

What long-term risk-modification should I recommend?

The AHA Scientific Statement on Lifestyle and Risk Factor Modification for Reduction of Atrial Fibrillation delineates risk factors that increase the incidence of AF, including alcohol consumption, obstructive sleep apnea, hypertension, and obesity.14 Among regular alcohol consumers with paroxysmal or persistent AF managed with a rhythm-control strategy, cessation of alcohol has been shown to significantly lower the incidence of recurrent AF (53.0% vs 73.0%, P = .005), and lead to a longer time until recurrence of AF compared to patients regularly consuming alcohol.15 Among patients with obstructive sleep apnea, a systematic review of nonrandomized studies showed continuous positive airway pressure is associated with maintenance of sinus rhythm.14 Control of these risk factors is associated with up to approximately 40% of patients maintaining sinus rhythm without intervention, and hospitalists should encourage lifestyle modification to maximize the probability of maintaining sinus rhythm. 

Summary

Hospitalists frequently determine the best initial management strategy for patients admitted with new-onset AF, and recent literature may shift more patients towards management with rhythm control. Based on the trials reviewed in this Progress Note, hospitalists should recommend a rhythm-control strategy for patients with symptomatic, paroxysmal, or persistent AF of <12 months’ duration and refer patients with HFrEF for CA. Adherence to guideline recommendations is essential when prescribing AADs to avoid adverse drug events. It is vital to ensure patients managed with a rhythm-control strategy receive anticoagulation for 4 weeks post cardioversion or 2 months post CA with long-term anticoagulation based on CHA2DS2-VASc score. Finally, admissions for AF should serve as a catalyst to communicate to patients the importance of addressing obstructive sleep apnea, obesity, and alcohol use disorders. Applying these evidence-based practices will enable hospitalists to make clinical decisions that improve symptom burden and survival for patients with AF.

 

References

1. Wyse DG, Waldo AL, DiMarco JP, et al. A comparison of rate control and rhythm control in patients with atrial fibrillation. N Engl J Med. 2002;347(23):1825-1833. https://doi.org/10.1056/NEJMoa021328

2. Corley SD, Epstein AE, DiMarco JP, et al. Relationships between sinus rhythm, treatment, and survival in the Atrial Fibrillation Follow-Up Investigation of Rhythm Management (AFFIRM) Study. Circulation. 2004;109(12):1509-1513. https://doi.org/10.1161/01.Cir.0000121736.16643.11

3. January CT, Wann LS, Alpert JS, et al. 2014 AHA/ACC/HRS guideline for the management of patients with atrial fibrillation. Circulation. 2014;130(23):e199-e267. https://doi.org/10.1161/CIR.0000000000000041

4. Kirchhof P, Camm AJ, Goette A, et al. Early rhythm-control therapy in patients with atrial fibrillation. N Engl J Med. 2020;383(14):1305-1316. https://doi.org/10.1056/NEJMoa2019422

5. Packer DL, Mark DB, Robb RA, et al. Effect of catheter ablation vs antiarrhythmic drug therapy on mortality, stroke, bleeding, and cardiac arrest among patients with atrial fibrillation: the CABANA randomized clinical trial. JAMA. 2019;321(13):1261-1274. https://doi.org/doi:10.1001/jama.2019.0693

6. Wazni OM, Dandamudi G, Sood N, et al. Cryoballoon ablation as initial therapy for atrial fibrillation. N Engl J Med. 2021;384(4):316-324. https://doi.org/10.1056/NEJMoa2029554

7. Andrade JG, Wells GA, Deyell MW, et al. Cryoablation or drug therapy for initial treatment of atrial fibrillation. N Engl J Med. 2021;384(4):305-315. https://doi.org/10.1056/NEJMoa2029980

8. Darkner S, Chen X, Hansen J, et al. Recurrence of arrhythmia following short-term oral AMIOdarone after CATheter ablation for atrial fibrillation: a double-blind, randomized, placebo-controlled study (AMIO-CAT trial). Eur Heart J. 2014;35(47):3356-3364. https://doi.org/10.1093/eurheartj/ehu354

9. Duytschaever M, Demolder A, Phlips T, et al. PulmOnary vein isolation with vs. without continued antiarrhythmic drug treatment in subjects with recurrent atrial fibrillation (POWDER AF): results from a multicentre randomized trial. Eur Heart J. 2018;39(16):1429-1437. https://doi.org/10.1093/eurheartj/ehx666

10. Di Biase L, Mohanty P, Mohanty S, et al. Ablation versus amiodarone for treatment of persistent atrial fibrillation in patients with congestive heart failure and an implanted device: results from the AATAC multicenter randomized trial. Circulation. 2016;133(17):1637-1344. https://doi.org/10.1161/circulationaha.115.019406

11. Marrouche NF, Brachmann J, Andresen D, et al. Catheter ablation for atrial fibrillation with heart failure. N Engl J Med. 2018;378(5):417-427. https://doi.org/10.1056/NEJMoa1707855

12. Kuck KH, Merkely B, Zahn R, et al. Catheter ablation versus best medical therapy in patients with persistent atrial fibrillation and congestive heart failure: the randomized AMICA Trial. Circ Arrhythm Electrophysiol. 2019;12(12):e007731. d https://doi.org/10.1161/circep.119.007731

13. Fukui A, Tanino T, Yamaguchi T, et al. Catheter ablation of atrial fibrillation reduces heart failure rehospitalization in patients with heart failure with preserved ejection fraction. J Cardiovasc Electrophysiol. 2020;31(3):682-688. https://doi.org/10.1111/jce.14369

14. Chung MK, Eckhardt LL, Chen LY, et al. Lifestyle and risk factor modification for reduction of atrial fibrillation: a scientific statement from the American Heart Association. Circulation. 2020;141(16):e750-e772. https://doi.org/10.1161/CIR.0000000000000748

15. Voskoboinik A, Kalman JM, De Silva A, et al. Alcohol abstinence in drinkers with atrial fibrillation. N Engl J Med. 2020;382(1):20-28. https://doi.org/10.1056/NEJMoa1817591

Article PDF
Author and Disclosure Information

1 University of Colorado Anschutz Medical Center, Department of Medicine, Aurora, Colorado; 2 University of Kentucky and Lexington Veterans Administration Medical Center, Lexington, Kentucky; 3 Denver Health Medical Center, Department of Medicine, Denver, Colorado; 4 University of Colorado Anschutz Medical Center, Division of Cardiac Electrophysiology, Aurora, Colorado.

Disclosures
The authors reported no conflicts of interest.

Publications
Topics
Sections
Author and Disclosure Information

1 University of Colorado Anschutz Medical Center, Department of Medicine, Aurora, Colorado; 2 University of Kentucky and Lexington Veterans Administration Medical Center, Lexington, Kentucky; 3 Denver Health Medical Center, Department of Medicine, Denver, Colorado; 4 University of Colorado Anschutz Medical Center, Division of Cardiac Electrophysiology, Aurora, Colorado.

Disclosures
The authors reported no conflicts of interest.

Author and Disclosure Information

1 University of Colorado Anschutz Medical Center, Department of Medicine, Aurora, Colorado; 2 University of Kentucky and Lexington Veterans Administration Medical Center, Lexington, Kentucky; 3 Denver Health Medical Center, Department of Medicine, Denver, Colorado; 4 University of Colorado Anschutz Medical Center, Division of Cardiac Electrophysiology, Aurora, Colorado.

Disclosures
The authors reported no conflicts of interest.

Article PDF
Article PDF

It has been 19 years since the publication of the landmark AFFIRM trial.1 At the time of publication, a “rhythm control” strategy was the preferred therapy, with a rate control approach an accepted alternative. AFFIRM showed no mortality benefit of rhythm control over rate control, and its result dramatically shifted the paradigm of atrial fibrillation (AF) management. However, the high crossover rate between treatment arms may have biased the study toward the null hypothesis. Post hoc analyses of AFFIRM and other observational studies indicate that sinus rhythm was associated with a lower risk of death.2 Since AFFIRM, technical advances and procedural experience have improved the safety and efficacy of catheter ablation (CA), and recently published randomized trials have shown improved outcomes with rhythm control. This Progress Note summarizes the recent evidence, updating hospitalists on the management of AF, including inpatient cardioversion, patient selection for CA, use of antiarrhythmic drugs (AADs), and lifestyle modifications associated with maintenance of sinus rhythm.

Search Strategy

A PubMed search for recent publications using combined the MeSH terms “atrial fibrillation” with “catheter ablation,” “antiarrhythmic drugs,” and “lifestyle modifications.” Our review filtered for randomized trials, guidelines, and selected reviews. 

Should I pursue inpatient cardioversion for my patient?

Urgent cardioversion is recommended for those with hemodynamic instability, AF associated ischemia, or acute heart failure.3 Whether to perform elective cardioversion depends on AF duration, symptoms, and the initial evaluation for structural heart disease or reversible causes of AF. Evaluation for new-onset AF includes eliciting a history of AF-associated comorbidities (hypertension, alcohol use, obstructive sleep apnea) and an echocardiogram and thyroid, renal, and liver function tests.3 Stable patients with AF precipitated by high-catecholamine states (eg, postoperative AF, sepsis, hyperthyroidism, pulmonary embolism, substance use) require management of the underlying condition before considering rhythm control. Inpatient electrical or pharmacologic cardioversion may be considered for patients with stable, new-onset AF sufficiently symptomatic to require hospitalization. Pre-procedure anticoagulation and a transesophageal echocardiogram to rule out left atrial thrombus before cardioversion is preferred for a first episode of AF suspected of lasting longer than 48 hours but requires anesthesia and considerable resources. In resource-constrained settings, patients asymptomatic once rate controlled may be safely discharged with a referral for outpatient cardioversion.

For patients with structural heart disease (left atrial dilation), previously failed cardioversion, or recurrent AF, initiating AADs (eg, ibutilide, amiodarone) before electrical cardioversion can improve the success rate of cardioversion.3 Ibutilide infusion requires cardiology consultation and postinfusion hemodynamic and QTc monitoring. Defer immediate cardioversion among stable patients unable to continue a minimum of 4 weeks of anticoagulation or with comorbidities for which risks of cardioversion outweigh benefits.

 

 

Is a rhythm control strategy best for my patient?

Successful maintenance of sinus rhythm is associated with reduced symptom burden and improved quality of life and is recommended for patients with persistent symptoms, failure of rate control, younger age, first episode of AF, or patient preference for rhythm control.3 Since AF progression results in irreversible cardiac remodeling, earlier rhythm control may prevent further atrial remodeling and atrial myopathy.

The EAST-AFNET 4 trial evaluated a rhythm-control strategy in patients with AF duration <12 months and who met two of the following: age > 65 years, female sex, heart failure, hypertension, diabetes, coronary artery disease, and chronic kidney disease.4 Maintenance of sinus rhythm was associated with a lower composite outcome of adverse cardiovascular outcomes and death from cardiovascular causes over 5 years compared to rate control (3.9/100 person-years vs 5.0/100 person-years, P = .005). Interestingly, roughly 20% of patients underwent CA and the remainder received AADs. The large proportion of patients treated with AADs raises the question of why the results differed from AFFIRM. There are four primary differences between these trials to consider. First, EAST-AFNET 4 used an early rhythm-control strategy (<12 months). Second, nearly all patients in EAST-AFNET 4 continued guideline-recommend anticoagulation compared to 70% receiving rhythm control in AFFIRM. Third, in AFFIRM, 62.8% of patients received amiodarone, which has significant long-term adverse effects compared to 11.8% by the end of EAST-AFNET 4. Finally, increased use of CA in EAST-AFNET 4 may have contributed to the success of rhythm control. In patients with cardiovascular disease or cardiovascular risk factors, a rhythm-control strategy will be best if implemented early (<12 months), before the development of long-standing persistent AF, and if clinicians adhere to anticoagulation recommendations.

Should my patient receive antiarrhythmics, catheter ablation, or both?

Antiarrhythmic Drugs

Antiarrhythmic drug use prior to CA remains the cornerstone of a rhythm-control strategy for patients meeting EAST-AFNET 4 trial criteria or patient preference for medical management. Hospitalists’ knowledge of key differences between AADs used in EAST-AFNET 4 and AFFIRM as well as American Heart Association/American College of Cardiology/Heart Rhythm Society (AHA/ACC/HRS) guideline recommendations help avoid harmful AAD prescribing. Notably, 21.9% of patients in AFFIRM received AADs no longer recommended to maintain sinus rhythm in the AHA/ACC/HRS guidelines (quinidine, disopyramide, procainamide, moricizine).3 For patients without structural heart disease, flecainide, propafenone, sotalol, or dronedarone are preferred. Dronedarone and sotalol remain an option for those with coronary artery disease. For patients with heart failure with reduced ejection fraction (HFrEF), amiodarone and dofetilide are preferred (Table).3

jhmvol16no12_patel06711215e_t1.jpg

Catheter Ablation

The AHA/ACC/HRS guidelines offer a Ia recommendation for CA in patients with recurrent, symptomatic AF who failed AAD therapy. Initial CA is a IIa recommendation and is increasingly common for patients with paroxysmal AF who prefer this strategy to long-term AAD use.3 Recent trials evaluated CA as a primary treatment modality in patients with heart failure and as initial management before AADs.

Initial Catheter Ablation

The CABANA trial compared CA with AADs as an initial approach for maintaining sinus rhythm.5 In the intention-to-treat analysis, there was no difference in all death or disabling stroke between AAD therapy and CA at 5-year follow-up. The results are limited by a 27.5% crossover rate from drug therapy to CA. The per-protocol analysis based on the treatment received favored CA for the primary composite outcome of death, disabling stroke, serious bleeding, or cardiac arrest at 12 months. The STOP-AF and EARLY-AF trials found that initial CA was more successful in maintaining freedom from atrial arrhythmias (74.6% vs 45.0%, P < .001)6 and fewer symptomatic atrial arrhythmias among patients with paroxysmal AF compared to AADs, without significant CA-associated adverse events.6,7

 

 

While hospitalists should interpret the per-protocol analysis cautiously when determining the clinical benefit, these trials indicate initial CA is as safe as AADs and improves freedom from AF. Duration of AF, knowledge of periprocedural anticoagulation recommendations, and CA procedural complications are important when recommending CA (Figure). Efficacy of CA is approximately 70% in paroxysmal AF and decreases for persistent and long-standing AF.6 Complications of CA include venous access site hematoma, cardiac tamponade, phrenic nerve injury, pulmonary vein stenosis, atrial-esophageal fistula, left atrial flutter, and stroke due to endothelial injury and intraprocedural thrombosis.3 Therapeutic anticoagulation is required before CA and for at least 2 months post ablation regardless of the CHA2DS2-VASc score.3

jhmvol16no12_patel06711215e_f1.jpg

Catheter Ablation Plus Antiarrhythmics

Ongoing AADs following CA may suppress AF triggers, especially in patients with persistent AF or high-risk for recurrence post ablation (left atrial dilation). The AMIO-CAT trial found that 4 weeks of amiodarone after ablation reduced early AF recurrence at 3 months (34% vs 53%, P = .006), arrhythmia-related hospitalizations, and need for cardioversion in patients with paroxysmal and persistent AF.8 However, amiodarone did not reduce recurrent atrial tachyarrhythmias at 6 months. The POWDER-AF trial evaluated AAD use for 1 year after CA in patients with drug-refractory paroxysmal AF.9 Continuation of class IC (eg, flecainide) and III (eg, amiodarone) AADs resulted in a near 20% absolute risk reduction in recurrent atrial arrhythmias and reduced the need for repeat CA. These trials suggest that discharging patients on adjunctive AADs decreases early recurrence of AF and arrhythmia-related hospitalizations; however, studies evaluating additional clinical outcomes are needed.

Heart Failure

The AATAC trial found CA was superior to amiodarone therapy at maintaining freedom from AF and reducing unplanned hospitalizations and mortality among patients with persistent AF and HFrEF.10 The larger CASTLE-AF trial randomized patients with an ejection fraction below 35% and NYHA class II or greater symptoms with symptomatic paroxysmal AF or persistent AF in whom AAD therapy failed to CA or medical therapy.11 The CA group experienced lower cardiovascular mortality (11.2% vs 22.3%, P = .009) and fewer heart failure hospitalizations (20.7% vs 35.9%, P = .004). The subsequent AMICA trial did not find a benefit of CA in patients with HFrEF and persistent or long-standing persistent AF; however, this trial was limited to 12 months, whereas the benefit of CA in CASTLE-AF was observed after 12 months.12 Also, AMICA enrolled patients with higher NYHA class. Therefore, hospitalists should refer AF patients with left ventricular systolic dysfunction and NYHA II or III symptoms for CA. Comparing AMICA and CASTLE-AF suggests earlier referral for CA, prior to the development of worsening heart failure symptoms, may improve outcomes.

Data for patients with heart failure with preserved EF (HFpEF) is limited. One small trial showed reduced heart failure hospitalizations in HFpEF patients treated with CA compared to AADs or beta-blockers.13 It is reasonable to refer HFpEF patients with persisting symptoms or reduced quality of life for CA.

What long-term risk-modification should I recommend?

The AHA Scientific Statement on Lifestyle and Risk Factor Modification for Reduction of Atrial Fibrillation delineates risk factors that increase the incidence of AF, including alcohol consumption, obstructive sleep apnea, hypertension, and obesity.14 Among regular alcohol consumers with paroxysmal or persistent AF managed with a rhythm-control strategy, cessation of alcohol has been shown to significantly lower the incidence of recurrent AF (53.0% vs 73.0%, P = .005), and lead to a longer time until recurrence of AF compared to patients regularly consuming alcohol.15 Among patients with obstructive sleep apnea, a systematic review of nonrandomized studies showed continuous positive airway pressure is associated with maintenance of sinus rhythm.14 Control of these risk factors is associated with up to approximately 40% of patients maintaining sinus rhythm without intervention, and hospitalists should encourage lifestyle modification to maximize the probability of maintaining sinus rhythm. 

Summary

Hospitalists frequently determine the best initial management strategy for patients admitted with new-onset AF, and recent literature may shift more patients towards management with rhythm control. Based on the trials reviewed in this Progress Note, hospitalists should recommend a rhythm-control strategy for patients with symptomatic, paroxysmal, or persistent AF of <12 months’ duration and refer patients with HFrEF for CA. Adherence to guideline recommendations is essential when prescribing AADs to avoid adverse drug events. It is vital to ensure patients managed with a rhythm-control strategy receive anticoagulation for 4 weeks post cardioversion or 2 months post CA with long-term anticoagulation based on CHA2DS2-VASc score. Finally, admissions for AF should serve as a catalyst to communicate to patients the importance of addressing obstructive sleep apnea, obesity, and alcohol use disorders. Applying these evidence-based practices will enable hospitalists to make clinical decisions that improve symptom burden and survival for patients with AF.

 

It has been 19 years since the publication of the landmark AFFIRM trial.1 At the time of publication, a “rhythm control” strategy was the preferred therapy, with a rate control approach an accepted alternative. AFFIRM showed no mortality benefit of rhythm control over rate control, and its result dramatically shifted the paradigm of atrial fibrillation (AF) management. However, the high crossover rate between treatment arms may have biased the study toward the null hypothesis. Post hoc analyses of AFFIRM and other observational studies indicate that sinus rhythm was associated with a lower risk of death.2 Since AFFIRM, technical advances and procedural experience have improved the safety and efficacy of catheter ablation (CA), and recently published randomized trials have shown improved outcomes with rhythm control. This Progress Note summarizes the recent evidence, updating hospitalists on the management of AF, including inpatient cardioversion, patient selection for CA, use of antiarrhythmic drugs (AADs), and lifestyle modifications associated with maintenance of sinus rhythm.

Search Strategy

A PubMed search for recent publications using combined the MeSH terms “atrial fibrillation” with “catheter ablation,” “antiarrhythmic drugs,” and “lifestyle modifications.” Our review filtered for randomized trials, guidelines, and selected reviews. 

Should I pursue inpatient cardioversion for my patient?

Urgent cardioversion is recommended for those with hemodynamic instability, AF associated ischemia, or acute heart failure.3 Whether to perform elective cardioversion depends on AF duration, symptoms, and the initial evaluation for structural heart disease or reversible causes of AF. Evaluation for new-onset AF includes eliciting a history of AF-associated comorbidities (hypertension, alcohol use, obstructive sleep apnea) and an echocardiogram and thyroid, renal, and liver function tests.3 Stable patients with AF precipitated by high-catecholamine states (eg, postoperative AF, sepsis, hyperthyroidism, pulmonary embolism, substance use) require management of the underlying condition before considering rhythm control. Inpatient electrical or pharmacologic cardioversion may be considered for patients with stable, new-onset AF sufficiently symptomatic to require hospitalization. Pre-procedure anticoagulation and a transesophageal echocardiogram to rule out left atrial thrombus before cardioversion is preferred for a first episode of AF suspected of lasting longer than 48 hours but requires anesthesia and considerable resources. In resource-constrained settings, patients asymptomatic once rate controlled may be safely discharged with a referral for outpatient cardioversion.

For patients with structural heart disease (left atrial dilation), previously failed cardioversion, or recurrent AF, initiating AADs (eg, ibutilide, amiodarone) before electrical cardioversion can improve the success rate of cardioversion.3 Ibutilide infusion requires cardiology consultation and postinfusion hemodynamic and QTc monitoring. Defer immediate cardioversion among stable patients unable to continue a minimum of 4 weeks of anticoagulation or with comorbidities for which risks of cardioversion outweigh benefits.

 

 

Is a rhythm control strategy best for my patient?

Successful maintenance of sinus rhythm is associated with reduced symptom burden and improved quality of life and is recommended for patients with persistent symptoms, failure of rate control, younger age, first episode of AF, or patient preference for rhythm control.3 Since AF progression results in irreversible cardiac remodeling, earlier rhythm control may prevent further atrial remodeling and atrial myopathy.

The EAST-AFNET 4 trial evaluated a rhythm-control strategy in patients with AF duration <12 months and who met two of the following: age > 65 years, female sex, heart failure, hypertension, diabetes, coronary artery disease, and chronic kidney disease.4 Maintenance of sinus rhythm was associated with a lower composite outcome of adverse cardiovascular outcomes and death from cardiovascular causes over 5 years compared to rate control (3.9/100 person-years vs 5.0/100 person-years, P = .005). Interestingly, roughly 20% of patients underwent CA and the remainder received AADs. The large proportion of patients treated with AADs raises the question of why the results differed from AFFIRM. There are four primary differences between these trials to consider. First, EAST-AFNET 4 used an early rhythm-control strategy (<12 months). Second, nearly all patients in EAST-AFNET 4 continued guideline-recommend anticoagulation compared to 70% receiving rhythm control in AFFIRM. Third, in AFFIRM, 62.8% of patients received amiodarone, which has significant long-term adverse effects compared to 11.8% by the end of EAST-AFNET 4. Finally, increased use of CA in EAST-AFNET 4 may have contributed to the success of rhythm control. In patients with cardiovascular disease or cardiovascular risk factors, a rhythm-control strategy will be best if implemented early (<12 months), before the development of long-standing persistent AF, and if clinicians adhere to anticoagulation recommendations.

Should my patient receive antiarrhythmics, catheter ablation, or both?

Antiarrhythmic Drugs

Antiarrhythmic drug use prior to CA remains the cornerstone of a rhythm-control strategy for patients meeting EAST-AFNET 4 trial criteria or patient preference for medical management. Hospitalists’ knowledge of key differences between AADs used in EAST-AFNET 4 and AFFIRM as well as American Heart Association/American College of Cardiology/Heart Rhythm Society (AHA/ACC/HRS) guideline recommendations help avoid harmful AAD prescribing. Notably, 21.9% of patients in AFFIRM received AADs no longer recommended to maintain sinus rhythm in the AHA/ACC/HRS guidelines (quinidine, disopyramide, procainamide, moricizine).3 For patients without structural heart disease, flecainide, propafenone, sotalol, or dronedarone are preferred. Dronedarone and sotalol remain an option for those with coronary artery disease. For patients with heart failure with reduced ejection fraction (HFrEF), amiodarone and dofetilide are preferred (Table).3

jhmvol16no12_patel06711215e_t1.jpg

Catheter Ablation

The AHA/ACC/HRS guidelines offer a Ia recommendation for CA in patients with recurrent, symptomatic AF who failed AAD therapy. Initial CA is a IIa recommendation and is increasingly common for patients with paroxysmal AF who prefer this strategy to long-term AAD use.3 Recent trials evaluated CA as a primary treatment modality in patients with heart failure and as initial management before AADs.

Initial Catheter Ablation

The CABANA trial compared CA with AADs as an initial approach for maintaining sinus rhythm.5 In the intention-to-treat analysis, there was no difference in all death or disabling stroke between AAD therapy and CA at 5-year follow-up. The results are limited by a 27.5% crossover rate from drug therapy to CA. The per-protocol analysis based on the treatment received favored CA for the primary composite outcome of death, disabling stroke, serious bleeding, or cardiac arrest at 12 months. The STOP-AF and EARLY-AF trials found that initial CA was more successful in maintaining freedom from atrial arrhythmias (74.6% vs 45.0%, P < .001)6 and fewer symptomatic atrial arrhythmias among patients with paroxysmal AF compared to AADs, without significant CA-associated adverse events.6,7

 

 

While hospitalists should interpret the per-protocol analysis cautiously when determining the clinical benefit, these trials indicate initial CA is as safe as AADs and improves freedom from AF. Duration of AF, knowledge of periprocedural anticoagulation recommendations, and CA procedural complications are important when recommending CA (Figure). Efficacy of CA is approximately 70% in paroxysmal AF and decreases for persistent and long-standing AF.6 Complications of CA include venous access site hematoma, cardiac tamponade, phrenic nerve injury, pulmonary vein stenosis, atrial-esophageal fistula, left atrial flutter, and stroke due to endothelial injury and intraprocedural thrombosis.3 Therapeutic anticoagulation is required before CA and for at least 2 months post ablation regardless of the CHA2DS2-VASc score.3

jhmvol16no12_patel06711215e_f1.jpg

Catheter Ablation Plus Antiarrhythmics

Ongoing AADs following CA may suppress AF triggers, especially in patients with persistent AF or high-risk for recurrence post ablation (left atrial dilation). The AMIO-CAT trial found that 4 weeks of amiodarone after ablation reduced early AF recurrence at 3 months (34% vs 53%, P = .006), arrhythmia-related hospitalizations, and need for cardioversion in patients with paroxysmal and persistent AF.8 However, amiodarone did not reduce recurrent atrial tachyarrhythmias at 6 months. The POWDER-AF trial evaluated AAD use for 1 year after CA in patients with drug-refractory paroxysmal AF.9 Continuation of class IC (eg, flecainide) and III (eg, amiodarone) AADs resulted in a near 20% absolute risk reduction in recurrent atrial arrhythmias and reduced the need for repeat CA. These trials suggest that discharging patients on adjunctive AADs decreases early recurrence of AF and arrhythmia-related hospitalizations; however, studies evaluating additional clinical outcomes are needed.

Heart Failure

The AATAC trial found CA was superior to amiodarone therapy at maintaining freedom from AF and reducing unplanned hospitalizations and mortality among patients with persistent AF and HFrEF.10 The larger CASTLE-AF trial randomized patients with an ejection fraction below 35% and NYHA class II or greater symptoms with symptomatic paroxysmal AF or persistent AF in whom AAD therapy failed to CA or medical therapy.11 The CA group experienced lower cardiovascular mortality (11.2% vs 22.3%, P = .009) and fewer heart failure hospitalizations (20.7% vs 35.9%, P = .004). The subsequent AMICA trial did not find a benefit of CA in patients with HFrEF and persistent or long-standing persistent AF; however, this trial was limited to 12 months, whereas the benefit of CA in CASTLE-AF was observed after 12 months.12 Also, AMICA enrolled patients with higher NYHA class. Therefore, hospitalists should refer AF patients with left ventricular systolic dysfunction and NYHA II or III symptoms for CA. Comparing AMICA and CASTLE-AF suggests earlier referral for CA, prior to the development of worsening heart failure symptoms, may improve outcomes.

Data for patients with heart failure with preserved EF (HFpEF) is limited. One small trial showed reduced heart failure hospitalizations in HFpEF patients treated with CA compared to AADs or beta-blockers.13 It is reasonable to refer HFpEF patients with persisting symptoms or reduced quality of life for CA.

What long-term risk-modification should I recommend?

The AHA Scientific Statement on Lifestyle and Risk Factor Modification for Reduction of Atrial Fibrillation delineates risk factors that increase the incidence of AF, including alcohol consumption, obstructive sleep apnea, hypertension, and obesity.14 Among regular alcohol consumers with paroxysmal or persistent AF managed with a rhythm-control strategy, cessation of alcohol has been shown to significantly lower the incidence of recurrent AF (53.0% vs 73.0%, P = .005), and lead to a longer time until recurrence of AF compared to patients regularly consuming alcohol.15 Among patients with obstructive sleep apnea, a systematic review of nonrandomized studies showed continuous positive airway pressure is associated with maintenance of sinus rhythm.14 Control of these risk factors is associated with up to approximately 40% of patients maintaining sinus rhythm without intervention, and hospitalists should encourage lifestyle modification to maximize the probability of maintaining sinus rhythm. 

Summary

Hospitalists frequently determine the best initial management strategy for patients admitted with new-onset AF, and recent literature may shift more patients towards management with rhythm control. Based on the trials reviewed in this Progress Note, hospitalists should recommend a rhythm-control strategy for patients with symptomatic, paroxysmal, or persistent AF of <12 months’ duration and refer patients with HFrEF for CA. Adherence to guideline recommendations is essential when prescribing AADs to avoid adverse drug events. It is vital to ensure patients managed with a rhythm-control strategy receive anticoagulation for 4 weeks post cardioversion or 2 months post CA with long-term anticoagulation based on CHA2DS2-VASc score. Finally, admissions for AF should serve as a catalyst to communicate to patients the importance of addressing obstructive sleep apnea, obesity, and alcohol use disorders. Applying these evidence-based practices will enable hospitalists to make clinical decisions that improve symptom burden and survival for patients with AF.

 

References

1. Wyse DG, Waldo AL, DiMarco JP, et al. A comparison of rate control and rhythm control in patients with atrial fibrillation. N Engl J Med. 2002;347(23):1825-1833. https://doi.org/10.1056/NEJMoa021328

2. Corley SD, Epstein AE, DiMarco JP, et al. Relationships between sinus rhythm, treatment, and survival in the Atrial Fibrillation Follow-Up Investigation of Rhythm Management (AFFIRM) Study. Circulation. 2004;109(12):1509-1513. https://doi.org/10.1161/01.Cir.0000121736.16643.11

3. January CT, Wann LS, Alpert JS, et al. 2014 AHA/ACC/HRS guideline for the management of patients with atrial fibrillation. Circulation. 2014;130(23):e199-e267. https://doi.org/10.1161/CIR.0000000000000041

4. Kirchhof P, Camm AJ, Goette A, et al. Early rhythm-control therapy in patients with atrial fibrillation. N Engl J Med. 2020;383(14):1305-1316. https://doi.org/10.1056/NEJMoa2019422

5. Packer DL, Mark DB, Robb RA, et al. Effect of catheter ablation vs antiarrhythmic drug therapy on mortality, stroke, bleeding, and cardiac arrest among patients with atrial fibrillation: the CABANA randomized clinical trial. JAMA. 2019;321(13):1261-1274. https://doi.org/doi:10.1001/jama.2019.0693

6. Wazni OM, Dandamudi G, Sood N, et al. Cryoballoon ablation as initial therapy for atrial fibrillation. N Engl J Med. 2021;384(4):316-324. https://doi.org/10.1056/NEJMoa2029554

7. Andrade JG, Wells GA, Deyell MW, et al. Cryoablation or drug therapy for initial treatment of atrial fibrillation. N Engl J Med. 2021;384(4):305-315. https://doi.org/10.1056/NEJMoa2029980

8. Darkner S, Chen X, Hansen J, et al. Recurrence of arrhythmia following short-term oral AMIOdarone after CATheter ablation for atrial fibrillation: a double-blind, randomized, placebo-controlled study (AMIO-CAT trial). Eur Heart J. 2014;35(47):3356-3364. https://doi.org/10.1093/eurheartj/ehu354

9. Duytschaever M, Demolder A, Phlips T, et al. PulmOnary vein isolation with vs. without continued antiarrhythmic drug treatment in subjects with recurrent atrial fibrillation (POWDER AF): results from a multicentre randomized trial. Eur Heart J. 2018;39(16):1429-1437. https://doi.org/10.1093/eurheartj/ehx666

10. Di Biase L, Mohanty P, Mohanty S, et al. Ablation versus amiodarone for treatment of persistent atrial fibrillation in patients with congestive heart failure and an implanted device: results from the AATAC multicenter randomized trial. Circulation. 2016;133(17):1637-1344. https://doi.org/10.1161/circulationaha.115.019406

11. Marrouche NF, Brachmann J, Andresen D, et al. Catheter ablation for atrial fibrillation with heart failure. N Engl J Med. 2018;378(5):417-427. https://doi.org/10.1056/NEJMoa1707855

12. Kuck KH, Merkely B, Zahn R, et al. Catheter ablation versus best medical therapy in patients with persistent atrial fibrillation and congestive heart failure: the randomized AMICA Trial. Circ Arrhythm Electrophysiol. 2019;12(12):e007731. d https://doi.org/10.1161/circep.119.007731

13. Fukui A, Tanino T, Yamaguchi T, et al. Catheter ablation of atrial fibrillation reduces heart failure rehospitalization in patients with heart failure with preserved ejection fraction. J Cardiovasc Electrophysiol. 2020;31(3):682-688. https://doi.org/10.1111/jce.14369

14. Chung MK, Eckhardt LL, Chen LY, et al. Lifestyle and risk factor modification for reduction of atrial fibrillation: a scientific statement from the American Heart Association. Circulation. 2020;141(16):e750-e772. https://doi.org/10.1161/CIR.0000000000000748

15. Voskoboinik A, Kalman JM, De Silva A, et al. Alcohol abstinence in drinkers with atrial fibrillation. N Engl J Med. 2020;382(1):20-28. https://doi.org/10.1056/NEJMoa1817591

References

1. Wyse DG, Waldo AL, DiMarco JP, et al. A comparison of rate control and rhythm control in patients with atrial fibrillation. N Engl J Med. 2002;347(23):1825-1833. https://doi.org/10.1056/NEJMoa021328

2. Corley SD, Epstein AE, DiMarco JP, et al. Relationships between sinus rhythm, treatment, and survival in the Atrial Fibrillation Follow-Up Investigation of Rhythm Management (AFFIRM) Study. Circulation. 2004;109(12):1509-1513. https://doi.org/10.1161/01.Cir.0000121736.16643.11

3. January CT, Wann LS, Alpert JS, et al. 2014 AHA/ACC/HRS guideline for the management of patients with atrial fibrillation. Circulation. 2014;130(23):e199-e267. https://doi.org/10.1161/CIR.0000000000000041

4. Kirchhof P, Camm AJ, Goette A, et al. Early rhythm-control therapy in patients with atrial fibrillation. N Engl J Med. 2020;383(14):1305-1316. https://doi.org/10.1056/NEJMoa2019422

5. Packer DL, Mark DB, Robb RA, et al. Effect of catheter ablation vs antiarrhythmic drug therapy on mortality, stroke, bleeding, and cardiac arrest among patients with atrial fibrillation: the CABANA randomized clinical trial. JAMA. 2019;321(13):1261-1274. https://doi.org/doi:10.1001/jama.2019.0693

6. Wazni OM, Dandamudi G, Sood N, et al. Cryoballoon ablation as initial therapy for atrial fibrillation. N Engl J Med. 2021;384(4):316-324. https://doi.org/10.1056/NEJMoa2029554

7. Andrade JG, Wells GA, Deyell MW, et al. Cryoablation or drug therapy for initial treatment of atrial fibrillation. N Engl J Med. 2021;384(4):305-315. https://doi.org/10.1056/NEJMoa2029980

8. Darkner S, Chen X, Hansen J, et al. Recurrence of arrhythmia following short-term oral AMIOdarone after CATheter ablation for atrial fibrillation: a double-blind, randomized, placebo-controlled study (AMIO-CAT trial). Eur Heart J. 2014;35(47):3356-3364. https://doi.org/10.1093/eurheartj/ehu354

9. Duytschaever M, Demolder A, Phlips T, et al. PulmOnary vein isolation with vs. without continued antiarrhythmic drug treatment in subjects with recurrent atrial fibrillation (POWDER AF): results from a multicentre randomized trial. Eur Heart J. 2018;39(16):1429-1437. https://doi.org/10.1093/eurheartj/ehx666

10. Di Biase L, Mohanty P, Mohanty S, et al. Ablation versus amiodarone for treatment of persistent atrial fibrillation in patients with congestive heart failure and an implanted device: results from the AATAC multicenter randomized trial. Circulation. 2016;133(17):1637-1344. https://doi.org/10.1161/circulationaha.115.019406

11. Marrouche NF, Brachmann J, Andresen D, et al. Catheter ablation for atrial fibrillation with heart failure. N Engl J Med. 2018;378(5):417-427. https://doi.org/10.1056/NEJMoa1707855

12. Kuck KH, Merkely B, Zahn R, et al. Catheter ablation versus best medical therapy in patients with persistent atrial fibrillation and congestive heart failure: the randomized AMICA Trial. Circ Arrhythm Electrophysiol. 2019;12(12):e007731. d https://doi.org/10.1161/circep.119.007731

13. Fukui A, Tanino T, Yamaguchi T, et al. Catheter ablation of atrial fibrillation reduces heart failure rehospitalization in patients with heart failure with preserved ejection fraction. J Cardiovasc Electrophysiol. 2020;31(3):682-688. https://doi.org/10.1111/jce.14369

14. Chung MK, Eckhardt LL, Chen LY, et al. Lifestyle and risk factor modification for reduction of atrial fibrillation: a scientific statement from the American Heart Association. Circulation. 2020;141(16):e750-e772. https://doi.org/10.1161/CIR.0000000000000748

15. Voskoboinik A, Kalman JM, De Silva A, et al. Alcohol abstinence in drinkers with atrial fibrillation. N Engl J Med. 2020;382(1):20-28. https://doi.org/10.1056/NEJMoa1817591

Publications
Publications
Topics
Article Type
Sections
Article Source

© 2021 Society of Hospital Medicine

Citation Override
J Hosp Med. Published Online First December 15, 2021. DOI: 12788/jhm.3734
Disallow All Ads
Correspondence Location
E-mail: rajiv.patel@cuanschutz.edu; Telephone: 215-817-7988.
Content Gating
Gated (full article locked unless allowed per User)
Alternative CME
Disqus Comments
Default
Use ProPublica
Hide sidebar & use full width
render the right sidebar.
Conference Recap Checkbox
Not Conference Recap
Clinical Edge
Display the Slideshow in this Article
Gating Strategy
First Page Free
Medscape Article
Display survey writer
Reuters content
Disable Inline Native ads
WebMD Article
Article PDF Media

Beyond a Purple Journal: Improving Hospital-Based Addiction Care

Article Type
Changed
Wed, 12/15/2021 - 01:15
Display Headline
Beyond a Purple Journal: Improving Hospital-Based Addiction Care

Rosa* was one of my first patients as an intern rotating at the county hospital. Her marriage had disintegrated years earlier. To cope with depression, she hid a daily ritual of orange juice and vodka from her children. She worked as a cashier, until nausea and fatigue overwhelmed her.

The first time I met her she sat on the gurney: petite, tanned, and pregnant. Then I saw her yellow eyes and revised: temporal wasting, jaundiced, and swollen with ascites. Rosa didn’t know that alcohol could cause liver disease. Without insurance or access to primary care, her untreated alcohol use disorder (AUD) and depression had snowballed for years. 

Midway through my intern year, I’d taken care of many people with AUD. However, I’d barely learned anything about it as a medical student, though we’d spent weeks studying esoteric diseases, that now––9 years after medical school––I still have not encountered. 

Among the 28.3 million individuals in the United States with AUD, only 1% receive medication treatment.1 In the United States, unhealthy alcohol use accounts for more than 95,000 deaths each year.2 This number likely under-captures alcohol-related mortality and is higher now given recent reports of increasing alcohol-related deaths and prevalence of unhealthy alcohol use, especially among women, younger age groups, and marginalized populations.3-5 

Rosa had alcohol-related hepatitis, which can cause severe inflammation and liver failure and quickly lead to death. As her liver failure progressed, I asked the gastroenterologists, “What other treatments can we offer? Is she a liver transplant candidate?” “Nothing” and “No” they answered.

Later, I emailed the hepatologist and transplant surgeon begging them to reevaluate her transplantation candidacy, but they told me there was no exception to the institution’s 6-month sobriety rule.

Maintaining a 6-month sobriety period is not an evidence-based criterion for transplantation. However, 50% of transplant centers do not perform transplantation prior to 6 months of alcohol abstinence for alcohol-related hepatitis due to concern for return to drinking after transplant.6 This practice may promote bias in patient selection for transplantation. A recent study found that individuals with alcohol-related liver disease transplanted before 6 months of abstinence had similar rates of survival and return to drinking compared to those who abstained from alcohol for 6 months and participated in AUD treatment before transplantation.7

There are other liver transplant practices that result in inequities for individuals with substance use disorders (SUD). Some liver transplant centers consider being on a medication for opioid use disorder a contraindication for transplantation—even if the individual is in recovery and abstaining from substances.8 Others mandate that individuals with alcohol-related liver disease attend Alcoholics Anonymous (AA) meetings prior to transplant. While mutual help groups, including AA, may benefit some individuals, different approaches work for different people.9 Other psychosocial interventions (eg, cognitive-behavioral therapy, contingency management, and residential treatment) and medications also help individuals reduce or stop drinking. Some meet their goals without any treatment. Addiction care works best when it respects autonomy and meets individuals where they are by allowing them to decide among options.

While organ allocations are a crystalized example of inequities in addiction care, they are also ethically complex. Many individuals—with and without SUD—die on waiting lists and must meet stringent transplantation criteria. However, we can at least remove the unnecessary biases that compound inequities in care people with SUD already face.

As Rosa’s liver succumbed, her kidneys failed too, and she required dialysis. She sensed what was coming. “I want everything…for now. I need to take care of my children.” I, too, wanted Rosa to live and see her youngest start kindergarten.

A few days before her discharge, I walked to the pharmacy and bought a purple journal. In a rare moment, I found Rosa alone in her room, without her ex-husband, sister, and mother, who rarely left her bedside. Together, we called AA and explored whether she could start participating in phone meetings from the hospital. I explained that one way to document a commitment to sobriety, as the transplant center’s rules dictated, was to attend and document AA meetings in this notebook. “In 5 months, you will be a liver transplant candidate,” I remember saying, wishing it to fruition.

I became Rosa’s primary care physician and saw her in clinic. Over the next few weeks, her skin took on an ashen tone. Sleep escaped her and her thoughts and speech blurred. Her walk slowed and she needed a wheelchair. The quiet fierceness that had defined her dissipated as encephalopathy took over. But until our last visit, she brought her purple journal, tracking the AA meetings she’d attended. Dialysis became intolerable, but not before Rosa made care arrangements for her girls. When that happened, she stopped dialysis and went to Mexico, where she died in her sleep after saying good-bye to her father. 

Earlier access to healthcare and effective depression and AUD treatment could have saved Rosa’s life. While it was too late for her, as hospitalists we care for many others with substance-related complications and may miss opportunities to discuss and offer evidence-based addiction treatment. For example, we initiate the most up-to-date management for a patient’s gastrointestinal bleed but may leave the alcohol discussion for someone else. It is similar for other SUD: we treat cellulitis, epidural abscesses, bacteremia, chronic obstructive pulmonary disease, heart failure exacerbations, and other complications of SUD without addressing the root cause of the hospitalization—other than to prescribe abstinence from substance use or, at our worst, scold individuals for continuing to use.

But what can we offer? Most healthcare professionals still do not receive addiction education during training. Without tools, we enact temporizing measures, until patients return to the hospital or die.

In addition to increasing alcohol-related morbidity, there have also been increases in drug-related overdoses, fueled by COVID-19, synthetic opioids like fentanyl, and stimulants.10 In the 12-month period ending April 2021, more than 100,000 individuals died of drug-related overdoses, the highest number of deaths ever recorded in a year.11 Despite this, most healthcare systems remain unequipped to provide addiction services during hospitalization due to inadequate training, stigma, and lack of systems-based care.

Hospitalists and healthcare systems cannot be bystanders amid our worsening addiction crisis. We must empower clinicians with addiction education and ensure health systems offer evidence-based SUD services.

Educational efforts can close the knowledge gaps for both medical students and hospitalists. Medical schools should include foundational curricular content in screening, assessing, diagnosing, and treating SUD in alignment with standards set by the Liaison Committee on Medical Education, which accredits US medical schools. Residency programs can offer educational conferences, cased-based discussions, and addiction medicine rotations. Hospitalists can participate in educational didactics and review evidence-based addiction guidelines.12,13 While the focus here is on hospitalists, clinicians across practice settings and specialties will encounter patients with SUD, and all need to be well-versed in the diagnosis and treatment of addiction given the all-hands-on deck approach necessary amidst our worsening addiction crisis.

With one in nine hospitalizations involving individuals with SUD, and this number quickly rising, and with an annual cost to US hospitals of $13.2 billion, healthcare system leaders must invest in addiction care.14,15 Hospital-based addiction services could pay for themselves and save healthcare systems money while improving the patient and clinician experience.16One way to implement hospital-based addiction care is through an addiction consult team (ACT).17 While ACT compositions vary, most are interprofessional, offer evidence-based addiction treatment, and connect patients to community care.18 Our hospital’s ACT has nurses, patient navigators, and physicians who assess, diagnose, and treat SUD, and arrange follow-up addiction care.19 In addition to caring for individual patients, our ACT has led systems change. For example, we created order sets to guide clinicians, added medications to our hospital formulary to ensure access to evidence-based addiction treatment, and partnered with community stakeholders to streamline care transitions and access to psychosocial and medication treatment. Our team also worked with hospital leadership, nursing, and a syringe service program to integrate hospital harm reduction education and supply provision. Additionally, we are building capacity among staff, trainees, and clinicians through education and systems changes.

In hospitals without an ACT, leadership can finance SUD champions and integrate them into policy-level decision-making to implement best practices in addiction care and lead hospital-wide educational efforts. This will transform hospital culture and improve care as all clinicians develop essential addiction skills.

Addiction champions and ACTs could also advocate for equitable practices for patients with SUD to reduce the stigma that both prevents patients from seeking care and results in self-discharges.20 For example, with interprofessional support, we revised our in-hospital substance use policy. It previously entailed hospital security responding to substance use concerns, which unintentionally harmed patients and perpetuated stigma. Our revised policy ensures we offer medications for cravings and withdrawal, adequate pain management, and other services that address patients’ reasons for in-hospital substance use.

With the increasing prevalence of SUD among hospitalized patients, escalating substance-related deaths, rising healthcare costs, and the impact of addiction on health and well-being, addiction care, including ACTs and champions, must be adequately funded. However, sustainable financing remains a challenge.18

Caring for Rosa and others with SUD sparked my desire to learn about addiction, obtain addiction medicine board certification as a practicing hospitalist, and create an ACT that offers evidence-based addiction treatment. While much remains to be done, by collaborating with addiction champions and engaging hospital leadership, we have transformed our hospital’s approach to substance use care.

With the knowledge and resources I now have as an addiction medicine physician, I reimagine the possibilities for patients like Rosa.

Rosa died when living was possible.

*Name has been changed for patient privacy.

References

1. Substance Abuse and Mental Health Services Administration. Key substance use and mental health indicators in the United States: Results from the 2020 National Survey on Drug Use and Health. HHS Publication No. PEP21-07-01-003, NSDUH Series H-56. Rockville, MD: Center for Behavioral Health Statistics and Quality, Substance Abuse and Mental Health Services Administration. Accessed December 1, 2021. www.samhsa.gov/data/
2. Centers for Disease Control and Prevention. Alcohol and public health: alcohol-related disease impact (ARDI) application, 2013. Average for United States 2006–2010 alcohol-attributable deaths due to excessive alcohol use. Accessed December 1, 2021. www.cdc.gov/ARDI
3. Spillane S, Shiels MS, Best AF, et al. Trends in alcohol-induced deaths in the United States, 2000-2016. JAMA Netw Open. 2020;3(2):e1921451. https://doi.org/ 10.1001/jamanetworkopen.2019.21451
4. Grant BF, Chou SP, Saha TD, et al. Prevalence of 12-month alcohol use, high-risk drinking, and DSM-IV alcohol use disorder in the United States, 2001-2002 to 2012-2013: results from the National Epidemiologic Survey on Alcohol and Related Conditions. JAMA Psychiatry. 2017;74(9):911-923. https://doi.org/10.1001/jamapsychiatry.2017.2161 https://doi.org/10.1001/jamapsychiatry.2017.2161
5. Pollard MS, Tucker JS, Green HD Jr. Changes in adult alcohol use and consequences during the covid-19 pandemic in the US. JAMA Netw Open. 2020;3(9):e2022942. https://doi.org/10.1001/jamanetworkopen.2020.22942
6. Bangaru S, Pedersen MR, Macconmara MP, Singal AG, Mufti AR. Survey of liver transplantation practices for severe acute alcoholic hepatitis. Liver Transpl. 2018;24(10):1357-1362. https://doi.org/10.1002/lt.25285
7. Herrick-Reynolds KM, Punchhi G, Greenberg RS, et al. Evaluation of early vs standard liver transplant for alcohol-associated liver disease. JAMA Surg. 2021;156(11):1026-1034. https://doi.org/10.1001/jamasurg.2021.3748
8. Fleming JN, Lai JC, Te HS, Said A, Spengler EK, Rogal SS. Opioid and opioid substitution therapy in liver transplant candidates: A survey of center policies and practices. Clin Transplant. 2017;31(12):e13119. https://doi.org/10.1111/ctr.13119
9. Klimas J, Fairgrieve C, Tobin H, et al. Psychosocial interventions to reduce alcohol consumption in concurrent problem alcohol and illicit drug users. Cochrane Database Syst Rev. 2018;12(12):CD009269. https://doi.org/10.1002/14651858.CD009269.pub4
10. Mattson CL, Tanz LJ, Quinn K, Kariisa M, Patel P, Davis NL. Trends and geographic patterns in drug and synthetic opioid overdose deaths—United States, 2013–2019. MMWR Morb Mortal Wkly Rep. 2021;70:202–207. https://doi.org/10.15585/mmwr.mm7006a4
11. Ahmad FB, Rossen LM, Sutton P. Provisional drug overdose death counts. National Center for Health Statistics. Accessed November 18, 2021. www.cdc.gov/nchs/nvss/vsrr/drug-overdose-data.htm
12. Englander H, Priest KC, Snyder H, Martin M, Calcaterra S, Gregg J. A call to action: hospitalists’ role in addressing substance use disorder. J Hosp Med. 2020;15(3):184-187. https://doi.org/10.12788/jhm.3311
13. California Bridge Program. Tools: Treat substance use disorders from the acute care setting. Accessed August 20, 2021. https://cabridge.org/tools
14. Peterson C, Li M, Xu L, Mikosz CA, Luo F. Assessment of annual cost of substance use disorder in US hospitals. JAMA Netw Open. 2021;4(3):e210242. https://doi.org/10.1001/jamanetworkopen.2021.0242
15. Suen LW, Makam AN, Snyder HR, et al. National prevalence of alcohol and other substance use disorders among emergency department visits and hospitalizations: NHAMCS 2014-2018. J Gen Intern Med. 2021;13:1-9. https://doi.org/10.1007/s11606-021-07069-w
16. Englander H, Collins D, Perry SP, Rabinowitz M, Phoutrides E, Nicolaidis C. “We’ve learned it’s a medical illness, not a moral choice”: Qualitative study of the effects of a multicomponent addiction intervention on hospital providers’ attitudes and experiences. J Hosp Med. 2018;13(11):752-758. https://doi.org/10.12788/jhm.2993
17. Priest KC, McCarty D. Making the business case for an addiction medicine consult service: a qualitative analysis. BMC Health Services Research. 2019;19(1):822. https://doi.org/10.1186/s12913-019-4670-4
18. Priest KC, McCarty D. Role of the hospital in the 21st century opioid overdose epidemic: the addiction medicine consult service. J Addict Med. 2019;13(2):104-112. https://doi.org/10.1097/ADM.0000000000000496
19. Martin M, Snyder HR, Coffa D, et al. Time to ACT: launching an Addiction Care Team (ACT) in an urban safety-net health system. BMJ Open Qual. 2021;10(1):e001111. https://doi.org/10.1136/bmjoq-2020-001111
20. Simon R, Snow R, Wakeman S. Understanding why patients with substance use disorders leave the hospital against medical advice: A qualitative study. Subst Abus. 2020;41(4):519-525. https://doi.org/10.1080/08897077.2019.1671942

Article PDF
Author and Disclosure Information

Department of Medicine, University of California, San Francisco, San Francisco, CA; San Francisco General Hospital, San Francisco, CA.

Publications
Topics
Sections
Author and Disclosure Information

Department of Medicine, University of California, San Francisco, San Francisco, CA; San Francisco General Hospital, San Francisco, CA.

Author and Disclosure Information

Department of Medicine, University of California, San Francisco, San Francisco, CA; San Francisco General Hospital, San Francisco, CA.

Article PDF
Article PDF
Related Articles

Rosa* was one of my first patients as an intern rotating at the county hospital. Her marriage had disintegrated years earlier. To cope with depression, she hid a daily ritual of orange juice and vodka from her children. She worked as a cashier, until nausea and fatigue overwhelmed her.

The first time I met her she sat on the gurney: petite, tanned, and pregnant. Then I saw her yellow eyes and revised: temporal wasting, jaundiced, and swollen with ascites. Rosa didn’t know that alcohol could cause liver disease. Without insurance or access to primary care, her untreated alcohol use disorder (AUD) and depression had snowballed for years. 

Midway through my intern year, I’d taken care of many people with AUD. However, I’d barely learned anything about it as a medical student, though we’d spent weeks studying esoteric diseases, that now––9 years after medical school––I still have not encountered. 

Among the 28.3 million individuals in the United States with AUD, only 1% receive medication treatment.1 In the United States, unhealthy alcohol use accounts for more than 95,000 deaths each year.2 This number likely under-captures alcohol-related mortality and is higher now given recent reports of increasing alcohol-related deaths and prevalence of unhealthy alcohol use, especially among women, younger age groups, and marginalized populations.3-5 

Rosa had alcohol-related hepatitis, which can cause severe inflammation and liver failure and quickly lead to death. As her liver failure progressed, I asked the gastroenterologists, “What other treatments can we offer? Is she a liver transplant candidate?” “Nothing” and “No” they answered.

Later, I emailed the hepatologist and transplant surgeon begging them to reevaluate her transplantation candidacy, but they told me there was no exception to the institution’s 6-month sobriety rule.

Maintaining a 6-month sobriety period is not an evidence-based criterion for transplantation. However, 50% of transplant centers do not perform transplantation prior to 6 months of alcohol abstinence for alcohol-related hepatitis due to concern for return to drinking after transplant.6 This practice may promote bias in patient selection for transplantation. A recent study found that individuals with alcohol-related liver disease transplanted before 6 months of abstinence had similar rates of survival and return to drinking compared to those who abstained from alcohol for 6 months and participated in AUD treatment before transplantation.7

There are other liver transplant practices that result in inequities for individuals with substance use disorders (SUD). Some liver transplant centers consider being on a medication for opioid use disorder a contraindication for transplantation—even if the individual is in recovery and abstaining from substances.8 Others mandate that individuals with alcohol-related liver disease attend Alcoholics Anonymous (AA) meetings prior to transplant. While mutual help groups, including AA, may benefit some individuals, different approaches work for different people.9 Other psychosocial interventions (eg, cognitive-behavioral therapy, contingency management, and residential treatment) and medications also help individuals reduce or stop drinking. Some meet their goals without any treatment. Addiction care works best when it respects autonomy and meets individuals where they are by allowing them to decide among options.

While organ allocations are a crystalized example of inequities in addiction care, they are also ethically complex. Many individuals—with and without SUD—die on waiting lists and must meet stringent transplantation criteria. However, we can at least remove the unnecessary biases that compound inequities in care people with SUD already face.

As Rosa’s liver succumbed, her kidneys failed too, and she required dialysis. She sensed what was coming. “I want everything…for now. I need to take care of my children.” I, too, wanted Rosa to live and see her youngest start kindergarten.

A few days before her discharge, I walked to the pharmacy and bought a purple journal. In a rare moment, I found Rosa alone in her room, without her ex-husband, sister, and mother, who rarely left her bedside. Together, we called AA and explored whether she could start participating in phone meetings from the hospital. I explained that one way to document a commitment to sobriety, as the transplant center’s rules dictated, was to attend and document AA meetings in this notebook. “In 5 months, you will be a liver transplant candidate,” I remember saying, wishing it to fruition.

I became Rosa’s primary care physician and saw her in clinic. Over the next few weeks, her skin took on an ashen tone. Sleep escaped her and her thoughts and speech blurred. Her walk slowed and she needed a wheelchair. The quiet fierceness that had defined her dissipated as encephalopathy took over. But until our last visit, she brought her purple journal, tracking the AA meetings she’d attended. Dialysis became intolerable, but not before Rosa made care arrangements for her girls. When that happened, she stopped dialysis and went to Mexico, where she died in her sleep after saying good-bye to her father. 

Earlier access to healthcare and effective depression and AUD treatment could have saved Rosa’s life. While it was too late for her, as hospitalists we care for many others with substance-related complications and may miss opportunities to discuss and offer evidence-based addiction treatment. For example, we initiate the most up-to-date management for a patient’s gastrointestinal bleed but may leave the alcohol discussion for someone else. It is similar for other SUD: we treat cellulitis, epidural abscesses, bacteremia, chronic obstructive pulmonary disease, heart failure exacerbations, and other complications of SUD without addressing the root cause of the hospitalization—other than to prescribe abstinence from substance use or, at our worst, scold individuals for continuing to use.

But what can we offer? Most healthcare professionals still do not receive addiction education during training. Without tools, we enact temporizing measures, until patients return to the hospital or die.

In addition to increasing alcohol-related morbidity, there have also been increases in drug-related overdoses, fueled by COVID-19, synthetic opioids like fentanyl, and stimulants.10 In the 12-month period ending April 2021, more than 100,000 individuals died of drug-related overdoses, the highest number of deaths ever recorded in a year.11 Despite this, most healthcare systems remain unequipped to provide addiction services during hospitalization due to inadequate training, stigma, and lack of systems-based care.

Hospitalists and healthcare systems cannot be bystanders amid our worsening addiction crisis. We must empower clinicians with addiction education and ensure health systems offer evidence-based SUD services.

Educational efforts can close the knowledge gaps for both medical students and hospitalists. Medical schools should include foundational curricular content in screening, assessing, diagnosing, and treating SUD in alignment with standards set by the Liaison Committee on Medical Education, which accredits US medical schools. Residency programs can offer educational conferences, cased-based discussions, and addiction medicine rotations. Hospitalists can participate in educational didactics and review evidence-based addiction guidelines.12,13 While the focus here is on hospitalists, clinicians across practice settings and specialties will encounter patients with SUD, and all need to be well-versed in the diagnosis and treatment of addiction given the all-hands-on deck approach necessary amidst our worsening addiction crisis.

With one in nine hospitalizations involving individuals with SUD, and this number quickly rising, and with an annual cost to US hospitals of $13.2 billion, healthcare system leaders must invest in addiction care.14,15 Hospital-based addiction services could pay for themselves and save healthcare systems money while improving the patient and clinician experience.16One way to implement hospital-based addiction care is through an addiction consult team (ACT).17 While ACT compositions vary, most are interprofessional, offer evidence-based addiction treatment, and connect patients to community care.18 Our hospital’s ACT has nurses, patient navigators, and physicians who assess, diagnose, and treat SUD, and arrange follow-up addiction care.19 In addition to caring for individual patients, our ACT has led systems change. For example, we created order sets to guide clinicians, added medications to our hospital formulary to ensure access to evidence-based addiction treatment, and partnered with community stakeholders to streamline care transitions and access to psychosocial and medication treatment. Our team also worked with hospital leadership, nursing, and a syringe service program to integrate hospital harm reduction education and supply provision. Additionally, we are building capacity among staff, trainees, and clinicians through education and systems changes.

In hospitals without an ACT, leadership can finance SUD champions and integrate them into policy-level decision-making to implement best practices in addiction care and lead hospital-wide educational efforts. This will transform hospital culture and improve care as all clinicians develop essential addiction skills.

Addiction champions and ACTs could also advocate for equitable practices for patients with SUD to reduce the stigma that both prevents patients from seeking care and results in self-discharges.20 For example, with interprofessional support, we revised our in-hospital substance use policy. It previously entailed hospital security responding to substance use concerns, which unintentionally harmed patients and perpetuated stigma. Our revised policy ensures we offer medications for cravings and withdrawal, adequate pain management, and other services that address patients’ reasons for in-hospital substance use.

With the increasing prevalence of SUD among hospitalized patients, escalating substance-related deaths, rising healthcare costs, and the impact of addiction on health and well-being, addiction care, including ACTs and champions, must be adequately funded. However, sustainable financing remains a challenge.18

Caring for Rosa and others with SUD sparked my desire to learn about addiction, obtain addiction medicine board certification as a practicing hospitalist, and create an ACT that offers evidence-based addiction treatment. While much remains to be done, by collaborating with addiction champions and engaging hospital leadership, we have transformed our hospital’s approach to substance use care.

With the knowledge and resources I now have as an addiction medicine physician, I reimagine the possibilities for patients like Rosa.

Rosa died when living was possible.

*Name has been changed for patient privacy.

Rosa* was one of my first patients as an intern rotating at the county hospital. Her marriage had disintegrated years earlier. To cope with depression, she hid a daily ritual of orange juice and vodka from her children. She worked as a cashier, until nausea and fatigue overwhelmed her.

The first time I met her she sat on the gurney: petite, tanned, and pregnant. Then I saw her yellow eyes and revised: temporal wasting, jaundiced, and swollen with ascites. Rosa didn’t know that alcohol could cause liver disease. Without insurance or access to primary care, her untreated alcohol use disorder (AUD) and depression had snowballed for years. 

Midway through my intern year, I’d taken care of many people with AUD. However, I’d barely learned anything about it as a medical student, though we’d spent weeks studying esoteric diseases, that now––9 years after medical school––I still have not encountered. 

Among the 28.3 million individuals in the United States with AUD, only 1% receive medication treatment.1 In the United States, unhealthy alcohol use accounts for more than 95,000 deaths each year.2 This number likely under-captures alcohol-related mortality and is higher now given recent reports of increasing alcohol-related deaths and prevalence of unhealthy alcohol use, especially among women, younger age groups, and marginalized populations.3-5 

Rosa had alcohol-related hepatitis, which can cause severe inflammation and liver failure and quickly lead to death. As her liver failure progressed, I asked the gastroenterologists, “What other treatments can we offer? Is she a liver transplant candidate?” “Nothing” and “No” they answered.

Later, I emailed the hepatologist and transplant surgeon begging them to reevaluate her transplantation candidacy, but they told me there was no exception to the institution’s 6-month sobriety rule.

Maintaining a 6-month sobriety period is not an evidence-based criterion for transplantation. However, 50% of transplant centers do not perform transplantation prior to 6 months of alcohol abstinence for alcohol-related hepatitis due to concern for return to drinking after transplant.6 This practice may promote bias in patient selection for transplantation. A recent study found that individuals with alcohol-related liver disease transplanted before 6 months of abstinence had similar rates of survival and return to drinking compared to those who abstained from alcohol for 6 months and participated in AUD treatment before transplantation.7

There are other liver transplant practices that result in inequities for individuals with substance use disorders (SUD). Some liver transplant centers consider being on a medication for opioid use disorder a contraindication for transplantation—even if the individual is in recovery and abstaining from substances.8 Others mandate that individuals with alcohol-related liver disease attend Alcoholics Anonymous (AA) meetings prior to transplant. While mutual help groups, including AA, may benefit some individuals, different approaches work for different people.9 Other psychosocial interventions (eg, cognitive-behavioral therapy, contingency management, and residential treatment) and medications also help individuals reduce or stop drinking. Some meet their goals without any treatment. Addiction care works best when it respects autonomy and meets individuals where they are by allowing them to decide among options.

While organ allocations are a crystalized example of inequities in addiction care, they are also ethically complex. Many individuals—with and without SUD—die on waiting lists and must meet stringent transplantation criteria. However, we can at least remove the unnecessary biases that compound inequities in care people with SUD already face.

As Rosa’s liver succumbed, her kidneys failed too, and she required dialysis. She sensed what was coming. “I want everything…for now. I need to take care of my children.” I, too, wanted Rosa to live and see her youngest start kindergarten.

A few days before her discharge, I walked to the pharmacy and bought a purple journal. In a rare moment, I found Rosa alone in her room, without her ex-husband, sister, and mother, who rarely left her bedside. Together, we called AA and explored whether she could start participating in phone meetings from the hospital. I explained that one way to document a commitment to sobriety, as the transplant center’s rules dictated, was to attend and document AA meetings in this notebook. “In 5 months, you will be a liver transplant candidate,” I remember saying, wishing it to fruition.

I became Rosa’s primary care physician and saw her in clinic. Over the next few weeks, her skin took on an ashen tone. Sleep escaped her and her thoughts and speech blurred. Her walk slowed and she needed a wheelchair. The quiet fierceness that had defined her dissipated as encephalopathy took over. But until our last visit, she brought her purple journal, tracking the AA meetings she’d attended. Dialysis became intolerable, but not before Rosa made care arrangements for her girls. When that happened, she stopped dialysis and went to Mexico, where she died in her sleep after saying good-bye to her father. 

Earlier access to healthcare and effective depression and AUD treatment could have saved Rosa’s life. While it was too late for her, as hospitalists we care for many others with substance-related complications and may miss opportunities to discuss and offer evidence-based addiction treatment. For example, we initiate the most up-to-date management for a patient’s gastrointestinal bleed but may leave the alcohol discussion for someone else. It is similar for other SUD: we treat cellulitis, epidural abscesses, bacteremia, chronic obstructive pulmonary disease, heart failure exacerbations, and other complications of SUD without addressing the root cause of the hospitalization—other than to prescribe abstinence from substance use or, at our worst, scold individuals for continuing to use.

But what can we offer? Most healthcare professionals still do not receive addiction education during training. Without tools, we enact temporizing measures, until patients return to the hospital or die.

In addition to increasing alcohol-related morbidity, there have also been increases in drug-related overdoses, fueled by COVID-19, synthetic opioids like fentanyl, and stimulants.10 In the 12-month period ending April 2021, more than 100,000 individuals died of drug-related overdoses, the highest number of deaths ever recorded in a year.11 Despite this, most healthcare systems remain unequipped to provide addiction services during hospitalization due to inadequate training, stigma, and lack of systems-based care.

Hospitalists and healthcare systems cannot be bystanders amid our worsening addiction crisis. We must empower clinicians with addiction education and ensure health systems offer evidence-based SUD services.

Educational efforts can close the knowledge gaps for both medical students and hospitalists. Medical schools should include foundational curricular content in screening, assessing, diagnosing, and treating SUD in alignment with standards set by the Liaison Committee on Medical Education, which accredits US medical schools. Residency programs can offer educational conferences, cased-based discussions, and addiction medicine rotations. Hospitalists can participate in educational didactics and review evidence-based addiction guidelines.12,13 While the focus here is on hospitalists, clinicians across practice settings and specialties will encounter patients with SUD, and all need to be well-versed in the diagnosis and treatment of addiction given the all-hands-on deck approach necessary amidst our worsening addiction crisis.

With one in nine hospitalizations involving individuals with SUD, and this number quickly rising, and with an annual cost to US hospitals of $13.2 billion, healthcare system leaders must invest in addiction care.14,15 Hospital-based addiction services could pay for themselves and save healthcare systems money while improving the patient and clinician experience.16One way to implement hospital-based addiction care is through an addiction consult team (ACT).17 While ACT compositions vary, most are interprofessional, offer evidence-based addiction treatment, and connect patients to community care.18 Our hospital’s ACT has nurses, patient navigators, and physicians who assess, diagnose, and treat SUD, and arrange follow-up addiction care.19 In addition to caring for individual patients, our ACT has led systems change. For example, we created order sets to guide clinicians, added medications to our hospital formulary to ensure access to evidence-based addiction treatment, and partnered with community stakeholders to streamline care transitions and access to psychosocial and medication treatment. Our team also worked with hospital leadership, nursing, and a syringe service program to integrate hospital harm reduction education and supply provision. Additionally, we are building capacity among staff, trainees, and clinicians through education and systems changes.

In hospitals without an ACT, leadership can finance SUD champions and integrate them into policy-level decision-making to implement best practices in addiction care and lead hospital-wide educational efforts. This will transform hospital culture and improve care as all clinicians develop essential addiction skills.

Addiction champions and ACTs could also advocate for equitable practices for patients with SUD to reduce the stigma that both prevents patients from seeking care and results in self-discharges.20 For example, with interprofessional support, we revised our in-hospital substance use policy. It previously entailed hospital security responding to substance use concerns, which unintentionally harmed patients and perpetuated stigma. Our revised policy ensures we offer medications for cravings and withdrawal, adequate pain management, and other services that address patients’ reasons for in-hospital substance use.

With the increasing prevalence of SUD among hospitalized patients, escalating substance-related deaths, rising healthcare costs, and the impact of addiction on health and well-being, addiction care, including ACTs and champions, must be adequately funded. However, sustainable financing remains a challenge.18

Caring for Rosa and others with SUD sparked my desire to learn about addiction, obtain addiction medicine board certification as a practicing hospitalist, and create an ACT that offers evidence-based addiction treatment. While much remains to be done, by collaborating with addiction champions and engaging hospital leadership, we have transformed our hospital’s approach to substance use care.

With the knowledge and resources I now have as an addiction medicine physician, I reimagine the possibilities for patients like Rosa.

Rosa died when living was possible.

*Name has been changed for patient privacy.

References

1. Substance Abuse and Mental Health Services Administration. Key substance use and mental health indicators in the United States: Results from the 2020 National Survey on Drug Use and Health. HHS Publication No. PEP21-07-01-003, NSDUH Series H-56. Rockville, MD: Center for Behavioral Health Statistics and Quality, Substance Abuse and Mental Health Services Administration. Accessed December 1, 2021. www.samhsa.gov/data/
2. Centers for Disease Control and Prevention. Alcohol and public health: alcohol-related disease impact (ARDI) application, 2013. Average for United States 2006–2010 alcohol-attributable deaths due to excessive alcohol use. Accessed December 1, 2021. www.cdc.gov/ARDI
3. Spillane S, Shiels MS, Best AF, et al. Trends in alcohol-induced deaths in the United States, 2000-2016. JAMA Netw Open. 2020;3(2):e1921451. https://doi.org/ 10.1001/jamanetworkopen.2019.21451
4. Grant BF, Chou SP, Saha TD, et al. Prevalence of 12-month alcohol use, high-risk drinking, and DSM-IV alcohol use disorder in the United States, 2001-2002 to 2012-2013: results from the National Epidemiologic Survey on Alcohol and Related Conditions. JAMA Psychiatry. 2017;74(9):911-923. https://doi.org/10.1001/jamapsychiatry.2017.2161 https://doi.org/10.1001/jamapsychiatry.2017.2161
5. Pollard MS, Tucker JS, Green HD Jr. Changes in adult alcohol use and consequences during the covid-19 pandemic in the US. JAMA Netw Open. 2020;3(9):e2022942. https://doi.org/10.1001/jamanetworkopen.2020.22942
6. Bangaru S, Pedersen MR, Macconmara MP, Singal AG, Mufti AR. Survey of liver transplantation practices for severe acute alcoholic hepatitis. Liver Transpl. 2018;24(10):1357-1362. https://doi.org/10.1002/lt.25285
7. Herrick-Reynolds KM, Punchhi G, Greenberg RS, et al. Evaluation of early vs standard liver transplant for alcohol-associated liver disease. JAMA Surg. 2021;156(11):1026-1034. https://doi.org/10.1001/jamasurg.2021.3748
8. Fleming JN, Lai JC, Te HS, Said A, Spengler EK, Rogal SS. Opioid and opioid substitution therapy in liver transplant candidates: A survey of center policies and practices. Clin Transplant. 2017;31(12):e13119. https://doi.org/10.1111/ctr.13119
9. Klimas J, Fairgrieve C, Tobin H, et al. Psychosocial interventions to reduce alcohol consumption in concurrent problem alcohol and illicit drug users. Cochrane Database Syst Rev. 2018;12(12):CD009269. https://doi.org/10.1002/14651858.CD009269.pub4
10. Mattson CL, Tanz LJ, Quinn K, Kariisa M, Patel P, Davis NL. Trends and geographic patterns in drug and synthetic opioid overdose deaths—United States, 2013–2019. MMWR Morb Mortal Wkly Rep. 2021;70:202–207. https://doi.org/10.15585/mmwr.mm7006a4
11. Ahmad FB, Rossen LM, Sutton P. Provisional drug overdose death counts. National Center for Health Statistics. Accessed November 18, 2021. www.cdc.gov/nchs/nvss/vsrr/drug-overdose-data.htm
12. Englander H, Priest KC, Snyder H, Martin M, Calcaterra S, Gregg J. A call to action: hospitalists’ role in addressing substance use disorder. J Hosp Med. 2020;15(3):184-187. https://doi.org/10.12788/jhm.3311
13. California Bridge Program. Tools: Treat substance use disorders from the acute care setting. Accessed August 20, 2021. https://cabridge.org/tools
14. Peterson C, Li M, Xu L, Mikosz CA, Luo F. Assessment of annual cost of substance use disorder in US hospitals. JAMA Netw Open. 2021;4(3):e210242. https://doi.org/10.1001/jamanetworkopen.2021.0242
15. Suen LW, Makam AN, Snyder HR, et al. National prevalence of alcohol and other substance use disorders among emergency department visits and hospitalizations: NHAMCS 2014-2018. J Gen Intern Med. 2021;13:1-9. https://doi.org/10.1007/s11606-021-07069-w
16. Englander H, Collins D, Perry SP, Rabinowitz M, Phoutrides E, Nicolaidis C. “We’ve learned it’s a medical illness, not a moral choice”: Qualitative study of the effects of a multicomponent addiction intervention on hospital providers’ attitudes and experiences. J Hosp Med. 2018;13(11):752-758. https://doi.org/10.12788/jhm.2993
17. Priest KC, McCarty D. Making the business case for an addiction medicine consult service: a qualitative analysis. BMC Health Services Research. 2019;19(1):822. https://doi.org/10.1186/s12913-019-4670-4
18. Priest KC, McCarty D. Role of the hospital in the 21st century opioid overdose epidemic: the addiction medicine consult service. J Addict Med. 2019;13(2):104-112. https://doi.org/10.1097/ADM.0000000000000496
19. Martin M, Snyder HR, Coffa D, et al. Time to ACT: launching an Addiction Care Team (ACT) in an urban safety-net health system. BMJ Open Qual. 2021;10(1):e001111. https://doi.org/10.1136/bmjoq-2020-001111
20. Simon R, Snow R, Wakeman S. Understanding why patients with substance use disorders leave the hospital against medical advice: A qualitative study. Subst Abus. 2020;41(4):519-525. https://doi.org/10.1080/08897077.2019.1671942

References

1. Substance Abuse and Mental Health Services Administration. Key substance use and mental health indicators in the United States: Results from the 2020 National Survey on Drug Use and Health. HHS Publication No. PEP21-07-01-003, NSDUH Series H-56. Rockville, MD: Center for Behavioral Health Statistics and Quality, Substance Abuse and Mental Health Services Administration. Accessed December 1, 2021. www.samhsa.gov/data/
2. Centers for Disease Control and Prevention. Alcohol and public health: alcohol-related disease impact (ARDI) application, 2013. Average for United States 2006–2010 alcohol-attributable deaths due to excessive alcohol use. Accessed December 1, 2021. www.cdc.gov/ARDI
3. Spillane S, Shiels MS, Best AF, et al. Trends in alcohol-induced deaths in the United States, 2000-2016. JAMA Netw Open. 2020;3(2):e1921451. https://doi.org/ 10.1001/jamanetworkopen.2019.21451
4. Grant BF, Chou SP, Saha TD, et al. Prevalence of 12-month alcohol use, high-risk drinking, and DSM-IV alcohol use disorder in the United States, 2001-2002 to 2012-2013: results from the National Epidemiologic Survey on Alcohol and Related Conditions. JAMA Psychiatry. 2017;74(9):911-923. https://doi.org/10.1001/jamapsychiatry.2017.2161 https://doi.org/10.1001/jamapsychiatry.2017.2161
5. Pollard MS, Tucker JS, Green HD Jr. Changes in adult alcohol use and consequences during the covid-19 pandemic in the US. JAMA Netw Open. 2020;3(9):e2022942. https://doi.org/10.1001/jamanetworkopen.2020.22942
6. Bangaru S, Pedersen MR, Macconmara MP, Singal AG, Mufti AR. Survey of liver transplantation practices for severe acute alcoholic hepatitis. Liver Transpl. 2018;24(10):1357-1362. https://doi.org/10.1002/lt.25285
7. Herrick-Reynolds KM, Punchhi G, Greenberg RS, et al. Evaluation of early vs standard liver transplant for alcohol-associated liver disease. JAMA Surg. 2021;156(11):1026-1034. https://doi.org/10.1001/jamasurg.2021.3748
8. Fleming JN, Lai JC, Te HS, Said A, Spengler EK, Rogal SS. Opioid and opioid substitution therapy in liver transplant candidates: A survey of center policies and practices. Clin Transplant. 2017;31(12):e13119. https://doi.org/10.1111/ctr.13119
9. Klimas J, Fairgrieve C, Tobin H, et al. Psychosocial interventions to reduce alcohol consumption in concurrent problem alcohol and illicit drug users. Cochrane Database Syst Rev. 2018;12(12):CD009269. https://doi.org/10.1002/14651858.CD009269.pub4
10. Mattson CL, Tanz LJ, Quinn K, Kariisa M, Patel P, Davis NL. Trends and geographic patterns in drug and synthetic opioid overdose deaths—United States, 2013–2019. MMWR Morb Mortal Wkly Rep. 2021;70:202–207. https://doi.org/10.15585/mmwr.mm7006a4
11. Ahmad FB, Rossen LM, Sutton P. Provisional drug overdose death counts. National Center for Health Statistics. Accessed November 18, 2021. www.cdc.gov/nchs/nvss/vsrr/drug-overdose-data.htm
12. Englander H, Priest KC, Snyder H, Martin M, Calcaterra S, Gregg J. A call to action: hospitalists’ role in addressing substance use disorder. J Hosp Med. 2020;15(3):184-187. https://doi.org/10.12788/jhm.3311
13. California Bridge Program. Tools: Treat substance use disorders from the acute care setting. Accessed August 20, 2021. https://cabridge.org/tools
14. Peterson C, Li M, Xu L, Mikosz CA, Luo F. Assessment of annual cost of substance use disorder in US hospitals. JAMA Netw Open. 2021;4(3):e210242. https://doi.org/10.1001/jamanetworkopen.2021.0242
15. Suen LW, Makam AN, Snyder HR, et al. National prevalence of alcohol and other substance use disorders among emergency department visits and hospitalizations: NHAMCS 2014-2018. J Gen Intern Med. 2021;13:1-9. https://doi.org/10.1007/s11606-021-07069-w
16. Englander H, Collins D, Perry SP, Rabinowitz M, Phoutrides E, Nicolaidis C. “We’ve learned it’s a medical illness, not a moral choice”: Qualitative study of the effects of a multicomponent addiction intervention on hospital providers’ attitudes and experiences. J Hosp Med. 2018;13(11):752-758. https://doi.org/10.12788/jhm.2993
17. Priest KC, McCarty D. Making the business case for an addiction medicine consult service: a qualitative analysis. BMC Health Services Research. 2019;19(1):822. https://doi.org/10.1186/s12913-019-4670-4
18. Priest KC, McCarty D. Role of the hospital in the 21st century opioid overdose epidemic: the addiction medicine consult service. J Addict Med. 2019;13(2):104-112. https://doi.org/10.1097/ADM.0000000000000496
19. Martin M, Snyder HR, Coffa D, et al. Time to ACT: launching an Addiction Care Team (ACT) in an urban safety-net health system. BMJ Open Qual. 2021;10(1):e001111. https://doi.org/10.1136/bmjoq-2020-001111
20. Simon R, Snow R, Wakeman S. Understanding why patients with substance use disorders leave the hospital against medical advice: A qualitative study. Subst Abus. 2020;41(4):519-525. https://doi.org/10.1080/08897077.2019.1671942

Publications
Publications
Topics
Article Type
Display Headline
Beyond a Purple Journal: Improving Hospital-Based Addiction Care
Display Headline
Beyond a Purple Journal: Improving Hospital-Based Addiction Care
Sections
Article Source

© 2021 Society of Hospital Medicine

Citation Override
J Hosp Med. Published Online Only December 15, 2021. DOI: 10.12788/jhm.3732
Disallow All Ads
Correspondence Location
Email: Marlene.Martin@ucsf.edu; Telephone: 628-206-4872.
Content Gating
Gated (full article locked unless allowed per User)
Alternative CME
Disqus Comments
Default
Use ProPublica
Hide sidebar & use full width
render the right sidebar.
Conference Recap Checkbox
Not Conference Recap
Clinical Edge
Display the Slideshow in this Article
Gating Strategy
First Page Free
Medscape Article
Display survey writer
Reuters content
Disable Inline Native ads
WebMD Article
Article PDF Media
Image
Teambase ID
18001F72.SIG
Disable zoom
Off
Media Folder

The Kids Are Not Alright

Article Type
Changed
Wed, 12/15/2021 - 01:15
Display Headline
The Kids Are Not Alright

“...but it all started to get worse during the pandemic.”

As the patient’s door closed, I (JS) thought about what his father had shared: his 12-year-old son had experienced a slow decline in his mental health since March 2020. There had been a gradual loss of all the things his son needed for psychological well-being: school went virtual and extracurricular activities ceased, and with them went any sense of routine, normalcy, or authentic opportunities to socialize. His feelings of isolation and depression culminated in an attempt to end his own life. My mind shifted to other patients under our care: an 8-year-old with behavioral outbursts intensifying after school-based therapy ended, a 13-year-old who became suicidal from isolation and virtual bullying. These children’s families sought emergent care because they no longer had the resources to care for their children at home. My team left each of these rooms heartbroken, unsure of exactly what to say and aware of the limitations of our current healthcare system.

Before and during the COVID-19 pandemic, many pediatric providers have had similar experiences caring for countless patients who are “boarding”—awaiting transfer to a psychiatric facility for their primary acute psychiatric issue, initially in the emergency room, often for 5 days or more,1 then ultimately admitted to a general medical floor if an appropriate psychiatric bed is still not available.2 Unfortunately, just as parents have run out of resources to care for their children’s psychiatric needs, so too is our medical system lacking in resources to provide the acute care these children need in general hospitals. 

This mental health crisis began before the COVID-19 pandemic3 but has only worsened in the wake of its resulting social isolation. During the pandemic, suicide hotlines had a 1000% increase in call volumes.4 COVID-19–induced bed closures simultaneously worsened an existing critical bed shortage5,6 and led to an increase in the average length of stay (LOS) for patients boarding in the emergency department (ED).7 In the state of Massachusetts, for example, psychiatric patients awaiting inpatient beds boarded for more than 10,000 hours in January 2021—more than ever before, and up approximately 4000 hours since January 2017.6 For pediatric patients, the average wait time is now 59 hours.6 In the first 6 months of the pandemic, 39% of children presenting to EDs for mental health complaints ended up boarding, which is an astounding figure and is unfortunately 7% higher than in 2019.8 Even these staggering numbers do not capture the full range of experiences, as many statistics do not account for time spent on inpatient units by patients who do not receive a bed placement after waiting hours to several days in the ED.

Shortages of space, as well as an underfunded and understaffed mental health workforce, lead to these prolonged, often traumatic boarding periods in hospitals designed to care for acute medical, rather than acute psychiatric, conditions. Patients awaiting psychiatric placement are waiting in settings that are chaotic, inconsistent, and lacking in privacy. A patient in the throes of psychosis or suicidality needs a therapeutic milieu, not one that interrupts their daily routine,2 disconnects them from their existing support networks, and is punctuated by the incessant clangs of bedside monitors and the hubbub of code teams. These environments are not therapeutic3 for young infants with fevers, let alone for teenagers battling suicidality and eating disorders. In fact, for these reasons, we suspect that many of our patients’ inpatient ”behavioral escalations” are in fact triggered by their hospital environment, which may contribute to the 300% increase in the number of pharmacological restraints used during mental health visits in the ED over the past 10 years.9

None of us imagined when we chose to pursue pediatrics a that significant—and at times predominant—portion of our training would encompass caring for patients with acute mental health concerns. And although we did not anticipate this crisis, we have now been tasked with managing it. Throughout the day, when we are called to see our patients with primarily psychiatric pathology, we are often at war with ourselves. We weigh forming deeply meaningful relationships with these patients against the potential of unintentionally retraumatizing them or forming bonds that will be abruptly severed when patients are transferred to a psychiatric facility, which often occurs with barely a few hours’ notice. Moreover, many healthcare workers have training ill-suited to meet the needs of these patients. Just as emergency physicians can diagnose appendicitis but rely on surgeons to provide timely surgical treatment, general pediatricians identify psychiatric crises but rely on psychiatrists for ideal treatment plans. And almost daily, we are called to an “escalating” patient and arrive minutes into a stressful situation that others expect us to extinguish expeditiously. Along with nursing colleagues and the behavioral response team, we enact the treatment plan laid out by our psychiatry colleagues and wonder whether there is a better way. 

We propose the following changes to create a more ideal health system (Table). We acknowledge that each health system has unique resources, challenges, and patient populations. Thus, our recommendations are not comprehensive and are largely based on experiences within our own institutions and state, but they encompass many domains that impact and are affected by child and adolescent mental healthcare in the United States, ranging from program- and hospital-level innovation to community and legislative action.

JHMVol16No12_Sundaresan06371215e.JPG

UPSTREAM PREVENTION

Like all good health system designs, we recommend prioritizing prevention. This would entail funding programs and legislation such as H.R. 3180, the RISE from Trauma Act, and H.R. 8544, the STRONG Support for Children Act of 2020 (both currently under consideration in the US House of Representatives) that support early childhood development and prevent adverse childhood experiences and trauma, averting mental health diagnoses such as depression and attention-deficit/hyperactivity disorder before they begin.10

OUTPATIENT AND COMMUNITY RESOURCES

We recognize that schools and general pediatricians have far more exposure to children at risk for mental health crises than do subspecialists. Thus, we urge an equitable increase in access to mental healthcare in the community so that patients needing assistance are screened and diagnosed earlier in their illness, allowing for secondary prevention of worsening mental health disorders. We support increased funding for programs such as the Massachusetts Child Psychiatry Access Program, which allows primary care doctors to consult psychiatrists in real time, closing the gap between a primary care visit and specialty follow-up. Telehealth services will be key to improving access for patients themselves and to allow pediatricians to consult with mental health professionals to initiate care prior to specialist availability. We envision that strengthening school-based behavioral health resources will also help prevent ED visits. Behavioral healthcare should be integrated into schools and community centers while police presence is simultaneously reduced, as there is evidence of an increased likelihood of juvenile justice involvement for children with disabilities and mental health needs.11,12

WORKFORCE DEVELOPMENT AND TRAINING

Ensuring access necessitates increasing the capacity of our psychiatric workforce by encouraging graduates to pursue mental health occupations with concrete financial incentives such as loan repayment and training grants. We thus support legislation such as H.R. 6597, the Mental Health Professionals Workforce Shortage Loan Repayment Act of 2018 (currently under consideration in the US House of Representatives). This may also improve recruitment and retention of individuals who are underrepresented in medicine, one step in helping ensure children have access to linguistically appropriate and culturally sensitive care. Residency programs and hospital systems should expand their training and education to identify and stabilize patients in mental health in extremis through culturally sensitive curricula focused on behavioral de-escalation techniques, trauma-informed care, and psychopharmacology. Our own residency program created a 2-week mental health rotation13 that includes rotating with outpatient mental health providers and our hospital’s behavioral response team, a group of trauma-informed responders for behavioral emergencies. Similar training should be available for nursing and other allied health professionals, who are often the first responders to behavioral escalations.13

INSTITUTIONAL DEVELOPMENT AND CLINICAL PRACTICES

Ideally, patients requiring higher-intensity psychiatric care would be referred to specialized pediatric behavioral health urgent care centers so their conditions can be adequately evaluated and addressed by staff trained in psychiatric management and in therapeutic environments. We believe all providers caring for children with mental health needs should be trained in basic, but core, behavioral health and de-escalation competencies, including specialized training for children with comorbid medical and neurodevelopmental diagnoses, such as autism. These centers should have specific beds for young children and those with developmental or complex care needs, and services should be available in numerous languages and levels of health literacy to allow all families to participate in their child’s care. At the same time, even nonpsychiatric EDs and inpatient units should commit resources to developing a maximally therapeutic environment, including allowing adjunctive services such as child life services, group therapy, and pet and music therapy, and create environments that support, rather than disrupt, normal routines.

HEALTH SYSTEMS REFORM AND ADVOCACY

Underpinning all the above innovations are changes to our healthcare payment system and provider networks, including the need for insurance coverage and payment parity for behavioral health, to ensure care is not only accessible but affordable. Additionally, for durable change, we need more than just education—we need coalition building and advocacy. Many organizations, including the American Academy of Pediatrics and the Children’s Hospital Association, have begun this work, which we must all continue.14 Bringing in diverse partners, including health systems, providers, educators, hospital administrators, payors, elected officials, and communities, will prioritize children’s needs and create a more ideal pediatric behavioral healthcare system.15

The COVID-19 pandemic has highlighted the dire need for comprehensive mental healthcare in the United States, a need that existed before the pandemic and will persist in a more fragile state long after it ends. Our hope is that the pandemic serves as the catalyst necessary to promote the magnitude of investments and stakeholder buy-in necessary to improve pediatric mental health and engender a radical redesign of our behavioral healthcare system. Our patients are counting on us to act. Together, we can build a system that ensures that the kids will be alright.

Patient details have been changed for patient privacy.

Acknowledgments

The authors thank Joanna Perdomo, MD, Amara Azubuike, JD, and Josh Greenberg, JD, for reading and providing feedback on earlier versions of this work.

References

1. “This is a crisis”: mom whose son has boarded 33 days for psych bed calls for state action. WBUR News. Updated March 2, 2021. Accessed August 4, 2021. www.wbur.org/news/2021/02/26/mental-health-boarding-hospitals
2. Moreno C, Wykes T, Galderisi S, et al. How mental health care should change as a consequence of the COVID-19 pandemic. Lancet Psychiatry. 2020;7(9):813-824. https://doi.org/10.1016/S2215-0366(20)30307-2
3. Nash KA, Zima BT, Rothenberg C, et al. Prolonged emergency department length of stay for US pediatric mental health visits (2005-2015). Pediatrics. 2021;147(5):e2020030692. https://doi.org/10.1542/peds.2020-030692
4. Cloutier RL, Marshaall R. A dangerous pandemic pair: Covid19 and adolescent mental health emergencies. Am J Emerg Med. 2021;46:776-777. https://doi.org/10.1016/j.ajem.2020.09.008
5. Schoenberg S. Lack of mental health beds means long ER waits. CommonWealth Magazine. April 15, 2021. Accessed August 5, 2021. https://commonwealthmagazine.org/health-care/lack-of-mental-health-beds-means-long-er-waits/
6. Jolicoeur L, Mullins L. Mass. physicians call on state to address ER “boarding” of patients awaiting admission. WBUR News. Updated February 3, 2021. Accessed August 5, 2021. www.wbur.org/news/2021/02/02/emergency-department-er-inpatient-beds-boarding
7. Krass P, Dalton E, Doupnik SK, Esposito J. US pediatric emergency department visits for mental health conditions during the COVID-19 pandemic. JAMA Netw Open. 2021;4(4):e218533. https://doi.org/10.1001/jamanetworkopen.2021.8533
8. Impact of COVID-19 on the Massachusetts Health Care System: Interim Report. Massachusetts Health Policy Commission. April 2021. Accessed September 25, 2021. www.mass.gov/doc/impact-of-covid-19-on-the-massachusetts-health-care-system-interim-report/download
9. Foster AA, Porter JJ, Monuteaux MC, Hoffmann JA, Hudgins JD. Pharmacologic restraint use during mental health visits in pediatric emergency departments. J Pediatr. 2021;236:276-283.e2. https://doi.org/10.1016/j.jpeds.2021.03.027
10. Brown NM, Brown SN, Briggs RD, Germán M, Belamarich PF, Oyeku SO. Associations between adverse childhood experiences and ADHD diagnosis and severity. Acad Pediatr. 2017;17(4):349-355. https://doi.org/10.1016/j.acap.2016.08.013
11. Harper K, Ryberg R, Temkin D. Black students and students with disabilities remain more likely to receive out-of-school suspensions, despite overall declines. Child Trends. April 29, 2019. Accessed August 5, 2021. www.childtrends.org/publications/black-students-disabilities-out-of-school-suspensions
12. Whitaker A, Torres-Guillén S, Morton M, et al. Cops and no counselors: how the lack of school mental health staff is harming students. American Civil Liberties Union. Accessed August 6, 2021. www.aclu.org/report/cops-and-no-counselors
13. Education. Boston Combined Residence Program. Accessed August 5, 2021. https://msbcrp.wpengine.com/program/education/
14. American Academy of Pediatrics. Interim guidance on supporting the emotional and behavioral health needs of children, adolescents, and families during the COVID-19 pandemic. Updated July 28, 2021. Accessed August 5, 2021. http://services.aap.org/en/pages/2019-novel-coronavirus-covid-19-infections/clinical-guidance/interim-guidance-on-supporting-the-emotional-and-behavioral-health-needs-of-children-adolescents-and-families-during-the-covid-19-pandemic/
15. Advocacy. Children’s Mental Health Campaign. Accessed August 4, 2021. https://childrensmentalhealthcampaign.org/advocacy

Article PDF
Author and Disclosure Information

1Department of Pediatrics, Boston Children’s Hospital, Boston, Massachusetts; 2Department of Pediatrics, Boston Medical Center, Boston, Massachusetts; 3Department of Pediatric Nephrology, Cincinnati Children’s Hospital Medical Center, Cincinnati, Ohio; 4Division of Emergency Medicine, Boston Children’s Hospital, Boston, Massachusetts.

Disclosures
The authors reported no conflicts of interest.

Publications
Topics
Sections
Author and Disclosure Information

1Department of Pediatrics, Boston Children’s Hospital, Boston, Massachusetts; 2Department of Pediatrics, Boston Medical Center, Boston, Massachusetts; 3Department of Pediatric Nephrology, Cincinnati Children’s Hospital Medical Center, Cincinnati, Ohio; 4Division of Emergency Medicine, Boston Children’s Hospital, Boston, Massachusetts.

Disclosures
The authors reported no conflicts of interest.

Author and Disclosure Information

1Department of Pediatrics, Boston Children’s Hospital, Boston, Massachusetts; 2Department of Pediatrics, Boston Medical Center, Boston, Massachusetts; 3Department of Pediatric Nephrology, Cincinnati Children’s Hospital Medical Center, Cincinnati, Ohio; 4Division of Emergency Medicine, Boston Children’s Hospital, Boston, Massachusetts.

Disclosures
The authors reported no conflicts of interest.

Article PDF
Article PDF
Related Articles

“...but it all started to get worse during the pandemic.”

As the patient’s door closed, I (JS) thought about what his father had shared: his 12-year-old son had experienced a slow decline in his mental health since March 2020. There had been a gradual loss of all the things his son needed for psychological well-being: school went virtual and extracurricular activities ceased, and with them went any sense of routine, normalcy, or authentic opportunities to socialize. His feelings of isolation and depression culminated in an attempt to end his own life. My mind shifted to other patients under our care: an 8-year-old with behavioral outbursts intensifying after school-based therapy ended, a 13-year-old who became suicidal from isolation and virtual bullying. These children’s families sought emergent care because they no longer had the resources to care for their children at home. My team left each of these rooms heartbroken, unsure of exactly what to say and aware of the limitations of our current healthcare system.

Before and during the COVID-19 pandemic, many pediatric providers have had similar experiences caring for countless patients who are “boarding”—awaiting transfer to a psychiatric facility for their primary acute psychiatric issue, initially in the emergency room, often for 5 days or more,1 then ultimately admitted to a general medical floor if an appropriate psychiatric bed is still not available.2 Unfortunately, just as parents have run out of resources to care for their children’s psychiatric needs, so too is our medical system lacking in resources to provide the acute care these children need in general hospitals. 

This mental health crisis began before the COVID-19 pandemic3 but has only worsened in the wake of its resulting social isolation. During the pandemic, suicide hotlines had a 1000% increase in call volumes.4 COVID-19–induced bed closures simultaneously worsened an existing critical bed shortage5,6 and led to an increase in the average length of stay (LOS) for patients boarding in the emergency department (ED).7 In the state of Massachusetts, for example, psychiatric patients awaiting inpatient beds boarded for more than 10,000 hours in January 2021—more than ever before, and up approximately 4000 hours since January 2017.6 For pediatric patients, the average wait time is now 59 hours.6 In the first 6 months of the pandemic, 39% of children presenting to EDs for mental health complaints ended up boarding, which is an astounding figure and is unfortunately 7% higher than in 2019.8 Even these staggering numbers do not capture the full range of experiences, as many statistics do not account for time spent on inpatient units by patients who do not receive a bed placement after waiting hours to several days in the ED.

Shortages of space, as well as an underfunded and understaffed mental health workforce, lead to these prolonged, often traumatic boarding periods in hospitals designed to care for acute medical, rather than acute psychiatric, conditions. Patients awaiting psychiatric placement are waiting in settings that are chaotic, inconsistent, and lacking in privacy. A patient in the throes of psychosis or suicidality needs a therapeutic milieu, not one that interrupts their daily routine,2 disconnects them from their existing support networks, and is punctuated by the incessant clangs of bedside monitors and the hubbub of code teams. These environments are not therapeutic3 for young infants with fevers, let alone for teenagers battling suicidality and eating disorders. In fact, for these reasons, we suspect that many of our patients’ inpatient ”behavioral escalations” are in fact triggered by their hospital environment, which may contribute to the 300% increase in the number of pharmacological restraints used during mental health visits in the ED over the past 10 years.9

None of us imagined when we chose to pursue pediatrics a that significant—and at times predominant—portion of our training would encompass caring for patients with acute mental health concerns. And although we did not anticipate this crisis, we have now been tasked with managing it. Throughout the day, when we are called to see our patients with primarily psychiatric pathology, we are often at war with ourselves. We weigh forming deeply meaningful relationships with these patients against the potential of unintentionally retraumatizing them or forming bonds that will be abruptly severed when patients are transferred to a psychiatric facility, which often occurs with barely a few hours’ notice. Moreover, many healthcare workers have training ill-suited to meet the needs of these patients. Just as emergency physicians can diagnose appendicitis but rely on surgeons to provide timely surgical treatment, general pediatricians identify psychiatric crises but rely on psychiatrists for ideal treatment plans. And almost daily, we are called to an “escalating” patient and arrive minutes into a stressful situation that others expect us to extinguish expeditiously. Along with nursing colleagues and the behavioral response team, we enact the treatment plan laid out by our psychiatry colleagues and wonder whether there is a better way. 

We propose the following changes to create a more ideal health system (Table). We acknowledge that each health system has unique resources, challenges, and patient populations. Thus, our recommendations are not comprehensive and are largely based on experiences within our own institutions and state, but they encompass many domains that impact and are affected by child and adolescent mental healthcare in the United States, ranging from program- and hospital-level innovation to community and legislative action.

JHMVol16No12_Sundaresan06371215e.JPG

UPSTREAM PREVENTION

Like all good health system designs, we recommend prioritizing prevention. This would entail funding programs and legislation such as H.R. 3180, the RISE from Trauma Act, and H.R. 8544, the STRONG Support for Children Act of 2020 (both currently under consideration in the US House of Representatives) that support early childhood development and prevent adverse childhood experiences and trauma, averting mental health diagnoses such as depression and attention-deficit/hyperactivity disorder before they begin.10

OUTPATIENT AND COMMUNITY RESOURCES

We recognize that schools and general pediatricians have far more exposure to children at risk for mental health crises than do subspecialists. Thus, we urge an equitable increase in access to mental healthcare in the community so that patients needing assistance are screened and diagnosed earlier in their illness, allowing for secondary prevention of worsening mental health disorders. We support increased funding for programs such as the Massachusetts Child Psychiatry Access Program, which allows primary care doctors to consult psychiatrists in real time, closing the gap between a primary care visit and specialty follow-up. Telehealth services will be key to improving access for patients themselves and to allow pediatricians to consult with mental health professionals to initiate care prior to specialist availability. We envision that strengthening school-based behavioral health resources will also help prevent ED visits. Behavioral healthcare should be integrated into schools and community centers while police presence is simultaneously reduced, as there is evidence of an increased likelihood of juvenile justice involvement for children with disabilities and mental health needs.11,12

WORKFORCE DEVELOPMENT AND TRAINING

Ensuring access necessitates increasing the capacity of our psychiatric workforce by encouraging graduates to pursue mental health occupations with concrete financial incentives such as loan repayment and training grants. We thus support legislation such as H.R. 6597, the Mental Health Professionals Workforce Shortage Loan Repayment Act of 2018 (currently under consideration in the US House of Representatives). This may also improve recruitment and retention of individuals who are underrepresented in medicine, one step in helping ensure children have access to linguistically appropriate and culturally sensitive care. Residency programs and hospital systems should expand their training and education to identify and stabilize patients in mental health in extremis through culturally sensitive curricula focused on behavioral de-escalation techniques, trauma-informed care, and psychopharmacology. Our own residency program created a 2-week mental health rotation13 that includes rotating with outpatient mental health providers and our hospital’s behavioral response team, a group of trauma-informed responders for behavioral emergencies. Similar training should be available for nursing and other allied health professionals, who are often the first responders to behavioral escalations.13

INSTITUTIONAL DEVELOPMENT AND CLINICAL PRACTICES

Ideally, patients requiring higher-intensity psychiatric care would be referred to specialized pediatric behavioral health urgent care centers so their conditions can be adequately evaluated and addressed by staff trained in psychiatric management and in therapeutic environments. We believe all providers caring for children with mental health needs should be trained in basic, but core, behavioral health and de-escalation competencies, including specialized training for children with comorbid medical and neurodevelopmental diagnoses, such as autism. These centers should have specific beds for young children and those with developmental or complex care needs, and services should be available in numerous languages and levels of health literacy to allow all families to participate in their child’s care. At the same time, even nonpsychiatric EDs and inpatient units should commit resources to developing a maximally therapeutic environment, including allowing adjunctive services such as child life services, group therapy, and pet and music therapy, and create environments that support, rather than disrupt, normal routines.

HEALTH SYSTEMS REFORM AND ADVOCACY

Underpinning all the above innovations are changes to our healthcare payment system and provider networks, including the need for insurance coverage and payment parity for behavioral health, to ensure care is not only accessible but affordable. Additionally, for durable change, we need more than just education—we need coalition building and advocacy. Many organizations, including the American Academy of Pediatrics and the Children’s Hospital Association, have begun this work, which we must all continue.14 Bringing in diverse partners, including health systems, providers, educators, hospital administrators, payors, elected officials, and communities, will prioritize children’s needs and create a more ideal pediatric behavioral healthcare system.15

The COVID-19 pandemic has highlighted the dire need for comprehensive mental healthcare in the United States, a need that existed before the pandemic and will persist in a more fragile state long after it ends. Our hope is that the pandemic serves as the catalyst necessary to promote the magnitude of investments and stakeholder buy-in necessary to improve pediatric mental health and engender a radical redesign of our behavioral healthcare system. Our patients are counting on us to act. Together, we can build a system that ensures that the kids will be alright.

Patient details have been changed for patient privacy.

Acknowledgments

The authors thank Joanna Perdomo, MD, Amara Azubuike, JD, and Josh Greenberg, JD, for reading and providing feedback on earlier versions of this work.

“...but it all started to get worse during the pandemic.”

As the patient’s door closed, I (JS) thought about what his father had shared: his 12-year-old son had experienced a slow decline in his mental health since March 2020. There had been a gradual loss of all the things his son needed for psychological well-being: school went virtual and extracurricular activities ceased, and with them went any sense of routine, normalcy, or authentic opportunities to socialize. His feelings of isolation and depression culminated in an attempt to end his own life. My mind shifted to other patients under our care: an 8-year-old with behavioral outbursts intensifying after school-based therapy ended, a 13-year-old who became suicidal from isolation and virtual bullying. These children’s families sought emergent care because they no longer had the resources to care for their children at home. My team left each of these rooms heartbroken, unsure of exactly what to say and aware of the limitations of our current healthcare system.

Before and during the COVID-19 pandemic, many pediatric providers have had similar experiences caring for countless patients who are “boarding”—awaiting transfer to a psychiatric facility for their primary acute psychiatric issue, initially in the emergency room, often for 5 days or more,1 then ultimately admitted to a general medical floor if an appropriate psychiatric bed is still not available.2 Unfortunately, just as parents have run out of resources to care for their children’s psychiatric needs, so too is our medical system lacking in resources to provide the acute care these children need in general hospitals. 

This mental health crisis began before the COVID-19 pandemic3 but has only worsened in the wake of its resulting social isolation. During the pandemic, suicide hotlines had a 1000% increase in call volumes.4 COVID-19–induced bed closures simultaneously worsened an existing critical bed shortage5,6 and led to an increase in the average length of stay (LOS) for patients boarding in the emergency department (ED).7 In the state of Massachusetts, for example, psychiatric patients awaiting inpatient beds boarded for more than 10,000 hours in January 2021—more than ever before, and up approximately 4000 hours since January 2017.6 For pediatric patients, the average wait time is now 59 hours.6 In the first 6 months of the pandemic, 39% of children presenting to EDs for mental health complaints ended up boarding, which is an astounding figure and is unfortunately 7% higher than in 2019.8 Even these staggering numbers do not capture the full range of experiences, as many statistics do not account for time spent on inpatient units by patients who do not receive a bed placement after waiting hours to several days in the ED.

Shortages of space, as well as an underfunded and understaffed mental health workforce, lead to these prolonged, often traumatic boarding periods in hospitals designed to care for acute medical, rather than acute psychiatric, conditions. Patients awaiting psychiatric placement are waiting in settings that are chaotic, inconsistent, and lacking in privacy. A patient in the throes of psychosis or suicidality needs a therapeutic milieu, not one that interrupts their daily routine,2 disconnects them from their existing support networks, and is punctuated by the incessant clangs of bedside monitors and the hubbub of code teams. These environments are not therapeutic3 for young infants with fevers, let alone for teenagers battling suicidality and eating disorders. In fact, for these reasons, we suspect that many of our patients’ inpatient ”behavioral escalations” are in fact triggered by their hospital environment, which may contribute to the 300% increase in the number of pharmacological restraints used during mental health visits in the ED over the past 10 years.9

None of us imagined when we chose to pursue pediatrics a that significant—and at times predominant—portion of our training would encompass caring for patients with acute mental health concerns. And although we did not anticipate this crisis, we have now been tasked with managing it. Throughout the day, when we are called to see our patients with primarily psychiatric pathology, we are often at war with ourselves. We weigh forming deeply meaningful relationships with these patients against the potential of unintentionally retraumatizing them or forming bonds that will be abruptly severed when patients are transferred to a psychiatric facility, which often occurs with barely a few hours’ notice. Moreover, many healthcare workers have training ill-suited to meet the needs of these patients. Just as emergency physicians can diagnose appendicitis but rely on surgeons to provide timely surgical treatment, general pediatricians identify psychiatric crises but rely on psychiatrists for ideal treatment plans. And almost daily, we are called to an “escalating” patient and arrive minutes into a stressful situation that others expect us to extinguish expeditiously. Along with nursing colleagues and the behavioral response team, we enact the treatment plan laid out by our psychiatry colleagues and wonder whether there is a better way. 

We propose the following changes to create a more ideal health system (Table). We acknowledge that each health system has unique resources, challenges, and patient populations. Thus, our recommendations are not comprehensive and are largely based on experiences within our own institutions and state, but they encompass many domains that impact and are affected by child and adolescent mental healthcare in the United States, ranging from program- and hospital-level innovation to community and legislative action.

JHMVol16No12_Sundaresan06371215e.JPG

UPSTREAM PREVENTION

Like all good health system designs, we recommend prioritizing prevention. This would entail funding programs and legislation such as H.R. 3180, the RISE from Trauma Act, and H.R. 8544, the STRONG Support for Children Act of 2020 (both currently under consideration in the US House of Representatives) that support early childhood development and prevent adverse childhood experiences and trauma, averting mental health diagnoses such as depression and attention-deficit/hyperactivity disorder before they begin.10

OUTPATIENT AND COMMUNITY RESOURCES

We recognize that schools and general pediatricians have far more exposure to children at risk for mental health crises than do subspecialists. Thus, we urge an equitable increase in access to mental healthcare in the community so that patients needing assistance are screened and diagnosed earlier in their illness, allowing for secondary prevention of worsening mental health disorders. We support increased funding for programs such as the Massachusetts Child Psychiatry Access Program, which allows primary care doctors to consult psychiatrists in real time, closing the gap between a primary care visit and specialty follow-up. Telehealth services will be key to improving access for patients themselves and to allow pediatricians to consult with mental health professionals to initiate care prior to specialist availability. We envision that strengthening school-based behavioral health resources will also help prevent ED visits. Behavioral healthcare should be integrated into schools and community centers while police presence is simultaneously reduced, as there is evidence of an increased likelihood of juvenile justice involvement for children with disabilities and mental health needs.11,12

WORKFORCE DEVELOPMENT AND TRAINING

Ensuring access necessitates increasing the capacity of our psychiatric workforce by encouraging graduates to pursue mental health occupations with concrete financial incentives such as loan repayment and training grants. We thus support legislation such as H.R. 6597, the Mental Health Professionals Workforce Shortage Loan Repayment Act of 2018 (currently under consideration in the US House of Representatives). This may also improve recruitment and retention of individuals who are underrepresented in medicine, one step in helping ensure children have access to linguistically appropriate and culturally sensitive care. Residency programs and hospital systems should expand their training and education to identify and stabilize patients in mental health in extremis through culturally sensitive curricula focused on behavioral de-escalation techniques, trauma-informed care, and psychopharmacology. Our own residency program created a 2-week mental health rotation13 that includes rotating with outpatient mental health providers and our hospital’s behavioral response team, a group of trauma-informed responders for behavioral emergencies. Similar training should be available for nursing and other allied health professionals, who are often the first responders to behavioral escalations.13

INSTITUTIONAL DEVELOPMENT AND CLINICAL PRACTICES

Ideally, patients requiring higher-intensity psychiatric care would be referred to specialized pediatric behavioral health urgent care centers so their conditions can be adequately evaluated and addressed by staff trained in psychiatric management and in therapeutic environments. We believe all providers caring for children with mental health needs should be trained in basic, but core, behavioral health and de-escalation competencies, including specialized training for children with comorbid medical and neurodevelopmental diagnoses, such as autism. These centers should have specific beds for young children and those with developmental or complex care needs, and services should be available in numerous languages and levels of health literacy to allow all families to participate in their child’s care. At the same time, even nonpsychiatric EDs and inpatient units should commit resources to developing a maximally therapeutic environment, including allowing adjunctive services such as child life services, group therapy, and pet and music therapy, and create environments that support, rather than disrupt, normal routines.

HEALTH SYSTEMS REFORM AND ADVOCACY

Underpinning all the above innovations are changes to our healthcare payment system and provider networks, including the need for insurance coverage and payment parity for behavioral health, to ensure care is not only accessible but affordable. Additionally, for durable change, we need more than just education—we need coalition building and advocacy. Many organizations, including the American Academy of Pediatrics and the Children’s Hospital Association, have begun this work, which we must all continue.14 Bringing in diverse partners, including health systems, providers, educators, hospital administrators, payors, elected officials, and communities, will prioritize children’s needs and create a more ideal pediatric behavioral healthcare system.15

The COVID-19 pandemic has highlighted the dire need for comprehensive mental healthcare in the United States, a need that existed before the pandemic and will persist in a more fragile state long after it ends. Our hope is that the pandemic serves as the catalyst necessary to promote the magnitude of investments and stakeholder buy-in necessary to improve pediatric mental health and engender a radical redesign of our behavioral healthcare system. Our patients are counting on us to act. Together, we can build a system that ensures that the kids will be alright.

Patient details have been changed for patient privacy.

Acknowledgments

The authors thank Joanna Perdomo, MD, Amara Azubuike, JD, and Josh Greenberg, JD, for reading and providing feedback on earlier versions of this work.

References

1. “This is a crisis”: mom whose son has boarded 33 days for psych bed calls for state action. WBUR News. Updated March 2, 2021. Accessed August 4, 2021. www.wbur.org/news/2021/02/26/mental-health-boarding-hospitals
2. Moreno C, Wykes T, Galderisi S, et al. How mental health care should change as a consequence of the COVID-19 pandemic. Lancet Psychiatry. 2020;7(9):813-824. https://doi.org/10.1016/S2215-0366(20)30307-2
3. Nash KA, Zima BT, Rothenberg C, et al. Prolonged emergency department length of stay for US pediatric mental health visits (2005-2015). Pediatrics. 2021;147(5):e2020030692. https://doi.org/10.1542/peds.2020-030692
4. Cloutier RL, Marshaall R. A dangerous pandemic pair: Covid19 and adolescent mental health emergencies. Am J Emerg Med. 2021;46:776-777. https://doi.org/10.1016/j.ajem.2020.09.008
5. Schoenberg S. Lack of mental health beds means long ER waits. CommonWealth Magazine. April 15, 2021. Accessed August 5, 2021. https://commonwealthmagazine.org/health-care/lack-of-mental-health-beds-means-long-er-waits/
6. Jolicoeur L, Mullins L. Mass. physicians call on state to address ER “boarding” of patients awaiting admission. WBUR News. Updated February 3, 2021. Accessed August 5, 2021. www.wbur.org/news/2021/02/02/emergency-department-er-inpatient-beds-boarding
7. Krass P, Dalton E, Doupnik SK, Esposito J. US pediatric emergency department visits for mental health conditions during the COVID-19 pandemic. JAMA Netw Open. 2021;4(4):e218533. https://doi.org/10.1001/jamanetworkopen.2021.8533
8. Impact of COVID-19 on the Massachusetts Health Care System: Interim Report. Massachusetts Health Policy Commission. April 2021. Accessed September 25, 2021. www.mass.gov/doc/impact-of-covid-19-on-the-massachusetts-health-care-system-interim-report/download
9. Foster AA, Porter JJ, Monuteaux MC, Hoffmann JA, Hudgins JD. Pharmacologic restraint use during mental health visits in pediatric emergency departments. J Pediatr. 2021;236:276-283.e2. https://doi.org/10.1016/j.jpeds.2021.03.027
10. Brown NM, Brown SN, Briggs RD, Germán M, Belamarich PF, Oyeku SO. Associations between adverse childhood experiences and ADHD diagnosis and severity. Acad Pediatr. 2017;17(4):349-355. https://doi.org/10.1016/j.acap.2016.08.013
11. Harper K, Ryberg R, Temkin D. Black students and students with disabilities remain more likely to receive out-of-school suspensions, despite overall declines. Child Trends. April 29, 2019. Accessed August 5, 2021. www.childtrends.org/publications/black-students-disabilities-out-of-school-suspensions
12. Whitaker A, Torres-Guillén S, Morton M, et al. Cops and no counselors: how the lack of school mental health staff is harming students. American Civil Liberties Union. Accessed August 6, 2021. www.aclu.org/report/cops-and-no-counselors
13. Education. Boston Combined Residence Program. Accessed August 5, 2021. https://msbcrp.wpengine.com/program/education/
14. American Academy of Pediatrics. Interim guidance on supporting the emotional and behavioral health needs of children, adolescents, and families during the COVID-19 pandemic. Updated July 28, 2021. Accessed August 5, 2021. http://services.aap.org/en/pages/2019-novel-coronavirus-covid-19-infections/clinical-guidance/interim-guidance-on-supporting-the-emotional-and-behavioral-health-needs-of-children-adolescents-and-families-during-the-covid-19-pandemic/
15. Advocacy. Children’s Mental Health Campaign. Accessed August 4, 2021. https://childrensmentalhealthcampaign.org/advocacy

References

1. “This is a crisis”: mom whose son has boarded 33 days for psych bed calls for state action. WBUR News. Updated March 2, 2021. Accessed August 4, 2021. www.wbur.org/news/2021/02/26/mental-health-boarding-hospitals
2. Moreno C, Wykes T, Galderisi S, et al. How mental health care should change as a consequence of the COVID-19 pandemic. Lancet Psychiatry. 2020;7(9):813-824. https://doi.org/10.1016/S2215-0366(20)30307-2
3. Nash KA, Zima BT, Rothenberg C, et al. Prolonged emergency department length of stay for US pediatric mental health visits (2005-2015). Pediatrics. 2021;147(5):e2020030692. https://doi.org/10.1542/peds.2020-030692
4. Cloutier RL, Marshaall R. A dangerous pandemic pair: Covid19 and adolescent mental health emergencies. Am J Emerg Med. 2021;46:776-777. https://doi.org/10.1016/j.ajem.2020.09.008
5. Schoenberg S. Lack of mental health beds means long ER waits. CommonWealth Magazine. April 15, 2021. Accessed August 5, 2021. https://commonwealthmagazine.org/health-care/lack-of-mental-health-beds-means-long-er-waits/
6. Jolicoeur L, Mullins L. Mass. physicians call on state to address ER “boarding” of patients awaiting admission. WBUR News. Updated February 3, 2021. Accessed August 5, 2021. www.wbur.org/news/2021/02/02/emergency-department-er-inpatient-beds-boarding
7. Krass P, Dalton E, Doupnik SK, Esposito J. US pediatric emergency department visits for mental health conditions during the COVID-19 pandemic. JAMA Netw Open. 2021;4(4):e218533. https://doi.org/10.1001/jamanetworkopen.2021.8533
8. Impact of COVID-19 on the Massachusetts Health Care System: Interim Report. Massachusetts Health Policy Commission. April 2021. Accessed September 25, 2021. www.mass.gov/doc/impact-of-covid-19-on-the-massachusetts-health-care-system-interim-report/download
9. Foster AA, Porter JJ, Monuteaux MC, Hoffmann JA, Hudgins JD. Pharmacologic restraint use during mental health visits in pediatric emergency departments. J Pediatr. 2021;236:276-283.e2. https://doi.org/10.1016/j.jpeds.2021.03.027
10. Brown NM, Brown SN, Briggs RD, Germán M, Belamarich PF, Oyeku SO. Associations between adverse childhood experiences and ADHD diagnosis and severity. Acad Pediatr. 2017;17(4):349-355. https://doi.org/10.1016/j.acap.2016.08.013
11. Harper K, Ryberg R, Temkin D. Black students and students with disabilities remain more likely to receive out-of-school suspensions, despite overall declines. Child Trends. April 29, 2019. Accessed August 5, 2021. www.childtrends.org/publications/black-students-disabilities-out-of-school-suspensions
12. Whitaker A, Torres-Guillén S, Morton M, et al. Cops and no counselors: how the lack of school mental health staff is harming students. American Civil Liberties Union. Accessed August 6, 2021. www.aclu.org/report/cops-and-no-counselors
13. Education. Boston Combined Residence Program. Accessed August 5, 2021. https://msbcrp.wpengine.com/program/education/
14. American Academy of Pediatrics. Interim guidance on supporting the emotional and behavioral health needs of children, adolescents, and families during the COVID-19 pandemic. Updated July 28, 2021. Accessed August 5, 2021. http://services.aap.org/en/pages/2019-novel-coronavirus-covid-19-infections/clinical-guidance/interim-guidance-on-supporting-the-emotional-and-behavioral-health-needs-of-children-adolescents-and-families-during-the-covid-19-pandemic/
15. Advocacy. Children’s Mental Health Campaign. Accessed August 4, 2021. https://childrensmentalhealthcampaign.org/advocacy

Publications
Publications
Topics
Article Type
Display Headline
The Kids Are Not Alright
Display Headline
The Kids Are Not Alright
Sections
Article Source

© 2021 Society of Hospital Medicine

Citation Override
J Hosp Med. Published Online Only December 15, 2021. DOI: 10.12788/jhm.3710
Disallow All Ads
Correspondence Location
H Rhodes Hambrick, MD; Email: rhodeshambrick@gmail.com; Telephone: 502-542-0398; Twitter: @rhodeshambrick.
Content Gating
Gated (full article locked unless allowed per User)
Alternative CME
Disqus Comments
Default
Use ProPublica
Hide sidebar & use full width
render the right sidebar.
Conference Recap Checkbox
Not Conference Recap
Clinical Edge
Display the Slideshow in this Article
Gating Strategy
First Page Free
Medscape Article
Display survey writer
Reuters content
Disable Inline Native ads
WebMD Article
Article PDF Media
Image
Teambase ID
18001F73.SIG
Disable zoom
Off

Things We Do for No Reason™: Discontinuing Urate-Lowering Therapy on Admission

Article Type
Changed
Wed, 12/15/2021 - 09:25
Display Headline
Things We Do for No Reason™: Discontinuing Urate-Lowering Therapy on Admission

Inspired by the ABIM Foundation’s Choosing Wisely® campaign, the “Things We Do for No Reason " (TWDFNR) series reviews practices that have become common parts of hospital care but may provide little value to our patients. Practices reviewed in the TWDFNR series do not represent clear-cut conclusions or clinical practice standards but are meant as a starting place for research and active discussions among hospitalists and patients. We invite you to be part of that discussion.

Clinical Scenario

An infected diabetic foot ulcer requiring intravenous antibiotics prompts admission for a 58-year-old man with hypertension, insulin-dependent diabetes mellitus, gout, stage 3 chronic kidney disease (CKD), and hyperlipidemia. On admission, the hospitalist discontinued the patient’s daily 300 mg of allopurinol, which had helped prevent a flare for more than 1 year. On day 3 of hospitalization, the patient developed right knee pain, swelling, and erythema. Due to concerns for septic arthritis, he underwent lab work, imaging, and joint aspiration, which confirmed the diagnosis of an acute gout flare. The prednisone he received for his gout flare caused hyperglycemia, requiring careful insulin titration during the remainder of his hospitalization.

Background

Gout, the most common form of inflammatory arthritis, affects 3.9% of the US population. Its incidence has doubled in the past 2 decades, partly due to an increase in risk factors for gout, including obesity, diabetes, hypertension, hyperlipidemia, and renal disease.1 Patients with gout incur high rates of hospitalization and costs related to the disease and its comorbidities.2 Volume depletion, diuretic use, fluid shifts, or discontinuation of gout medications put patients at high risk of developing acute flares during hospitalization.2-4

Acute inflammatory response to monosodium urate crystal deposition in joints causes gout flares. Over time, uncontrolled gout leads to chronic inflammatory damage, causing permanent deformities and disability. Patients with uncontrolled gout have decreased work productivity and higher healthcare utilization and costs than patients with controlled gout.5

Gout treatment has two components: acute flare management and long-term therapy to lower serum uric acid levels. Patients with frequent gout attacks (≥two annually), tophi, or radiographic damage require urate-lowering therapy (ULT) to prevent further damage. Additionally, ULT is conditionally recommended for patients with their first flare and concomitant CKD stage 3 or higher, serum uric acid >9 mg/dL, or urolithiasis. First-line ULT incorporates xanthine oxidase inhibitors, such as allopurinol, due to efficacy and low cost.6 Using a treat-to-target approach, allopurinol is titrated to achieve uric acid levels <6 mg/dL.6,7 Controlling gout can take many months and requires careful medication titration, lifestyle modifications, and clear communication with patients. Poor adherence to ULT treatment complicates overall gout control and partly results from patients’ and providers’ knowledge gaps about gout and gout medications.8,9 Prior studies demonstrated that poor adherence to ULT contributes to increased gout flares and resource utilization.6,9

Why You Might Think Stopping Urate-Lowering Therapy Is Helpful

In the authors’ experience, hospitalists discontinue ULT for three reasons. First, hospitalists hold ULT, particularly allopurinol, when a patient has either acute or chronic kidney injury, due to concern that decreased excretion of drug metabolites increases the risk of allopurinol hypersensitivity syndrome (AHS) and allopurinol toxicity.10 One small study reported a decrease or discontinuation of allopurinol in 21% of 73 admissions, citing concerns of using allopurinol in renal impairment.10 Oxipurinol, a renally excreted metabolite of allopurinol, accumulates at higher concentrations in individuals with kidney impairment. The belief that elevated concentrations increase the risk of adverse effects has guided past recommendations about safety and dosing of allopurinol in patients with CKD.11,12 Due to safety concerns, older guidelines and literature11 suggest not increasing allopurinol more than 300 mg daily in patients with CKD.

Second, clinicians may want to stop “nonessential” medications on admission in order to simplify a medication list. If a patient’s last gout flare occurred a long time ago, a clinician may think their gout no longer requires ULT.

Finally, ULT is discontinued during an acute gout flare because clinicians believe that continuing ULT will make flare symptoms worse. Allopurinol dissolves uric acid crystals, which can cause inflammation. The inflammation increases the risk of precipitating a gout flare when first starting allopurinol and during dose titration. Clinicians may feel that holding the medication during an acute flare avoids iatrogenesis that worsens the flare.

Why Stopping Urate-Lowering Therapy Is Not Helpful

While physicians cite concerns of using allopurinol in renal impairment,10 there are no absolute contraindications to allopurinol in kidney impairment. Clinicians can prescribe xanthine oxidase inhibitors to patients with moderate-to-severe CKD and can titrate allopurinol to doses greater than 300 mg daily safely in these same patients.6,7,12-14 Prior studies sparked concern that poor allopurinol metabolite excretion in CKD might contribute to AHS or toxicity. However, more recent studies show that patients with CKD can take allopurinol safely, but that they require slower up-titration to mitigate the risk of flares and AHS. Guidelines recommend a starting dose of ≤100 mg of allopurinol in patients with normal renal function, and even lower doses in patients with CKD.6 In studies showing safe dose titration in CKD, patients received an initial dose of allopurinol 50 mg daily, which increased by 50 mg every month.13,14 When hospitalists abruptly stop ULT during hospitalization in patients with CKD, those patients have to restart from the initial low dose and up-titrate slowly back to the lowest dose that achieves serum uric acid <6 mg/dL.6

Acute kidney injury (AKI) is not an absolute contraindication to allopurinol use, and the scant amount of published literature does not support discontinuation. In this acute situation, a patient may require a dose reduction in allopurinol to avoid toxicity depending on the severity of AKI. A discussion with inpatient pharmacy can help find a safe dose based on current creatinine clearance.

Physicians anecdotally recognize ULT discontinuation as a cause of inpatient gout flares. Clinicians and patients should view ULT as essential, even in patients who remain symptom-free for years. Between acute flares, a patient enters a potentially asymptomatic phase called “intercritical gout” that varies in duration. Urate deposition causing tophi and damage still occur during this phase, so patients must continue on ULT even if they have no recent flare history.

ULT that appears on any outpatient medication list needs verification of dose and compliance before ordering. If a patient is actually taking a lower dose than listed or not taking ULT at all, starting at a higher dose puts them at risk for flare and AHS, especially in patients with renal disease. Continuing ULT during hospitalization after verifying dose and compliance can potentially prevent gout flares and their downstream effects, including increased costs and potential side effects from additional pain medications.

Patients on chronic ULT should continue it during an acute gout flare.6,7 Literature and guidelines do not suggest that continuing ULT significantly worsens the intensity or duration of a flare. The initiation or up-titration of ULT, not the continuation of it, causes uric acid to dissolve, triggering an inflammatory response that increases the risk of gout flare. Therefore, guidelines recommend giving flare prophylaxis simultaneously for at least 3 to 6 months to prevent flares while starting and titrating ULT. Flare prophylaxis may continue longer depending on when a patient reaches a stable dose of ULT.6,7 While patients are receiving acute flare treatment, continuing ULT will help lower their serum uric acid levels over time.

To emphasize the importance of treating gout with ULT even further, the most recent American College of Rheumatology gout management guidelines conditionally recommend starting ULT during an acute flare for increased adherence. Small studies have shown that initiation of ULT does not precipitate attacks or significantly increase duration of flare. Input from patients influenced this recommendation, as they felt highly motivated to start ULT during acute flare due to symptoms.6

Additionally, due to comorbidities, inpatients often cannot tolerate standard flare therapies, such as nonsteroidal anti-inflammatory drugs, corticosteroids, or oral colchicine, to treat their acute symptoms. Moreover, patients often have other analgesics, such as opiates, prescribed for pain control. During an acute flare, hospitalists will likely need to add medications to treat the acute symptoms, but ULT should be considered an essential medication and continued as well.

When Stopping Urate-Lowering Therapy Might Be Helpful

Allopurinol can cause mild-to-severe cutaneous adverse reactions. AHS, a rare reaction that causes significant morbidity and mortality, presents with a rash, eosinophilia, fever, hepatitis, and progressive kidney failure. Risk factors for developing AHS include kidney impairment, higher starting doses, concurrent diuretic use, and presence of the genetic marker HLA B*5801.12 AHS usually occurs in the first 8 weeks of initiation of allopurinol, but can occur later in treatment, especially in those with risk factors—notably kidney impairment.12 When a patient on allopurinol develops a rash, the clinician should consider stopping allopurinol if concerned about AHS or, in milder cases, decrease the dose until the rash resolves.

What You Should Do Instead

When you see ULT on a patient’s medication list, verify the dose with the patient and continue it (even during an acute gout flare) unless a new rash has developed, or you are concerned about a drug-drug interaction. If a patient has a significant AKI, consider discussing dose modifications with your inpatient pharmacist.

Recommendations

  • Consider ULT an essential medication and continue it during the hospitalization of a patient with a history of gout.
  • Continue ULT while treating an acute gout flare.
  • Continue ULT in patients with AKI and CKD, but discuss dose modifications with a pharmacist for AKI patients.

Conclusion

In the clinical scenario, the hospitalist did not treat ULT as an essential medication on admission, and the patient’s gout flared, leading to increased morbidity, resource utilization, and cost of hospitalization. Stopping ULT has downstream effects after discharge, including delays in achieving prior gout control. If ULT is discontinued, outpatient clinicians must restart it at lower doses and then up-titrate slowly, increasing the risk of flares and possibly contributing to nonadherence. During hospitalization, clinicians should continue ULT.

Do you think this is a low-value practice? Is this truly a “Thing We Do for No Reason™”? Share what you do in your practice and join in the conversation online by retweeting it on Twitter (#TWDFNR) and liking it on Facebook. We invite you to propose ideas for other “Things We Do for No Reason™” topics by emailing TWDFNR@hospitalmedicine.org

References

1. Elfishawi MM, Zleik N, Kvrgic Z, et al. The rising incidence of gout and the increasing burden of comorbidities: a population-based study over 20 years. J Rheumatol. 2018;45(4):574-579. https://doi.org/10.3899/jrheum.170806
2. Fisher MC, Pillinger MH, Keenan RT. Inpatient gout: a review. Curr Rheumatol Rep. 2014;16(11):458. https://doi.org/10.1007/s11926-014-0458-z
3. Zleik N, Elfishawi MM, Kvrgic Z, et al. Hospitalization increases the risk of acute arthritic flares in gout: a population-based study over 2 decades. J Rheumatol. 2018;45(8):1188-1191. https://doi.org/10.3899/jrheum.171320
4. Dubreuil M, Neogi T, Chen CA, et al. Increased risk of recurrent gout attacks with hospitalization. Am J Med. 2013;126(12):1138-1141.e1. https://doi.org/10.1016/j.amjmed.2013.06.026
5. Flores NM, Neuvo J, Klein AB, Baumgartner S, Morlock R. The economic burden of uncontrolled gout: how controlling gout reduces cost. J Med Econ. 2019;22(1):1-6. https://doi.org/10.1080/13696998.2018.1532904
6. FitzGerald JD, Dalbeth N, Mikuls T, et al. 2020 American College of Rheumatology guideline for the management of gout. Arthritis Care Res (Hoboken). 2020;72(6):744-760. https://doi.org/10.1002/acr.24180
7. Khanna D, Khanna PP, FitzGerald JD, et al. 2012 American College of Rheumatology guidelines for management of gout. Part 2: therapy and antiinflammatory prophylaxis of acute gouty arthritis. Arthritis Care Res (Hoboken). 2012;64(10):1447-1461. https://doi.org/10.1002/acr.21773
8. Abhishek A, Doherty M. Education and non-pharmacological approaches for gout. Rheumatology (Oxford). 2018;57(suppl 1):i51-i58. https://doi.org/10.1093/rheumatology/kex421
9. Fields TR. The challenges of approaching and managing gout. Rheum Dis Clin North Am. 2019;45(1):145-157. https://doi.org/10.1016/j.rdc.2018.09.009
10. Huang IJ, Bays AM, Liew JW. Frequency of allopurinol dose reduction in hospitalized patients with gout flares. J Rheumatol. 2021;48(3):467-468. https://doi.org/10.3899/jrheum.201142
11. Hande KR, Noone RM, Stone WJ. Severe allopurinol toxicity. Description and guidelines for prevention in patients with renal insufficiency. Am J Med. 1984;76:47-56. https://doi.org/10.1016/0002-9343(84)90743-5
12. Stamp LK, Day RO, Yun J. Allopurinol hypersensitivity: investigating the cause and minimizing the risk. Nat Rev Rheumatol. 2016;12(4):235-242. https://doi.org/10.1038/nrrheum.2015.132
13. Stamp LK, Chapman PT, Barclay M, et al. The effect of kidney function on the urate lowering effect and safety of increasing allopurinol above doses based on creatinine clearance: a post hoc analysis of a randomized controlled trial. Arthritis Res Ther. 2017;19(1):283. https://doi.org/10.1186/s13075-017-1491-x
14. Stamp LK, O’Donnell JL, Zhang M, et al. Using allopurinol above the dose based on creatinine clearance is effective and safe in patients with chronic gout, including those with renal impairment. Arthritis Rheum. 2011;63(2):412-421. https://doi.org/10.1002/art.30119

Article PDF
Author and Disclosure Information

1Division of Rheumatology, Department of Medicine, University of Texas at Austin, Dell Medical School, Austin, Texas; 2Dell Medical School, Austin, Texas.

Disclosures
The authors reported no conflicts of interest.

Publications
Topics
Sections
Author and Disclosure Information

1Division of Rheumatology, Department of Medicine, University of Texas at Austin, Dell Medical School, Austin, Texas; 2Dell Medical School, Austin, Texas.

Disclosures
The authors reported no conflicts of interest.

Author and Disclosure Information

1Division of Rheumatology, Department of Medicine, University of Texas at Austin, Dell Medical School, Austin, Texas; 2Dell Medical School, Austin, Texas.

Disclosures
The authors reported no conflicts of interest.

Article PDF
Article PDF
Related Articles

Inspired by the ABIM Foundation’s Choosing Wisely® campaign, the “Things We Do for No Reason " (TWDFNR) series reviews practices that have become common parts of hospital care but may provide little value to our patients. Practices reviewed in the TWDFNR series do not represent clear-cut conclusions or clinical practice standards but are meant as a starting place for research and active discussions among hospitalists and patients. We invite you to be part of that discussion.

Clinical Scenario

An infected diabetic foot ulcer requiring intravenous antibiotics prompts admission for a 58-year-old man with hypertension, insulin-dependent diabetes mellitus, gout, stage 3 chronic kidney disease (CKD), and hyperlipidemia. On admission, the hospitalist discontinued the patient’s daily 300 mg of allopurinol, which had helped prevent a flare for more than 1 year. On day 3 of hospitalization, the patient developed right knee pain, swelling, and erythema. Due to concerns for septic arthritis, he underwent lab work, imaging, and joint aspiration, which confirmed the diagnosis of an acute gout flare. The prednisone he received for his gout flare caused hyperglycemia, requiring careful insulin titration during the remainder of his hospitalization.

Background

Gout, the most common form of inflammatory arthritis, affects 3.9% of the US population. Its incidence has doubled in the past 2 decades, partly due to an increase in risk factors for gout, including obesity, diabetes, hypertension, hyperlipidemia, and renal disease.1 Patients with gout incur high rates of hospitalization and costs related to the disease and its comorbidities.2 Volume depletion, diuretic use, fluid shifts, or discontinuation of gout medications put patients at high risk of developing acute flares during hospitalization.2-4

Acute inflammatory response to monosodium urate crystal deposition in joints causes gout flares. Over time, uncontrolled gout leads to chronic inflammatory damage, causing permanent deformities and disability. Patients with uncontrolled gout have decreased work productivity and higher healthcare utilization and costs than patients with controlled gout.5

Gout treatment has two components: acute flare management and long-term therapy to lower serum uric acid levels. Patients with frequent gout attacks (≥two annually), tophi, or radiographic damage require urate-lowering therapy (ULT) to prevent further damage. Additionally, ULT is conditionally recommended for patients with their first flare and concomitant CKD stage 3 or higher, serum uric acid >9 mg/dL, or urolithiasis. First-line ULT incorporates xanthine oxidase inhibitors, such as allopurinol, due to efficacy and low cost.6 Using a treat-to-target approach, allopurinol is titrated to achieve uric acid levels <6 mg/dL.6,7 Controlling gout can take many months and requires careful medication titration, lifestyle modifications, and clear communication with patients. Poor adherence to ULT treatment complicates overall gout control and partly results from patients’ and providers’ knowledge gaps about gout and gout medications.8,9 Prior studies demonstrated that poor adherence to ULT contributes to increased gout flares and resource utilization.6,9

Why You Might Think Stopping Urate-Lowering Therapy Is Helpful

In the authors’ experience, hospitalists discontinue ULT for three reasons. First, hospitalists hold ULT, particularly allopurinol, when a patient has either acute or chronic kidney injury, due to concern that decreased excretion of drug metabolites increases the risk of allopurinol hypersensitivity syndrome (AHS) and allopurinol toxicity.10 One small study reported a decrease or discontinuation of allopurinol in 21% of 73 admissions, citing concerns of using allopurinol in renal impairment.10 Oxipurinol, a renally excreted metabolite of allopurinol, accumulates at higher concentrations in individuals with kidney impairment. The belief that elevated concentrations increase the risk of adverse effects has guided past recommendations about safety and dosing of allopurinol in patients with CKD.11,12 Due to safety concerns, older guidelines and literature11 suggest not increasing allopurinol more than 300 mg daily in patients with CKD.

Second, clinicians may want to stop “nonessential” medications on admission in order to simplify a medication list. If a patient’s last gout flare occurred a long time ago, a clinician may think their gout no longer requires ULT.

Finally, ULT is discontinued during an acute gout flare because clinicians believe that continuing ULT will make flare symptoms worse. Allopurinol dissolves uric acid crystals, which can cause inflammation. The inflammation increases the risk of precipitating a gout flare when first starting allopurinol and during dose titration. Clinicians may feel that holding the medication during an acute flare avoids iatrogenesis that worsens the flare.

Why Stopping Urate-Lowering Therapy Is Not Helpful

While physicians cite concerns of using allopurinol in renal impairment,10 there are no absolute contraindications to allopurinol in kidney impairment. Clinicians can prescribe xanthine oxidase inhibitors to patients with moderate-to-severe CKD and can titrate allopurinol to doses greater than 300 mg daily safely in these same patients.6,7,12-14 Prior studies sparked concern that poor allopurinol metabolite excretion in CKD might contribute to AHS or toxicity. However, more recent studies show that patients with CKD can take allopurinol safely, but that they require slower up-titration to mitigate the risk of flares and AHS. Guidelines recommend a starting dose of ≤100 mg of allopurinol in patients with normal renal function, and even lower doses in patients with CKD.6 In studies showing safe dose titration in CKD, patients received an initial dose of allopurinol 50 mg daily, which increased by 50 mg every month.13,14 When hospitalists abruptly stop ULT during hospitalization in patients with CKD, those patients have to restart from the initial low dose and up-titrate slowly back to the lowest dose that achieves serum uric acid <6 mg/dL.6

Acute kidney injury (AKI) is not an absolute contraindication to allopurinol use, and the scant amount of published literature does not support discontinuation. In this acute situation, a patient may require a dose reduction in allopurinol to avoid toxicity depending on the severity of AKI. A discussion with inpatient pharmacy can help find a safe dose based on current creatinine clearance.

Physicians anecdotally recognize ULT discontinuation as a cause of inpatient gout flares. Clinicians and patients should view ULT as essential, even in patients who remain symptom-free for years. Between acute flares, a patient enters a potentially asymptomatic phase called “intercritical gout” that varies in duration. Urate deposition causing tophi and damage still occur during this phase, so patients must continue on ULT even if they have no recent flare history.

ULT that appears on any outpatient medication list needs verification of dose and compliance before ordering. If a patient is actually taking a lower dose than listed or not taking ULT at all, starting at a higher dose puts them at risk for flare and AHS, especially in patients with renal disease. Continuing ULT during hospitalization after verifying dose and compliance can potentially prevent gout flares and their downstream effects, including increased costs and potential side effects from additional pain medications.

Patients on chronic ULT should continue it during an acute gout flare.6,7 Literature and guidelines do not suggest that continuing ULT significantly worsens the intensity or duration of a flare. The initiation or up-titration of ULT, not the continuation of it, causes uric acid to dissolve, triggering an inflammatory response that increases the risk of gout flare. Therefore, guidelines recommend giving flare prophylaxis simultaneously for at least 3 to 6 months to prevent flares while starting and titrating ULT. Flare prophylaxis may continue longer depending on when a patient reaches a stable dose of ULT.6,7 While patients are receiving acute flare treatment, continuing ULT will help lower their serum uric acid levels over time.

To emphasize the importance of treating gout with ULT even further, the most recent American College of Rheumatology gout management guidelines conditionally recommend starting ULT during an acute flare for increased adherence. Small studies have shown that initiation of ULT does not precipitate attacks or significantly increase duration of flare. Input from patients influenced this recommendation, as they felt highly motivated to start ULT during acute flare due to symptoms.6

Additionally, due to comorbidities, inpatients often cannot tolerate standard flare therapies, such as nonsteroidal anti-inflammatory drugs, corticosteroids, or oral colchicine, to treat their acute symptoms. Moreover, patients often have other analgesics, such as opiates, prescribed for pain control. During an acute flare, hospitalists will likely need to add medications to treat the acute symptoms, but ULT should be considered an essential medication and continued as well.

When Stopping Urate-Lowering Therapy Might Be Helpful

Allopurinol can cause mild-to-severe cutaneous adverse reactions. AHS, a rare reaction that causes significant morbidity and mortality, presents with a rash, eosinophilia, fever, hepatitis, and progressive kidney failure. Risk factors for developing AHS include kidney impairment, higher starting doses, concurrent diuretic use, and presence of the genetic marker HLA B*5801.12 AHS usually occurs in the first 8 weeks of initiation of allopurinol, but can occur later in treatment, especially in those with risk factors—notably kidney impairment.12 When a patient on allopurinol develops a rash, the clinician should consider stopping allopurinol if concerned about AHS or, in milder cases, decrease the dose until the rash resolves.

What You Should Do Instead

When you see ULT on a patient’s medication list, verify the dose with the patient and continue it (even during an acute gout flare) unless a new rash has developed, or you are concerned about a drug-drug interaction. If a patient has a significant AKI, consider discussing dose modifications with your inpatient pharmacist.

Recommendations

  • Consider ULT an essential medication and continue it during the hospitalization of a patient with a history of gout.
  • Continue ULT while treating an acute gout flare.
  • Continue ULT in patients with AKI and CKD, but discuss dose modifications with a pharmacist for AKI patients.

Conclusion

In the clinical scenario, the hospitalist did not treat ULT as an essential medication on admission, and the patient’s gout flared, leading to increased morbidity, resource utilization, and cost of hospitalization. Stopping ULT has downstream effects after discharge, including delays in achieving prior gout control. If ULT is discontinued, outpatient clinicians must restart it at lower doses and then up-titrate slowly, increasing the risk of flares and possibly contributing to nonadherence. During hospitalization, clinicians should continue ULT.

Do you think this is a low-value practice? Is this truly a “Thing We Do for No Reason™”? Share what you do in your practice and join in the conversation online by retweeting it on Twitter (#TWDFNR) and liking it on Facebook. We invite you to propose ideas for other “Things We Do for No Reason™” topics by emailing TWDFNR@hospitalmedicine.org

Inspired by the ABIM Foundation’s Choosing Wisely® campaign, the “Things We Do for No Reason " (TWDFNR) series reviews practices that have become common parts of hospital care but may provide little value to our patients. Practices reviewed in the TWDFNR series do not represent clear-cut conclusions or clinical practice standards but are meant as a starting place for research and active discussions among hospitalists and patients. We invite you to be part of that discussion.

Clinical Scenario

An infected diabetic foot ulcer requiring intravenous antibiotics prompts admission for a 58-year-old man with hypertension, insulin-dependent diabetes mellitus, gout, stage 3 chronic kidney disease (CKD), and hyperlipidemia. On admission, the hospitalist discontinued the patient’s daily 300 mg of allopurinol, which had helped prevent a flare for more than 1 year. On day 3 of hospitalization, the patient developed right knee pain, swelling, and erythema. Due to concerns for septic arthritis, he underwent lab work, imaging, and joint aspiration, which confirmed the diagnosis of an acute gout flare. The prednisone he received for his gout flare caused hyperglycemia, requiring careful insulin titration during the remainder of his hospitalization.

Background

Gout, the most common form of inflammatory arthritis, affects 3.9% of the US population. Its incidence has doubled in the past 2 decades, partly due to an increase in risk factors for gout, including obesity, diabetes, hypertension, hyperlipidemia, and renal disease.1 Patients with gout incur high rates of hospitalization and costs related to the disease and its comorbidities.2 Volume depletion, diuretic use, fluid shifts, or discontinuation of gout medications put patients at high risk of developing acute flares during hospitalization.2-4

Acute inflammatory response to monosodium urate crystal deposition in joints causes gout flares. Over time, uncontrolled gout leads to chronic inflammatory damage, causing permanent deformities and disability. Patients with uncontrolled gout have decreased work productivity and higher healthcare utilization and costs than patients with controlled gout.5

Gout treatment has two components: acute flare management and long-term therapy to lower serum uric acid levels. Patients with frequent gout attacks (≥two annually), tophi, or radiographic damage require urate-lowering therapy (ULT) to prevent further damage. Additionally, ULT is conditionally recommended for patients with their first flare and concomitant CKD stage 3 or higher, serum uric acid >9 mg/dL, or urolithiasis. First-line ULT incorporates xanthine oxidase inhibitors, such as allopurinol, due to efficacy and low cost.6 Using a treat-to-target approach, allopurinol is titrated to achieve uric acid levels <6 mg/dL.6,7 Controlling gout can take many months and requires careful medication titration, lifestyle modifications, and clear communication with patients. Poor adherence to ULT treatment complicates overall gout control and partly results from patients’ and providers’ knowledge gaps about gout and gout medications.8,9 Prior studies demonstrated that poor adherence to ULT contributes to increased gout flares and resource utilization.6,9

Why You Might Think Stopping Urate-Lowering Therapy Is Helpful

In the authors’ experience, hospitalists discontinue ULT for three reasons. First, hospitalists hold ULT, particularly allopurinol, when a patient has either acute or chronic kidney injury, due to concern that decreased excretion of drug metabolites increases the risk of allopurinol hypersensitivity syndrome (AHS) and allopurinol toxicity.10 One small study reported a decrease or discontinuation of allopurinol in 21% of 73 admissions, citing concerns of using allopurinol in renal impairment.10 Oxipurinol, a renally excreted metabolite of allopurinol, accumulates at higher concentrations in individuals with kidney impairment. The belief that elevated concentrations increase the risk of adverse effects has guided past recommendations about safety and dosing of allopurinol in patients with CKD.11,12 Due to safety concerns, older guidelines and literature11 suggest not increasing allopurinol more than 300 mg daily in patients with CKD.

Second, clinicians may want to stop “nonessential” medications on admission in order to simplify a medication list. If a patient’s last gout flare occurred a long time ago, a clinician may think their gout no longer requires ULT.

Finally, ULT is discontinued during an acute gout flare because clinicians believe that continuing ULT will make flare symptoms worse. Allopurinol dissolves uric acid crystals, which can cause inflammation. The inflammation increases the risk of precipitating a gout flare when first starting allopurinol and during dose titration. Clinicians may feel that holding the medication during an acute flare avoids iatrogenesis that worsens the flare.

Why Stopping Urate-Lowering Therapy Is Not Helpful

While physicians cite concerns of using allopurinol in renal impairment,10 there are no absolute contraindications to allopurinol in kidney impairment. Clinicians can prescribe xanthine oxidase inhibitors to patients with moderate-to-severe CKD and can titrate allopurinol to doses greater than 300 mg daily safely in these same patients.6,7,12-14 Prior studies sparked concern that poor allopurinol metabolite excretion in CKD might contribute to AHS or toxicity. However, more recent studies show that patients with CKD can take allopurinol safely, but that they require slower up-titration to mitigate the risk of flares and AHS. Guidelines recommend a starting dose of ≤100 mg of allopurinol in patients with normal renal function, and even lower doses in patients with CKD.6 In studies showing safe dose titration in CKD, patients received an initial dose of allopurinol 50 mg daily, which increased by 50 mg every month.13,14 When hospitalists abruptly stop ULT during hospitalization in patients with CKD, those patients have to restart from the initial low dose and up-titrate slowly back to the lowest dose that achieves serum uric acid <6 mg/dL.6

Acute kidney injury (AKI) is not an absolute contraindication to allopurinol use, and the scant amount of published literature does not support discontinuation. In this acute situation, a patient may require a dose reduction in allopurinol to avoid toxicity depending on the severity of AKI. A discussion with inpatient pharmacy can help find a safe dose based on current creatinine clearance.

Physicians anecdotally recognize ULT discontinuation as a cause of inpatient gout flares. Clinicians and patients should view ULT as essential, even in patients who remain symptom-free for years. Between acute flares, a patient enters a potentially asymptomatic phase called “intercritical gout” that varies in duration. Urate deposition causing tophi and damage still occur during this phase, so patients must continue on ULT even if they have no recent flare history.

ULT that appears on any outpatient medication list needs verification of dose and compliance before ordering. If a patient is actually taking a lower dose than listed or not taking ULT at all, starting at a higher dose puts them at risk for flare and AHS, especially in patients with renal disease. Continuing ULT during hospitalization after verifying dose and compliance can potentially prevent gout flares and their downstream effects, including increased costs and potential side effects from additional pain medications.

Patients on chronic ULT should continue it during an acute gout flare.6,7 Literature and guidelines do not suggest that continuing ULT significantly worsens the intensity or duration of a flare. The initiation or up-titration of ULT, not the continuation of it, causes uric acid to dissolve, triggering an inflammatory response that increases the risk of gout flare. Therefore, guidelines recommend giving flare prophylaxis simultaneously for at least 3 to 6 months to prevent flares while starting and titrating ULT. Flare prophylaxis may continue longer depending on when a patient reaches a stable dose of ULT.6,7 While patients are receiving acute flare treatment, continuing ULT will help lower their serum uric acid levels over time.

To emphasize the importance of treating gout with ULT even further, the most recent American College of Rheumatology gout management guidelines conditionally recommend starting ULT during an acute flare for increased adherence. Small studies have shown that initiation of ULT does not precipitate attacks or significantly increase duration of flare. Input from patients influenced this recommendation, as they felt highly motivated to start ULT during acute flare due to symptoms.6

Additionally, due to comorbidities, inpatients often cannot tolerate standard flare therapies, such as nonsteroidal anti-inflammatory drugs, corticosteroids, or oral colchicine, to treat their acute symptoms. Moreover, patients often have other analgesics, such as opiates, prescribed for pain control. During an acute flare, hospitalists will likely need to add medications to treat the acute symptoms, but ULT should be considered an essential medication and continued as well.

When Stopping Urate-Lowering Therapy Might Be Helpful

Allopurinol can cause mild-to-severe cutaneous adverse reactions. AHS, a rare reaction that causes significant morbidity and mortality, presents with a rash, eosinophilia, fever, hepatitis, and progressive kidney failure. Risk factors for developing AHS include kidney impairment, higher starting doses, concurrent diuretic use, and presence of the genetic marker HLA B*5801.12 AHS usually occurs in the first 8 weeks of initiation of allopurinol, but can occur later in treatment, especially in those with risk factors—notably kidney impairment.12 When a patient on allopurinol develops a rash, the clinician should consider stopping allopurinol if concerned about AHS or, in milder cases, decrease the dose until the rash resolves.

What You Should Do Instead

When you see ULT on a patient’s medication list, verify the dose with the patient and continue it (even during an acute gout flare) unless a new rash has developed, or you are concerned about a drug-drug interaction. If a patient has a significant AKI, consider discussing dose modifications with your inpatient pharmacist.

Recommendations

  • Consider ULT an essential medication and continue it during the hospitalization of a patient with a history of gout.
  • Continue ULT while treating an acute gout flare.
  • Continue ULT in patients with AKI and CKD, but discuss dose modifications with a pharmacist for AKI patients.

Conclusion

In the clinical scenario, the hospitalist did not treat ULT as an essential medication on admission, and the patient’s gout flared, leading to increased morbidity, resource utilization, and cost of hospitalization. Stopping ULT has downstream effects after discharge, including delays in achieving prior gout control. If ULT is discontinued, outpatient clinicians must restart it at lower doses and then up-titrate slowly, increasing the risk of flares and possibly contributing to nonadherence. During hospitalization, clinicians should continue ULT.

Do you think this is a low-value practice? Is this truly a “Thing We Do for No Reason™”? Share what you do in your practice and join in the conversation online by retweeting it on Twitter (#TWDFNR) and liking it on Facebook. We invite you to propose ideas for other “Things We Do for No Reason™” topics by emailing TWDFNR@hospitalmedicine.org

References

1. Elfishawi MM, Zleik N, Kvrgic Z, et al. The rising incidence of gout and the increasing burden of comorbidities: a population-based study over 20 years. J Rheumatol. 2018;45(4):574-579. https://doi.org/10.3899/jrheum.170806
2. Fisher MC, Pillinger MH, Keenan RT. Inpatient gout: a review. Curr Rheumatol Rep. 2014;16(11):458. https://doi.org/10.1007/s11926-014-0458-z
3. Zleik N, Elfishawi MM, Kvrgic Z, et al. Hospitalization increases the risk of acute arthritic flares in gout: a population-based study over 2 decades. J Rheumatol. 2018;45(8):1188-1191. https://doi.org/10.3899/jrheum.171320
4. Dubreuil M, Neogi T, Chen CA, et al. Increased risk of recurrent gout attacks with hospitalization. Am J Med. 2013;126(12):1138-1141.e1. https://doi.org/10.1016/j.amjmed.2013.06.026
5. Flores NM, Neuvo J, Klein AB, Baumgartner S, Morlock R. The economic burden of uncontrolled gout: how controlling gout reduces cost. J Med Econ. 2019;22(1):1-6. https://doi.org/10.1080/13696998.2018.1532904
6. FitzGerald JD, Dalbeth N, Mikuls T, et al. 2020 American College of Rheumatology guideline for the management of gout. Arthritis Care Res (Hoboken). 2020;72(6):744-760. https://doi.org/10.1002/acr.24180
7. Khanna D, Khanna PP, FitzGerald JD, et al. 2012 American College of Rheumatology guidelines for management of gout. Part 2: therapy and antiinflammatory prophylaxis of acute gouty arthritis. Arthritis Care Res (Hoboken). 2012;64(10):1447-1461. https://doi.org/10.1002/acr.21773
8. Abhishek A, Doherty M. Education and non-pharmacological approaches for gout. Rheumatology (Oxford). 2018;57(suppl 1):i51-i58. https://doi.org/10.1093/rheumatology/kex421
9. Fields TR. The challenges of approaching and managing gout. Rheum Dis Clin North Am. 2019;45(1):145-157. https://doi.org/10.1016/j.rdc.2018.09.009
10. Huang IJ, Bays AM, Liew JW. Frequency of allopurinol dose reduction in hospitalized patients with gout flares. J Rheumatol. 2021;48(3):467-468. https://doi.org/10.3899/jrheum.201142
11. Hande KR, Noone RM, Stone WJ. Severe allopurinol toxicity. Description and guidelines for prevention in patients with renal insufficiency. Am J Med. 1984;76:47-56. https://doi.org/10.1016/0002-9343(84)90743-5
12. Stamp LK, Day RO, Yun J. Allopurinol hypersensitivity: investigating the cause and minimizing the risk. Nat Rev Rheumatol. 2016;12(4):235-242. https://doi.org/10.1038/nrrheum.2015.132
13. Stamp LK, Chapman PT, Barclay M, et al. The effect of kidney function on the urate lowering effect and safety of increasing allopurinol above doses based on creatinine clearance: a post hoc analysis of a randomized controlled trial. Arthritis Res Ther. 2017;19(1):283. https://doi.org/10.1186/s13075-017-1491-x
14. Stamp LK, O’Donnell JL, Zhang M, et al. Using allopurinol above the dose based on creatinine clearance is effective and safe in patients with chronic gout, including those with renal impairment. Arthritis Rheum. 2011;63(2):412-421. https://doi.org/10.1002/art.30119

References

1. Elfishawi MM, Zleik N, Kvrgic Z, et al. The rising incidence of gout and the increasing burden of comorbidities: a population-based study over 20 years. J Rheumatol. 2018;45(4):574-579. https://doi.org/10.3899/jrheum.170806
2. Fisher MC, Pillinger MH, Keenan RT. Inpatient gout: a review. Curr Rheumatol Rep. 2014;16(11):458. https://doi.org/10.1007/s11926-014-0458-z
3. Zleik N, Elfishawi MM, Kvrgic Z, et al. Hospitalization increases the risk of acute arthritic flares in gout: a population-based study over 2 decades. J Rheumatol. 2018;45(8):1188-1191. https://doi.org/10.3899/jrheum.171320
4. Dubreuil M, Neogi T, Chen CA, et al. Increased risk of recurrent gout attacks with hospitalization. Am J Med. 2013;126(12):1138-1141.e1. https://doi.org/10.1016/j.amjmed.2013.06.026
5. Flores NM, Neuvo J, Klein AB, Baumgartner S, Morlock R. The economic burden of uncontrolled gout: how controlling gout reduces cost. J Med Econ. 2019;22(1):1-6. https://doi.org/10.1080/13696998.2018.1532904
6. FitzGerald JD, Dalbeth N, Mikuls T, et al. 2020 American College of Rheumatology guideline for the management of gout. Arthritis Care Res (Hoboken). 2020;72(6):744-760. https://doi.org/10.1002/acr.24180
7. Khanna D, Khanna PP, FitzGerald JD, et al. 2012 American College of Rheumatology guidelines for management of gout. Part 2: therapy and antiinflammatory prophylaxis of acute gouty arthritis. Arthritis Care Res (Hoboken). 2012;64(10):1447-1461. https://doi.org/10.1002/acr.21773
8. Abhishek A, Doherty M. Education and non-pharmacological approaches for gout. Rheumatology (Oxford). 2018;57(suppl 1):i51-i58. https://doi.org/10.1093/rheumatology/kex421
9. Fields TR. The challenges of approaching and managing gout. Rheum Dis Clin North Am. 2019;45(1):145-157. https://doi.org/10.1016/j.rdc.2018.09.009
10. Huang IJ, Bays AM, Liew JW. Frequency of allopurinol dose reduction in hospitalized patients with gout flares. J Rheumatol. 2021;48(3):467-468. https://doi.org/10.3899/jrheum.201142
11. Hande KR, Noone RM, Stone WJ. Severe allopurinol toxicity. Description and guidelines for prevention in patients with renal insufficiency. Am J Med. 1984;76:47-56. https://doi.org/10.1016/0002-9343(84)90743-5
12. Stamp LK, Day RO, Yun J. Allopurinol hypersensitivity: investigating the cause and minimizing the risk. Nat Rev Rheumatol. 2016;12(4):235-242. https://doi.org/10.1038/nrrheum.2015.132
13. Stamp LK, Chapman PT, Barclay M, et al. The effect of kidney function on the urate lowering effect and safety of increasing allopurinol above doses based on creatinine clearance: a post hoc analysis of a randomized controlled trial. Arthritis Res Ther. 2017;19(1):283. https://doi.org/10.1186/s13075-017-1491-x
14. Stamp LK, O’Donnell JL, Zhang M, et al. Using allopurinol above the dose based on creatinine clearance is effective and safe in patients with chronic gout, including those with renal impairment. Arthritis Rheum. 2011;63(2):412-421. https://doi.org/10.1002/art.30119

Publications
Publications
Topics
Article Type
Display Headline
Things We Do for No Reason™: Discontinuing Urate-Lowering Therapy on Admission
Display Headline
Things We Do for No Reason™: Discontinuing Urate-Lowering Therapy on Admission
Sections
Article Source

© 2021 Society of Hospital Medicine

Citation Override
J Hosp Med. Published Online First December 15, 2021. DOI: 12788/jhm.3705
Disallow All Ads
Correspondence Location
Veena Patel, MD; Email: Veena.patel@austin.utexas.edu; Telephone: 512-495-5139; Twitter: @VeenaP_MD.
Content Gating
Gated (full article locked unless allowed per User)
Alternative CME
Disqus Comments
Default
Use ProPublica
Hide sidebar & use full width
render the right sidebar.
Conference Recap Checkbox
Not Conference Recap
Clinical Edge
Display the Slideshow in this Article
Gating Strategy
First Page Free
Medscape Article
Display survey writer
Reuters content
Disable Inline Native ads
WebMD Article
Article PDF Media
Image
Teambase ID
18001F75.SIG
Disable zoom
Off

Clinical Guideline Highlights for the Hospitalist: 2020 American Society of Addiction Medicine Clinical Practice Guideline on Alcohol Withdrawal Management

Article Type
Changed
Wed, 12/15/2021 - 01:15
Display Headline
Clinical Guideline Highlights for the Hospitalist: 2020 American Society of Addiction Medicine Clinical Practice Guideline on Alcohol Withdrawal Management

Alcohol is the most common substance implicated in hospitalizations for substance use disorders,1 and as a result, hospitalists commonly diagnose and manage alcohol withdrawal syndrome (AWS) in the inpatient medical setting. The 2020 guidelines of the American Society of Addiction Medicine (ASAM) provide updated recommendations for the diagnosis, monitoring, and treatment of patients hospitalized with AWS, which we have condensed to emphasize key changes from the last update2 and clarify ongoing areas of uncertainty. 

KEY RECOMMENDATIONS FOR THE HOSPITALIST

Diagnosis

Recommendation 1. All inpatients who have used alcohol recently or regularly should be risk-stratified for AWS, regardless of whether or not they have suggestive symptoms (recommendations I.3, I.4, I.5, II.10). The Alcohol Use Disorders Identification Test-(Piccinelli) Consumption (AUDIT-PC) identifies patients at risk for AWS, and the Prediction of Alcohol Withdrawal Severity Scale (PAWSS) identifies those at risk for severe or complicated AWS, which includes seizures and alcohol withdrawal delirium (formerly delirium tremens). The guideline emphasizes use of these tools rather than simply initiating Clinical Institute Withdrawal Assessment for Alcohol, revised (CIWA-Ar) monitoring on all such patients to diagnose AWS, as CIWA-Ar was developed for monitoring response to treatment, not diagnosis (recommendation I.6). 

Treating Mild/Moderate or Uncomplicated AWS

Recommendation 2. Because of their proven track record of reducing the incidence of seizure and alcohol withdrawal delirium, benzodiazepines remain the recommended first-line therapy (recommendations V.13, V.16). Symptom-triggered administration of benzodiazepines (via CIWA-Ar) is recommended over fixed-dose administration because the former is associated with shorter length of stay and lower cumulative benzodiazepine administration3,4 (recommendation V.23). Patients with mild AWS who are at low risk for severe or complicated withdrawal should be monitored for up to 36 hours for the development of worsening symptoms (recommendation V.1). For patients with high CIWA-Ar scores or who are at increased risk for severe or complicated AWS, frequent administration of moderate to high doses of a long-acting benzodiazepine early in AWS treatment (a practice called frontloading) is recommended to quickly control symptoms and prevent clinical worsening. This approach has been shown to reduce the incidence of seizures and alcohol withdrawal delirium (recommendations V.14, V.19, V.24).

Carbamazepine or gabapentin may be used in mild or moderate AWS if benzodiazepines are contraindicated; however, neither agent is recommended as first-line therapy because a clear reduction in seizure and withdrawal delirium has not been established (recommendation V.16). Alpha-2 agonists (eg, clonidine, dexmedetomidine) may be used to treat persistent autonomic hyperactivity or anxiety when these are not adequately controlled by benzodiazepines alone (recommendation V.36).

Treating Severe or Complicated AWS

Recommendation 3. The guideline defines severe AWS as withdrawal with severe signs and symptoms, and complicated AWS as withdrawal accompanied by seizures or delirium (Appendix Table5). The development of complications warrants prompt treatment. Patients who experience seizure should receive a fast-acting benzodiazepine (eg, intravenous [IV] diazepam or lorazepam) (recommendation VI.4). Patients with withdrawal delirium should receive a benzodiazepine (preferably parenterally) dosed to achieve light sedation. Clinicians should be prepared for the possibility that large doses may be required and to monitor patients for oversedation and respiratory depression (recommendations VI.13, VI.17). Antipsychotics may be used as adjuncts when withdrawal delirium or other symptoms, such as hallucinosis, are not adequately controlled by benzodiazepines alone, but should not be used as monotherapy (recommendation VI.20). The guideline emphasizes that alpha-2 agonists should not be used to treat withdrawal delirium (recommendation VI.21), but they may be used as adjuncts for resistant alcohol withdrawal in the intensive care unit (ICU) (recommendations VI.27, VI.29). Phenobarbital is an acceptable alternative to benzodiazepines for severe withdrawal (recommendation V.17); however, the guideline recommends that clinicians should be experienced in its use.

Treating Wernicke Encephalopathy

Recommendation 4. Thiamine should be administered to prevent Wernicke encephalopathy (WE), with parenteral formulations recommended in patients with malnutrition, severe/complicated withdrawal, or requiring ICU-level care (recommendations V.7, V.8). In particular, all patients admitted to an ICU for AWS should receive thiamine, as diagnosis of WE is often difficult in this population. Although there is no consensus on the required dose of thiamine to treat WE, 100 mg IV or intramuscularly (IM) daily for 3 to 5 days is commonly administered (recommendation V.7). Because of a lack of evidence of harm, thiamine may be given before, after, or concurrently with glucose or dextrose (recommendation V.7). The guideline does not make a specific recommendation regarding how to risk-stratify patients for WE.

Treating Underlying Alcohol Use Disorder 

Recommendation 5. Hospitalization for AWS is an important opportunity to engage patients in treatment for alcohol use disorder (AUD), including pharmacotherapy and connection with outpatient providers (recommendation V.12). The guideline emphasizes that treatment for AUD should be initiated concomitantly with AWS management whenever possible but does not make recommendations regarding specific pharmacotherapies.

CRITIQUE

This guideline was authored by a committee of emergency medicine physicians, psychiatrists, and internists using the Department of Veterans Affairs/Department of Defense guidelines and the RAND/UCLA appropriateness method to combine the scientific literature with expert opinion. The result is a series of recommendations for physicians, physician assistants, nurse practitioners, and pharmacists that are not rated by strength; an assessment of the quality of the supporting evidence is available in an appendix. Four of the nine guideline committee members reported significant financial relationships with industry and other entities relevant to these guidelines.

Despite concern about oversedation from phenobarbital raised in small case series,6 observational studies comparing phenobarbital with benzodiazepines suggest phenobarbital has similar efficacy for treating AWS and that oversedation is rare.7-9 Large randomized controlled trials in this area are lacking; however, at least one small randomized controlled trial10 among patients with AWS presenting to emergency departments supports the safety and efficacy of phenobarbital when used in combination with benzodiazepines. Given the growing body of evidence supporting the safety of phenobarbital, we believe a stronger recommendation for use in patients presenting with alcohol withdrawal delirium or treatment-resistant alcohol withdrawal is warranted. The guidelines also suggest that only “experienced clinicians” use phenobarbital for AWS, which may suppress appropriate use. Nationally, phenobarbital use for AWS remains low.11Finally, although the guideline recommends initiation of treatment for AUD, specific recommendations for pharmacotherapy are not provided. Three medications currently have approval from the US Food and Drug Administration for treatment of AUD: acamprosate, naltrexone, and disulfiram. Large randomized controlled trials support the safety and efficacy of acamprosate and naltrexone, with or without counselling, in the treatment of AUD,12 and disulfiram may be appropriate for selected highly motivated patients. We believe more specific recommendations to assist in choosing among these options would be useful.

AREAS IN NEED OF FUTURE STUDY

More data are needed on the safety and efficacy of phenobarbital in patients with AWS, as well as comparative effectiveness against benzodiazepines. Recruitment is ongoing for a single clinical trial comparing the effect of phenobarbital and lorazepam on length of stay among patients in the ICU with AWS (NCT04156464); to date, no randomized trials of phenobarbital have been conducted in medical inpatients with AWS. In addition, gaps in the literature exist regarding benzodiazepine selection, and head-to-head comparisons of symptom-triggered usage of different benzodiazepines are lacking.

Files
References

1. Heslin KC, Elixhauser A, Steiner CA. Hospitalizations involving mental and substance use disorders among adults, 2012. HCUP Statistical Brief #191. June 2015. Accessed November 17, 2021. www.hcup-us.ahrq.gov/reports/statbriefs/sb191-Hospitalization-Mental-Substance-Use-Disorders-2012.pdf
2. Mayo-Smith MF, Beecher LH, Fischer TL, et al. Management of alcohol withdrawal delirium. An evidence-based practice guideline. Arch Intern Med. 2004;164(13):1405-1412. https://doi.org/10.1001/archinte.164.13.1405
3. Saitz R, Mayo-Smith MF, Roberts MS, Redmond HA, Bernard DR, Calkins DR. Individualized treatment for alcohol withdrawal. A randomized double-blind controlled trial. JAMA. 1994;272(7):519-523.
4. Daeppen J-B, Gache P, Landry U, et al. Symptom-triggered vs fixed-schedule doses of benzodiazepine for alcohol withdrawal: a randomized treatment trial. Arch Intern Med. 2002;162(10):1117-1121. https://doi.org/10.1001/archinte.162.10.1117
5. The ASAM Clinical Practice Guideline on Alcohol Withdrawal Management. J Addict Med. 2020;14(3S suppl):1-72. https://doi.org/10.1097/ADM.0000000000000668
6. Oks M, Cleven KL, Healy L, et al. The safety and utility of phenobarbital use for the treatment of severe alcohol withdrawal syndrome in the medical intensive care unit. J Intensive Care Med. 2020;35(9):844-850. https://doi.org/10.1177/0885066618783947
7. Sullivan JT, Sykora K, Schneiderman J, Naranjo CA, Sellers EM. Assessment of alcohol withdrawal: the revised clinical institute withdrawal assessment for alcohol scale (CIWA-Ar). Br J Addict. 1989;84(11):1353-1357. https://doi.org/10.1111/j.1360-0443.1989.tb00737.x
8. Ibarra Jr F. Single dose phenobarbital in addition to symptom-triggered lorazepam in alcohol withdrawal. Am J Emerg Med. 2020;38(2):178-181. https://doi.org/10.1016/j.ajem.2019.01.053
9. Nisavic M, Nejad SH, Isenberg BM, et al. Use of phenobarbital in alcohol withdrawal management–a retrospective comparison study of phenobarbital and benzodiazepines for acute alcohol withdrawal management in general medical patients. Psychosomatics. 2019;60(5):458-467. https://doi.org/10.1016/j.psym.2019.02.002
10. Rosenson J, Clements C, Simon B, et al. Phenobarbital for acute alcohol withdrawal: a prospective randomized double-blind placebo-controlled study. J Emerg Med. 2013;44(3):592-598.e2. https://doi.org/10.1016/j.jemermed.2012.07.056
11. Gupta N, Emerman CL. Trends in the management of inpatients with alcohol withdrawal syndrome. Addict Disord Their Treat. 2021;20(1):29-32. https://doi.org/10.1097/ADT.0000000000000203
12. Anton RF, O’Malley SS, Ciraulo DA, et al. Combined pharmacotherapies and behavioral interventions for alcohol dependence: the COMBINE study: a randomized controlled trial. JAMA. 2006;295(17):2003-2017. https://doi.org/10.1001/jama.295.17.2003

Article PDF
Author and Disclosure Information

1Medical Service, VA Boston Healthcare System, West Roxbury, Massachusetts; 2Harvard Medical School, Boston, Massachusetts.

Disclosures
The authors reported no conflicts of interest.

Publications
Topics
Sections
Files
Files
Author and Disclosure Information

1Medical Service, VA Boston Healthcare System, West Roxbury, Massachusetts; 2Harvard Medical School, Boston, Massachusetts.

Disclosures
The authors reported no conflicts of interest.

Author and Disclosure Information

1Medical Service, VA Boston Healthcare System, West Roxbury, Massachusetts; 2Harvard Medical School, Boston, Massachusetts.

Disclosures
The authors reported no conflicts of interest.

Article PDF
Article PDF
Related Articles

Alcohol is the most common substance implicated in hospitalizations for substance use disorders,1 and as a result, hospitalists commonly diagnose and manage alcohol withdrawal syndrome (AWS) in the inpatient medical setting. The 2020 guidelines of the American Society of Addiction Medicine (ASAM) provide updated recommendations for the diagnosis, monitoring, and treatment of patients hospitalized with AWS, which we have condensed to emphasize key changes from the last update2 and clarify ongoing areas of uncertainty. 

KEY RECOMMENDATIONS FOR THE HOSPITALIST

Diagnosis

Recommendation 1. All inpatients who have used alcohol recently or regularly should be risk-stratified for AWS, regardless of whether or not they have suggestive symptoms (recommendations I.3, I.4, I.5, II.10). The Alcohol Use Disorders Identification Test-(Piccinelli) Consumption (AUDIT-PC) identifies patients at risk for AWS, and the Prediction of Alcohol Withdrawal Severity Scale (PAWSS) identifies those at risk for severe or complicated AWS, which includes seizures and alcohol withdrawal delirium (formerly delirium tremens). The guideline emphasizes use of these tools rather than simply initiating Clinical Institute Withdrawal Assessment for Alcohol, revised (CIWA-Ar) monitoring on all such patients to diagnose AWS, as CIWA-Ar was developed for monitoring response to treatment, not diagnosis (recommendation I.6). 

Treating Mild/Moderate or Uncomplicated AWS

Recommendation 2. Because of their proven track record of reducing the incidence of seizure and alcohol withdrawal delirium, benzodiazepines remain the recommended first-line therapy (recommendations V.13, V.16). Symptom-triggered administration of benzodiazepines (via CIWA-Ar) is recommended over fixed-dose administration because the former is associated with shorter length of stay and lower cumulative benzodiazepine administration3,4 (recommendation V.23). Patients with mild AWS who are at low risk for severe or complicated withdrawal should be monitored for up to 36 hours for the development of worsening symptoms (recommendation V.1). For patients with high CIWA-Ar scores or who are at increased risk for severe or complicated AWS, frequent administration of moderate to high doses of a long-acting benzodiazepine early in AWS treatment (a practice called frontloading) is recommended to quickly control symptoms and prevent clinical worsening. This approach has been shown to reduce the incidence of seizures and alcohol withdrawal delirium (recommendations V.14, V.19, V.24).

Carbamazepine or gabapentin may be used in mild or moderate AWS if benzodiazepines are contraindicated; however, neither agent is recommended as first-line therapy because a clear reduction in seizure and withdrawal delirium has not been established (recommendation V.16). Alpha-2 agonists (eg, clonidine, dexmedetomidine) may be used to treat persistent autonomic hyperactivity or anxiety when these are not adequately controlled by benzodiazepines alone (recommendation V.36).

Treating Severe or Complicated AWS

Recommendation 3. The guideline defines severe AWS as withdrawal with severe signs and symptoms, and complicated AWS as withdrawal accompanied by seizures or delirium (Appendix Table5). The development of complications warrants prompt treatment. Patients who experience seizure should receive a fast-acting benzodiazepine (eg, intravenous [IV] diazepam or lorazepam) (recommendation VI.4). Patients with withdrawal delirium should receive a benzodiazepine (preferably parenterally) dosed to achieve light sedation. Clinicians should be prepared for the possibility that large doses may be required and to monitor patients for oversedation and respiratory depression (recommendations VI.13, VI.17). Antipsychotics may be used as adjuncts when withdrawal delirium or other symptoms, such as hallucinosis, are not adequately controlled by benzodiazepines alone, but should not be used as monotherapy (recommendation VI.20). The guideline emphasizes that alpha-2 agonists should not be used to treat withdrawal delirium (recommendation VI.21), but they may be used as adjuncts for resistant alcohol withdrawal in the intensive care unit (ICU) (recommendations VI.27, VI.29). Phenobarbital is an acceptable alternative to benzodiazepines for severe withdrawal (recommendation V.17); however, the guideline recommends that clinicians should be experienced in its use.

Treating Wernicke Encephalopathy

Recommendation 4. Thiamine should be administered to prevent Wernicke encephalopathy (WE), with parenteral formulations recommended in patients with malnutrition, severe/complicated withdrawal, or requiring ICU-level care (recommendations V.7, V.8). In particular, all patients admitted to an ICU for AWS should receive thiamine, as diagnosis of WE is often difficult in this population. Although there is no consensus on the required dose of thiamine to treat WE, 100 mg IV or intramuscularly (IM) daily for 3 to 5 days is commonly administered (recommendation V.7). Because of a lack of evidence of harm, thiamine may be given before, after, or concurrently with glucose or dextrose (recommendation V.7). The guideline does not make a specific recommendation regarding how to risk-stratify patients for WE.

Treating Underlying Alcohol Use Disorder 

Recommendation 5. Hospitalization for AWS is an important opportunity to engage patients in treatment for alcohol use disorder (AUD), including pharmacotherapy and connection with outpatient providers (recommendation V.12). The guideline emphasizes that treatment for AUD should be initiated concomitantly with AWS management whenever possible but does not make recommendations regarding specific pharmacotherapies.

CRITIQUE

This guideline was authored by a committee of emergency medicine physicians, psychiatrists, and internists using the Department of Veterans Affairs/Department of Defense guidelines and the RAND/UCLA appropriateness method to combine the scientific literature with expert opinion. The result is a series of recommendations for physicians, physician assistants, nurse practitioners, and pharmacists that are not rated by strength; an assessment of the quality of the supporting evidence is available in an appendix. Four of the nine guideline committee members reported significant financial relationships with industry and other entities relevant to these guidelines.

Despite concern about oversedation from phenobarbital raised in small case series,6 observational studies comparing phenobarbital with benzodiazepines suggest phenobarbital has similar efficacy for treating AWS and that oversedation is rare.7-9 Large randomized controlled trials in this area are lacking; however, at least one small randomized controlled trial10 among patients with AWS presenting to emergency departments supports the safety and efficacy of phenobarbital when used in combination with benzodiazepines. Given the growing body of evidence supporting the safety of phenobarbital, we believe a stronger recommendation for use in patients presenting with alcohol withdrawal delirium or treatment-resistant alcohol withdrawal is warranted. The guidelines also suggest that only “experienced clinicians” use phenobarbital for AWS, which may suppress appropriate use. Nationally, phenobarbital use for AWS remains low.11Finally, although the guideline recommends initiation of treatment for AUD, specific recommendations for pharmacotherapy are not provided. Three medications currently have approval from the US Food and Drug Administration for treatment of AUD: acamprosate, naltrexone, and disulfiram. Large randomized controlled trials support the safety and efficacy of acamprosate and naltrexone, with or without counselling, in the treatment of AUD,12 and disulfiram may be appropriate for selected highly motivated patients. We believe more specific recommendations to assist in choosing among these options would be useful.

AREAS IN NEED OF FUTURE STUDY

More data are needed on the safety and efficacy of phenobarbital in patients with AWS, as well as comparative effectiveness against benzodiazepines. Recruitment is ongoing for a single clinical trial comparing the effect of phenobarbital and lorazepam on length of stay among patients in the ICU with AWS (NCT04156464); to date, no randomized trials of phenobarbital have been conducted in medical inpatients with AWS. In addition, gaps in the literature exist regarding benzodiazepine selection, and head-to-head comparisons of symptom-triggered usage of different benzodiazepines are lacking.

Alcohol is the most common substance implicated in hospitalizations for substance use disorders,1 and as a result, hospitalists commonly diagnose and manage alcohol withdrawal syndrome (AWS) in the inpatient medical setting. The 2020 guidelines of the American Society of Addiction Medicine (ASAM) provide updated recommendations for the diagnosis, monitoring, and treatment of patients hospitalized with AWS, which we have condensed to emphasize key changes from the last update2 and clarify ongoing areas of uncertainty. 

KEY RECOMMENDATIONS FOR THE HOSPITALIST

Diagnosis

Recommendation 1. All inpatients who have used alcohol recently or regularly should be risk-stratified for AWS, regardless of whether or not they have suggestive symptoms (recommendations I.3, I.4, I.5, II.10). The Alcohol Use Disorders Identification Test-(Piccinelli) Consumption (AUDIT-PC) identifies patients at risk for AWS, and the Prediction of Alcohol Withdrawal Severity Scale (PAWSS) identifies those at risk for severe or complicated AWS, which includes seizures and alcohol withdrawal delirium (formerly delirium tremens). The guideline emphasizes use of these tools rather than simply initiating Clinical Institute Withdrawal Assessment for Alcohol, revised (CIWA-Ar) monitoring on all such patients to diagnose AWS, as CIWA-Ar was developed for monitoring response to treatment, not diagnosis (recommendation I.6). 

Treating Mild/Moderate or Uncomplicated AWS

Recommendation 2. Because of their proven track record of reducing the incidence of seizure and alcohol withdrawal delirium, benzodiazepines remain the recommended first-line therapy (recommendations V.13, V.16). Symptom-triggered administration of benzodiazepines (via CIWA-Ar) is recommended over fixed-dose administration because the former is associated with shorter length of stay and lower cumulative benzodiazepine administration3,4 (recommendation V.23). Patients with mild AWS who are at low risk for severe or complicated withdrawal should be monitored for up to 36 hours for the development of worsening symptoms (recommendation V.1). For patients with high CIWA-Ar scores or who are at increased risk for severe or complicated AWS, frequent administration of moderate to high doses of a long-acting benzodiazepine early in AWS treatment (a practice called frontloading) is recommended to quickly control symptoms and prevent clinical worsening. This approach has been shown to reduce the incidence of seizures and alcohol withdrawal delirium (recommendations V.14, V.19, V.24).

Carbamazepine or gabapentin may be used in mild or moderate AWS if benzodiazepines are contraindicated; however, neither agent is recommended as first-line therapy because a clear reduction in seizure and withdrawal delirium has not been established (recommendation V.16). Alpha-2 agonists (eg, clonidine, dexmedetomidine) may be used to treat persistent autonomic hyperactivity or anxiety when these are not adequately controlled by benzodiazepines alone (recommendation V.36).

Treating Severe or Complicated AWS

Recommendation 3. The guideline defines severe AWS as withdrawal with severe signs and symptoms, and complicated AWS as withdrawal accompanied by seizures or delirium (Appendix Table5). The development of complications warrants prompt treatment. Patients who experience seizure should receive a fast-acting benzodiazepine (eg, intravenous [IV] diazepam or lorazepam) (recommendation VI.4). Patients with withdrawal delirium should receive a benzodiazepine (preferably parenterally) dosed to achieve light sedation. Clinicians should be prepared for the possibility that large doses may be required and to monitor patients for oversedation and respiratory depression (recommendations VI.13, VI.17). Antipsychotics may be used as adjuncts when withdrawal delirium or other symptoms, such as hallucinosis, are not adequately controlled by benzodiazepines alone, but should not be used as monotherapy (recommendation VI.20). The guideline emphasizes that alpha-2 agonists should not be used to treat withdrawal delirium (recommendation VI.21), but they may be used as adjuncts for resistant alcohol withdrawal in the intensive care unit (ICU) (recommendations VI.27, VI.29). Phenobarbital is an acceptable alternative to benzodiazepines for severe withdrawal (recommendation V.17); however, the guideline recommends that clinicians should be experienced in its use.

Treating Wernicke Encephalopathy

Recommendation 4. Thiamine should be administered to prevent Wernicke encephalopathy (WE), with parenteral formulations recommended in patients with malnutrition, severe/complicated withdrawal, or requiring ICU-level care (recommendations V.7, V.8). In particular, all patients admitted to an ICU for AWS should receive thiamine, as diagnosis of WE is often difficult in this population. Although there is no consensus on the required dose of thiamine to treat WE, 100 mg IV or intramuscularly (IM) daily for 3 to 5 days is commonly administered (recommendation V.7). Because of a lack of evidence of harm, thiamine may be given before, after, or concurrently with glucose or dextrose (recommendation V.7). The guideline does not make a specific recommendation regarding how to risk-stratify patients for WE.

Treating Underlying Alcohol Use Disorder 

Recommendation 5. Hospitalization for AWS is an important opportunity to engage patients in treatment for alcohol use disorder (AUD), including pharmacotherapy and connection with outpatient providers (recommendation V.12). The guideline emphasizes that treatment for AUD should be initiated concomitantly with AWS management whenever possible but does not make recommendations regarding specific pharmacotherapies.

CRITIQUE

This guideline was authored by a committee of emergency medicine physicians, psychiatrists, and internists using the Department of Veterans Affairs/Department of Defense guidelines and the RAND/UCLA appropriateness method to combine the scientific literature with expert opinion. The result is a series of recommendations for physicians, physician assistants, nurse practitioners, and pharmacists that are not rated by strength; an assessment of the quality of the supporting evidence is available in an appendix. Four of the nine guideline committee members reported significant financial relationships with industry and other entities relevant to these guidelines.

Despite concern about oversedation from phenobarbital raised in small case series,6 observational studies comparing phenobarbital with benzodiazepines suggest phenobarbital has similar efficacy for treating AWS and that oversedation is rare.7-9 Large randomized controlled trials in this area are lacking; however, at least one small randomized controlled trial10 among patients with AWS presenting to emergency departments supports the safety and efficacy of phenobarbital when used in combination with benzodiazepines. Given the growing body of evidence supporting the safety of phenobarbital, we believe a stronger recommendation for use in patients presenting with alcohol withdrawal delirium or treatment-resistant alcohol withdrawal is warranted. The guidelines also suggest that only “experienced clinicians” use phenobarbital for AWS, which may suppress appropriate use. Nationally, phenobarbital use for AWS remains low.11Finally, although the guideline recommends initiation of treatment for AUD, specific recommendations for pharmacotherapy are not provided. Three medications currently have approval from the US Food and Drug Administration for treatment of AUD: acamprosate, naltrexone, and disulfiram. Large randomized controlled trials support the safety and efficacy of acamprosate and naltrexone, with or without counselling, in the treatment of AUD,12 and disulfiram may be appropriate for selected highly motivated patients. We believe more specific recommendations to assist in choosing among these options would be useful.

AREAS IN NEED OF FUTURE STUDY

More data are needed on the safety and efficacy of phenobarbital in patients with AWS, as well as comparative effectiveness against benzodiazepines. Recruitment is ongoing for a single clinical trial comparing the effect of phenobarbital and lorazepam on length of stay among patients in the ICU with AWS (NCT04156464); to date, no randomized trials of phenobarbital have been conducted in medical inpatients with AWS. In addition, gaps in the literature exist regarding benzodiazepine selection, and head-to-head comparisons of symptom-triggered usage of different benzodiazepines are lacking.

References

1. Heslin KC, Elixhauser A, Steiner CA. Hospitalizations involving mental and substance use disorders among adults, 2012. HCUP Statistical Brief #191. June 2015. Accessed November 17, 2021. www.hcup-us.ahrq.gov/reports/statbriefs/sb191-Hospitalization-Mental-Substance-Use-Disorders-2012.pdf
2. Mayo-Smith MF, Beecher LH, Fischer TL, et al. Management of alcohol withdrawal delirium. An evidence-based practice guideline. Arch Intern Med. 2004;164(13):1405-1412. https://doi.org/10.1001/archinte.164.13.1405
3. Saitz R, Mayo-Smith MF, Roberts MS, Redmond HA, Bernard DR, Calkins DR. Individualized treatment for alcohol withdrawal. A randomized double-blind controlled trial. JAMA. 1994;272(7):519-523.
4. Daeppen J-B, Gache P, Landry U, et al. Symptom-triggered vs fixed-schedule doses of benzodiazepine for alcohol withdrawal: a randomized treatment trial. Arch Intern Med. 2002;162(10):1117-1121. https://doi.org/10.1001/archinte.162.10.1117
5. The ASAM Clinical Practice Guideline on Alcohol Withdrawal Management. J Addict Med. 2020;14(3S suppl):1-72. https://doi.org/10.1097/ADM.0000000000000668
6. Oks M, Cleven KL, Healy L, et al. The safety and utility of phenobarbital use for the treatment of severe alcohol withdrawal syndrome in the medical intensive care unit. J Intensive Care Med. 2020;35(9):844-850. https://doi.org/10.1177/0885066618783947
7. Sullivan JT, Sykora K, Schneiderman J, Naranjo CA, Sellers EM. Assessment of alcohol withdrawal: the revised clinical institute withdrawal assessment for alcohol scale (CIWA-Ar). Br J Addict. 1989;84(11):1353-1357. https://doi.org/10.1111/j.1360-0443.1989.tb00737.x
8. Ibarra Jr F. Single dose phenobarbital in addition to symptom-triggered lorazepam in alcohol withdrawal. Am J Emerg Med. 2020;38(2):178-181. https://doi.org/10.1016/j.ajem.2019.01.053
9. Nisavic M, Nejad SH, Isenberg BM, et al. Use of phenobarbital in alcohol withdrawal management–a retrospective comparison study of phenobarbital and benzodiazepines for acute alcohol withdrawal management in general medical patients. Psychosomatics. 2019;60(5):458-467. https://doi.org/10.1016/j.psym.2019.02.002
10. Rosenson J, Clements C, Simon B, et al. Phenobarbital for acute alcohol withdrawal: a prospective randomized double-blind placebo-controlled study. J Emerg Med. 2013;44(3):592-598.e2. https://doi.org/10.1016/j.jemermed.2012.07.056
11. Gupta N, Emerman CL. Trends in the management of inpatients with alcohol withdrawal syndrome. Addict Disord Their Treat. 2021;20(1):29-32. https://doi.org/10.1097/ADT.0000000000000203
12. Anton RF, O’Malley SS, Ciraulo DA, et al. Combined pharmacotherapies and behavioral interventions for alcohol dependence: the COMBINE study: a randomized controlled trial. JAMA. 2006;295(17):2003-2017. https://doi.org/10.1001/jama.295.17.2003

References

1. Heslin KC, Elixhauser A, Steiner CA. Hospitalizations involving mental and substance use disorders among adults, 2012. HCUP Statistical Brief #191. June 2015. Accessed November 17, 2021. www.hcup-us.ahrq.gov/reports/statbriefs/sb191-Hospitalization-Mental-Substance-Use-Disorders-2012.pdf
2. Mayo-Smith MF, Beecher LH, Fischer TL, et al. Management of alcohol withdrawal delirium. An evidence-based practice guideline. Arch Intern Med. 2004;164(13):1405-1412. https://doi.org/10.1001/archinte.164.13.1405
3. Saitz R, Mayo-Smith MF, Roberts MS, Redmond HA, Bernard DR, Calkins DR. Individualized treatment for alcohol withdrawal. A randomized double-blind controlled trial. JAMA. 1994;272(7):519-523.
4. Daeppen J-B, Gache P, Landry U, et al. Symptom-triggered vs fixed-schedule doses of benzodiazepine for alcohol withdrawal: a randomized treatment trial. Arch Intern Med. 2002;162(10):1117-1121. https://doi.org/10.1001/archinte.162.10.1117
5. The ASAM Clinical Practice Guideline on Alcohol Withdrawal Management. J Addict Med. 2020;14(3S suppl):1-72. https://doi.org/10.1097/ADM.0000000000000668
6. Oks M, Cleven KL, Healy L, et al. The safety and utility of phenobarbital use for the treatment of severe alcohol withdrawal syndrome in the medical intensive care unit. J Intensive Care Med. 2020;35(9):844-850. https://doi.org/10.1177/0885066618783947
7. Sullivan JT, Sykora K, Schneiderman J, Naranjo CA, Sellers EM. Assessment of alcohol withdrawal: the revised clinical institute withdrawal assessment for alcohol scale (CIWA-Ar). Br J Addict. 1989;84(11):1353-1357. https://doi.org/10.1111/j.1360-0443.1989.tb00737.x
8. Ibarra Jr F. Single dose phenobarbital in addition to symptom-triggered lorazepam in alcohol withdrawal. Am J Emerg Med. 2020;38(2):178-181. https://doi.org/10.1016/j.ajem.2019.01.053
9. Nisavic M, Nejad SH, Isenberg BM, et al. Use of phenobarbital in alcohol withdrawal management–a retrospective comparison study of phenobarbital and benzodiazepines for acute alcohol withdrawal management in general medical patients. Psychosomatics. 2019;60(5):458-467. https://doi.org/10.1016/j.psym.2019.02.002
10. Rosenson J, Clements C, Simon B, et al. Phenobarbital for acute alcohol withdrawal: a prospective randomized double-blind placebo-controlled study. J Emerg Med. 2013;44(3):592-598.e2. https://doi.org/10.1016/j.jemermed.2012.07.056
11. Gupta N, Emerman CL. Trends in the management of inpatients with alcohol withdrawal syndrome. Addict Disord Their Treat. 2021;20(1):29-32. https://doi.org/10.1097/ADT.0000000000000203
12. Anton RF, O’Malley SS, Ciraulo DA, et al. Combined pharmacotherapies and behavioral interventions for alcohol dependence: the COMBINE study: a randomized controlled trial. JAMA. 2006;295(17):2003-2017. https://doi.org/10.1001/jama.295.17.2003

Publications
Publications
Topics
Article Type
Display Headline
Clinical Guideline Highlights for the Hospitalist: 2020 American Society of Addiction Medicine Clinical Practice Guideline on Alcohol Withdrawal Management
Display Headline
Clinical Guideline Highlights for the Hospitalist: 2020 American Society of Addiction Medicine Clinical Practice Guideline on Alcohol Withdrawal Management
Sections
Article Source

© 2021 Society of Hospital Medicine

Citation Override
J Hosp Med. Published Online First December 15, 2021. DOI: 10.12788/jhm.3729
Disallow All Ads
Correspondence Location
Rahul B Ganatra, MD MPH; Email: rbganatra@gmail.com; Twitter: @rbganatra.
Content Gating
Gated (full article locked unless allowed per User)
Alternative CME
Disqus Comments
Default
Use ProPublica
Hide sidebar & use full width
render the right sidebar.
Conference Recap Checkbox
Not Conference Recap
Clinical Edge
Display the Slideshow in this Article
Gating Strategy
First Page Free
Medscape Article
Display survey writer
Reuters content
Disable Inline Native ads
WebMD Article
Article PDF Media
Image
Teambase ID
18001F74.SIG
Disable zoom
Off
Media Files

Things We Do For No Reason™: Ultrasonography After an Initial Negative CT in Patients Presenting With Acute Abdominal or Pelvic Pain

Article Type
Changed
Wed, 11/17/2021 - 01:15
Display Headline
Things We Do For No Reason™: Ultrasonography After an Initial Negative CT in Patients Presenting With Acute Abdominal or Pelvic Pain

Inspired by the ABIM Foundation’s Choosing Wisely® campaign, the “Things We Do for No Reason” (TWDFNR) series reviews practices that have become common parts of hospital care but may provide little value to our patients. Practices reviewed in the TWDFNR series do not represent clear-cut conclusions or clinical practice standards but are meant as a starting place for research and active discussions among hospitalists and patients. We invite you to be part of that discussion.

Clinical Scenario  

A 70-year-old woman presented to the emergency department (ED) with diffuse abdominal pain, nausea, and vomiting with normal liver function tests and lipase. Computed tomography (CT) of the abdomen and pelvis with intravenous contrast revealed no acute intraabdominal pathology except for an incidentally noted, mildly enlarged but nondistended gallbladder without evident cholelithiasis, pericholecystic fluid, or gallbladder wall edema. The hospitalist orders an abdominal ultrasound to evaluate for acute biliary pathology potentially missed by CT. 

Why You Might Consider Ordering an Abdominal Ultrasound After a Negative CT

Guidelines and expert opinion recommend an “ultrasound-first” approach when patients present with right upper quadrant (RUQ) abdominal pain or pelvic pain of suspected gynecologic origin.1-3 When evaluating suspected biliary disease, experts recommend beginning with ultrasonography based on the speed of obtaining results, absence of radiation exposure, reduced cost, and good diagnostic accuracy.1 Ultrasound has superior sensitivity, of 98%,4 in identifying radiolucent gallstones, compared to CT’s 79% sensitivity.5 Ultrasonography also differentiates gallbladder sludge from cholelithiasis, evaluates the extrahepatic and intrahepatic bile ducts, and can identify alternate causes of RUQ pain.1,3 Since ultrasound has important advantages, a negative initial CT may lead the clinician to consider an ultrasound to evaluate for gallbladder diseases.

Additionally, ultrasound provides improved anatomic detail of pelvic structures when diagnosing endometrial or ovarian pathology2 and improves diagnostic accuracy when the initial CT reveals an abnormal pelvic finding (eg, defining an enlarged ovary on CT as ovarian torsion, a cyst, or an adnexal mass).6 While CT excludes emergent surgical diagnoses, ultrasound may add value in elucidating a cause of the pain, even when urgent surgical management is not necessary.7

Many providers believe that a CT lacks sensitivity for acute biliary or pelvic pathology and will order an ultrasound to avoid missing an important diagnosis.7 Within 6 months at a single center, clinicians ordered 614 abdominal ultrasounds within 72 hours of an abdominal CT; 227 of these orders were to evaluate the gallbladder. Clinicians documented a discussion with a radiologist in only 19% of cases.8

Why Ordering an Ultrasound After a Negative CT Is Unnecessary

While ultrasound is more sensitive for detecting gallstones, the data do not indicate that it is more sensitive than CT for detecting acute cholecystitis. Abdominal ultrasound has a sensitivity for the diagnosis of acute cholecystitis of 81%, with a specificity of 83%,9 while CT has a comparable 85% to 94%9,10 sensitivity and specificity ranging from 59% to 99%.9,11 A recent study using more stringent radiographic criteria (two or more abnormal features) for diagnosing acute cholecystitis found ultrasound and CT had near equivalent sensitivities of 61% and 55%, respectively.12 Even with these stringent criteria, CT had a negative predictive value of 90% and approached 95% when applying a less strict (one feature) criterion.12 As a result, an abdominal ultrasound will rarely diagnose cholecystitis after a normal CT.

A 2020 study evaluated the diagnostic yield and clinical impact of ordering an abdominal or pelvic ultrasound within 24 hours of a negative abdominal CT.7It found that only 3/132 (2.3%) of abdominal ultrasounds ordered after a negative CT revealed acute pathology potentially requiring surgery. Only one of these three patients (1/132) required surgical intervention for confirmed acute cholecystitis.7 The follow-up abdominal ultrasound identified asymptomatic gallstones in 9/132 (6.8%) and gallbladder polyps in 4/132 (3.0%) of cases.7 Selective use of ultrasound after CT for patients with clinically worsening or progressive RUQ pain will avoid missing a “can’t miss” diagnosis and reduce low-yield testing for a majority of patients.

As with abdominal CT and ultrasound, the recommendation for an initial pelvic ultrasound when evaluating female pelvic pain also stems from the reduced cost, absence of radiation exposure, and superior anatomic visualization of the pelvic organs when compared with pelvic CT.2,13 However, as with the results of studies investigating the use of abdominal ultrasound after negative CT, a study of pelvic ultrasound after a negative CT revealed that only 4/126 (3.2%) follow-up ultrasounds had an abnormal finding not identified on CT.13 Pelvic ultrasound found four endometrial abnormalities that did not alter acute management.13 Notably, in 58% of the cases, the indication for ordering the subsequent ultrasound was “rule out ovarian torsion.” However, CT almost always finds a morphologically abnormal ovary in the case of torsion.6 One study and literature review found that all 28 patients studied and all 85 patients from previous studies with proven ovarian torsion had either an adnexal mass or an enlarged ovary on pelvic CT.6 Harfouch et al found that 0 out of 199 pelvic ultrasounds ordered after a negative CT revealed acute surgical pathology, but pelvic ultrasound did identify nonsurgical uterine and ovarian abnormalities.7 In conclusion, when clinicians order CT as the first study to diagnose acute, surgical biliary or gynecologic causes of pain, follow-up ultrasound has a low probability of affecting diagnosis or management if the CT is normal.

When You Should Consider Ultrasound After CT

The previous discussion only applies if hospitalists order an ultrasound within 24 to 48 hours of the initial CT. Time and clinical course are critical diagnostic tools during an admission for abdominal pain. Consider pelvic or abdominal ultrasound based on guideline recommendations if a patient develops new or evolving RUQ or pelvic pain.1,2 The rationale for obtaining the initial negative CT may no longer apply, and the clinician must consider the changing characteristics of the patient’s symptoms. For example, initial CT imaging may miss cholelithiasis in a patient presenting for biliary colic. Under observation, the patient may develop acute cholecystitis, potentially requiring an abdominal ultrasound. Also, the data for pelvic ultrasound apply to a normal CT of the abdomen and pelvis. Ultrasound may help to further evaluate indeterminate findings present on initial CT or if recommended by radiology.

What You Should Do Instead

When the hospitalist assumes care for a patient with abdominal pain and a negative CT, appropriate next steps include taking time to reexamine the differential diagnosis, repeating the history and physical, and communicating directly with a radiologist. These steps ensure the highest diagnostic yield and the lowest cost and help prevent diagnostic error arising from anchoring on the initial negative ED evaluation. Prior research demonstrates that the initial history alone can lead to the correct diagnosis in up to 76% of cases of abdominal pain.14 If repeat evaluation determines that additional imaging is necessary, the American College of Radiology provides evidence-based guidelines to help clinicians determine the correct imaging test based on the clinical situation (Appendix Table).1,2 For example, an equivocal ultrasound or CT exam with continued suspicion for acute cholecystitis or an alternate diagnosis, such as acalculous cholecystitis or choledocholithiasis, merits alternative tests with improved sensitivity and specificity profiles (Tc 99 m hepatobiliary iminodiacetic acid scan, also known as cholescintigraphy, for cholecystitis and acalculous cholecystitis, or magnetic resonance cholangiopancreatography for choledocholithiasis).1

Remember to communicate with the radiologist to rule out “can’t miss” diagnoses, increase mutual understanding of the radiographic test characteristics for specific disease processes, and improve the radiologist’s understanding of the patient’s history and clinical question.15 Collaboration with the radiologist can also determine the need for follow-up imaging and its timing. One single-center study found that surgeons’ diagnostic impression and management changed in 35/100 (35%) cases after an in-person review with the radiologist.15 Observing patients in the hospital with a nondiagnostic initial evaluation but concerning clinical features often allows for either a trial of cure or for the disease process to “declare itself.”14 This allows clinicians to target additional testing to a specific diagnosis and avoid reflexive ordering of additional radiographic studies.

Recommendations

  • Order an ultrasound for initial imaging of RUQ and female pelvic pain.
  • Do not reflexively order an ultrasound within 24 to 48 hours of a negative CT scan to pursue biliary or pelvic pathology.
  • Only order repeat abdominal imaging if clinical circumstances evolve or discussions with a radiologist conclude it will answer a more specific diagnostic question.

Conclusion

In our clinical scenario involving a patient with diffuse abdominal pain and a negative CT, the hospitalist should reevaluate the history, exam, and differential diagnosis before pursuing further diagnostic imaging. Based on the evidence presented, CT has similar diagnostic accuracy to ultrasound for biliary and gynecologic pathologies necessitating urgent surgical management (eg, acute cholecystitis, ovarian torsion), and a follow-up ultrasound adds little. If the utility of imaging remains in question, hospitalist consultation with a radiologist can clarify whether prior imaging answered the clinical question and the diagnostic utility of repeat abdominal imaging. With thoughtful reevaluation of the history and physical, and communication with radiology, hospitalists can reduce unnecessary, low-yield imaging and reduce healthcare costs when evaluating patients with abdominal pain.

Do you think this is a low-value practice? Is this truly a “Thing We Do for No Reason”? Share what you do in your practice and join in the conversation online by retweeting it on Twitter (#TWDFNR) and liking it on Facebook. We invite you to propose ideas for other “Things We Do for No Reason” topics by emailing TWDFNR@hospitalmedicine.org

Files
References

1. Expert Panel on Gastrointestinal Imaging; Peterson CM, McNamara MM, Kamel IR, et al. ACR Appropriateness Criteria® Right Upper Quadrant Pain. J Am Coll Radiol. 2019;16(5S):S235-S243. https://doi.org/10.1016/j.jacr.2019.02.013
2. Bhosale PR, Javitt MC, Atri M, et al. ACR Appropriateness Criteria® Acute Pelvic Pain in the Reproductive Age Group. Ultrasound Q. 2016;32(2):108-115. https://doi.org/10.1097/RUQ.0000000000000200
3. Revzin MV, Scoutt LM, Garner JG, Moore CL. Right upper quadrant pain: ultrasound first! J Ultrasound Med. 2017;36(10):1975-1985. https://doi.org/10.1002/jum.14274
4. Cooperberg PL, Burhenne HJ. Real-time ultrasonography. Diagnostic technique of choice in calculous gallbladder disease. N Engl J Med. 1980;302(23):1277-1279. https://doi.org/10.1056/NEJM198006053022303
5. Barakos JA, Ralls PW, Lapin SA, et al. Cholelithiasis: evaluation with CT. Radiology. 1987;162(2):415-418. https://doi.org/10.1148/radiology.162.2.3797654
6. Moore C, Meyers AB, Capotasto J, Bokhari J. Prevalence of abnormal CT findings in patients with proven ovarian torsion and a proposed triage schema. Emerg Radiol. 2009;16(2):115-120. https://doi.org/10.1007/s10140-008-0754-x
7. Harfouch N, Stern J, Chowdhary V, et al. Utility of ultrasound after a negative CT abdomen and pelvis in the emergency department. Clin Imaging. 2020;68:29-35. https://doi.org/10.1016/j.clinimag.2020.06.007
8. Adenaw N, Wen J, Pahwa AK, Sheth S, Johnson PT. Decreasing duplicative imaging: inpatient and emergency medicine abdominal ultrasound within 72 hours of abdominal CT. J Am Coll Radiol. 2020;17(5):590-596. https://doi.org/10.1016/j.jacr.2020.03.010
9. Kiewiet JJ, Leeuwenburgh MM, Bipat S, Bossuyt PM, Stoker J, Boermeester MA. A systematic review and meta-analysis of diagnostic performance of imaging in acute cholecystitis. Radiology. 2012;264(3):708-720. https://doi.org/10.1148/radiol.12111561
10. Wertz JR, Lopez JM, Olson D, Thompson WM. Comparing the diagnostic accuracy of ultrasound and CT in evaluating acute cholecystitis. AJR Am J Roentgenol. 2018;211(2):W92-W97. https://doi.org/10.2214/AJR.17.18884
11. Bennett GL, Rusinek H, Lisi V, et al. CT findings in acute gangrenous cholecystitis. AJR Am J Roentgenol. 2002;178(2):275-281. https://doi.org/10.2214/ajr.178.2.1780275
12. Hiatt KD, Ou JJ, Childs DD. Role of ultrasound and CT in the workup of right upper quadrant pain in adults in the emergency department: a retrospective review of more than 2800 cases. AJR Am J Roentgenol. 2020;214(6):1305-1310. https://doi.org/10.2214/AJR.19.22188
13. Gao Y, Lee K, Camacho M. Utility of pelvic ultrasound following negative abdominal and pelvic CT in the emergency room. Clin Radiol. 2013;68(11):e586-e592. https://doi.org/10.1016/j.crad.2013.05.101
14. Natesan S, Lee J, Volkamer H, Thoureen T. Evidence-based medicine approach to abdominal pain. Emerg Med Clin North Am. 2016;34(2):165-190. https://doi.org/10.1016/j.emc.2015.12.008.
15. Dickerson EC, Alam HB, Brown RK, Stojanovska J, Davenport MS; Michigan Radiology Quality Collaborative. In-person communication between radiologists and acute care surgeons leads to significant alterations in surgical decision making. J Am Coll Radiol. 2016;13(8):943-949. https://doi.org/10.1016/j.jacr.2016.02.005

Article PDF
Author and Disclosure Information

1Department of Internal Medicine, Denver Health and Hospital Authority, Denver, Colorado; 2Department of Internal Medicine, University of Colorado Anschutz Medical Campus, Aurora, Colorado; 3Department of Radiology, Denver Health and Hospital Authority, Denver, Colorado.

Disclosures
The authors reported no conflicts of interest.

Publications
Topics
Sections
Files
Files
Author and Disclosure Information

1Department of Internal Medicine, Denver Health and Hospital Authority, Denver, Colorado; 2Department of Internal Medicine, University of Colorado Anschutz Medical Campus, Aurora, Colorado; 3Department of Radiology, Denver Health and Hospital Authority, Denver, Colorado.

Disclosures
The authors reported no conflicts of interest.

Author and Disclosure Information

1Department of Internal Medicine, Denver Health and Hospital Authority, Denver, Colorado; 2Department of Internal Medicine, University of Colorado Anschutz Medical Campus, Aurora, Colorado; 3Department of Radiology, Denver Health and Hospital Authority, Denver, Colorado.

Disclosures
The authors reported no conflicts of interest.

Article PDF
Article PDF
Related Articles

Inspired by the ABIM Foundation’s Choosing Wisely® campaign, the “Things We Do for No Reason” (TWDFNR) series reviews practices that have become common parts of hospital care but may provide little value to our patients. Practices reviewed in the TWDFNR series do not represent clear-cut conclusions or clinical practice standards but are meant as a starting place for research and active discussions among hospitalists and patients. We invite you to be part of that discussion.

Clinical Scenario  

A 70-year-old woman presented to the emergency department (ED) with diffuse abdominal pain, nausea, and vomiting with normal liver function tests and lipase. Computed tomography (CT) of the abdomen and pelvis with intravenous contrast revealed no acute intraabdominal pathology except for an incidentally noted, mildly enlarged but nondistended gallbladder without evident cholelithiasis, pericholecystic fluid, or gallbladder wall edema. The hospitalist orders an abdominal ultrasound to evaluate for acute biliary pathology potentially missed by CT. 

Why You Might Consider Ordering an Abdominal Ultrasound After a Negative CT

Guidelines and expert opinion recommend an “ultrasound-first” approach when patients present with right upper quadrant (RUQ) abdominal pain or pelvic pain of suspected gynecologic origin.1-3 When evaluating suspected biliary disease, experts recommend beginning with ultrasonography based on the speed of obtaining results, absence of radiation exposure, reduced cost, and good diagnostic accuracy.1 Ultrasound has superior sensitivity, of 98%,4 in identifying radiolucent gallstones, compared to CT’s 79% sensitivity.5 Ultrasonography also differentiates gallbladder sludge from cholelithiasis, evaluates the extrahepatic and intrahepatic bile ducts, and can identify alternate causes of RUQ pain.1,3 Since ultrasound has important advantages, a negative initial CT may lead the clinician to consider an ultrasound to evaluate for gallbladder diseases.

Additionally, ultrasound provides improved anatomic detail of pelvic structures when diagnosing endometrial or ovarian pathology2 and improves diagnostic accuracy when the initial CT reveals an abnormal pelvic finding (eg, defining an enlarged ovary on CT as ovarian torsion, a cyst, or an adnexal mass).6 While CT excludes emergent surgical diagnoses, ultrasound may add value in elucidating a cause of the pain, even when urgent surgical management is not necessary.7

Many providers believe that a CT lacks sensitivity for acute biliary or pelvic pathology and will order an ultrasound to avoid missing an important diagnosis.7 Within 6 months at a single center, clinicians ordered 614 abdominal ultrasounds within 72 hours of an abdominal CT; 227 of these orders were to evaluate the gallbladder. Clinicians documented a discussion with a radiologist in only 19% of cases.8

Why Ordering an Ultrasound After a Negative CT Is Unnecessary

While ultrasound is more sensitive for detecting gallstones, the data do not indicate that it is more sensitive than CT for detecting acute cholecystitis. Abdominal ultrasound has a sensitivity for the diagnosis of acute cholecystitis of 81%, with a specificity of 83%,9 while CT has a comparable 85% to 94%9,10 sensitivity and specificity ranging from 59% to 99%.9,11 A recent study using more stringent radiographic criteria (two or more abnormal features) for diagnosing acute cholecystitis found ultrasound and CT had near equivalent sensitivities of 61% and 55%, respectively.12 Even with these stringent criteria, CT had a negative predictive value of 90% and approached 95% when applying a less strict (one feature) criterion.12 As a result, an abdominal ultrasound will rarely diagnose cholecystitis after a normal CT.

A 2020 study evaluated the diagnostic yield and clinical impact of ordering an abdominal or pelvic ultrasound within 24 hours of a negative abdominal CT.7It found that only 3/132 (2.3%) of abdominal ultrasounds ordered after a negative CT revealed acute pathology potentially requiring surgery. Only one of these three patients (1/132) required surgical intervention for confirmed acute cholecystitis.7 The follow-up abdominal ultrasound identified asymptomatic gallstones in 9/132 (6.8%) and gallbladder polyps in 4/132 (3.0%) of cases.7 Selective use of ultrasound after CT for patients with clinically worsening or progressive RUQ pain will avoid missing a “can’t miss” diagnosis and reduce low-yield testing for a majority of patients.

As with abdominal CT and ultrasound, the recommendation for an initial pelvic ultrasound when evaluating female pelvic pain also stems from the reduced cost, absence of radiation exposure, and superior anatomic visualization of the pelvic organs when compared with pelvic CT.2,13 However, as with the results of studies investigating the use of abdominal ultrasound after negative CT, a study of pelvic ultrasound after a negative CT revealed that only 4/126 (3.2%) follow-up ultrasounds had an abnormal finding not identified on CT.13 Pelvic ultrasound found four endometrial abnormalities that did not alter acute management.13 Notably, in 58% of the cases, the indication for ordering the subsequent ultrasound was “rule out ovarian torsion.” However, CT almost always finds a morphologically abnormal ovary in the case of torsion.6 One study and literature review found that all 28 patients studied and all 85 patients from previous studies with proven ovarian torsion had either an adnexal mass or an enlarged ovary on pelvic CT.6 Harfouch et al found that 0 out of 199 pelvic ultrasounds ordered after a negative CT revealed acute surgical pathology, but pelvic ultrasound did identify nonsurgical uterine and ovarian abnormalities.7 In conclusion, when clinicians order CT as the first study to diagnose acute, surgical biliary or gynecologic causes of pain, follow-up ultrasound has a low probability of affecting diagnosis or management if the CT is normal.

When You Should Consider Ultrasound After CT

The previous discussion only applies if hospitalists order an ultrasound within 24 to 48 hours of the initial CT. Time and clinical course are critical diagnostic tools during an admission for abdominal pain. Consider pelvic or abdominal ultrasound based on guideline recommendations if a patient develops new or evolving RUQ or pelvic pain.1,2 The rationale for obtaining the initial negative CT may no longer apply, and the clinician must consider the changing characteristics of the patient’s symptoms. For example, initial CT imaging may miss cholelithiasis in a patient presenting for biliary colic. Under observation, the patient may develop acute cholecystitis, potentially requiring an abdominal ultrasound. Also, the data for pelvic ultrasound apply to a normal CT of the abdomen and pelvis. Ultrasound may help to further evaluate indeterminate findings present on initial CT or if recommended by radiology.

What You Should Do Instead

When the hospitalist assumes care for a patient with abdominal pain and a negative CT, appropriate next steps include taking time to reexamine the differential diagnosis, repeating the history and physical, and communicating directly with a radiologist. These steps ensure the highest diagnostic yield and the lowest cost and help prevent diagnostic error arising from anchoring on the initial negative ED evaluation. Prior research demonstrates that the initial history alone can lead to the correct diagnosis in up to 76% of cases of abdominal pain.14 If repeat evaluation determines that additional imaging is necessary, the American College of Radiology provides evidence-based guidelines to help clinicians determine the correct imaging test based on the clinical situation (Appendix Table).1,2 For example, an equivocal ultrasound or CT exam with continued suspicion for acute cholecystitis or an alternate diagnosis, such as acalculous cholecystitis or choledocholithiasis, merits alternative tests with improved sensitivity and specificity profiles (Tc 99 m hepatobiliary iminodiacetic acid scan, also known as cholescintigraphy, for cholecystitis and acalculous cholecystitis, or magnetic resonance cholangiopancreatography for choledocholithiasis).1

Remember to communicate with the radiologist to rule out “can’t miss” diagnoses, increase mutual understanding of the radiographic test characteristics for specific disease processes, and improve the radiologist’s understanding of the patient’s history and clinical question.15 Collaboration with the radiologist can also determine the need for follow-up imaging and its timing. One single-center study found that surgeons’ diagnostic impression and management changed in 35/100 (35%) cases after an in-person review with the radiologist.15 Observing patients in the hospital with a nondiagnostic initial evaluation but concerning clinical features often allows for either a trial of cure or for the disease process to “declare itself.”14 This allows clinicians to target additional testing to a specific diagnosis and avoid reflexive ordering of additional radiographic studies.

Recommendations

  • Order an ultrasound for initial imaging of RUQ and female pelvic pain.
  • Do not reflexively order an ultrasound within 24 to 48 hours of a negative CT scan to pursue biliary or pelvic pathology.
  • Only order repeat abdominal imaging if clinical circumstances evolve or discussions with a radiologist conclude it will answer a more specific diagnostic question.

Conclusion

In our clinical scenario involving a patient with diffuse abdominal pain and a negative CT, the hospitalist should reevaluate the history, exam, and differential diagnosis before pursuing further diagnostic imaging. Based on the evidence presented, CT has similar diagnostic accuracy to ultrasound for biliary and gynecologic pathologies necessitating urgent surgical management (eg, acute cholecystitis, ovarian torsion), and a follow-up ultrasound adds little. If the utility of imaging remains in question, hospitalist consultation with a radiologist can clarify whether prior imaging answered the clinical question and the diagnostic utility of repeat abdominal imaging. With thoughtful reevaluation of the history and physical, and communication with radiology, hospitalists can reduce unnecessary, low-yield imaging and reduce healthcare costs when evaluating patients with abdominal pain.

Do you think this is a low-value practice? Is this truly a “Thing We Do for No Reason”? Share what you do in your practice and join in the conversation online by retweeting it on Twitter (#TWDFNR) and liking it on Facebook. We invite you to propose ideas for other “Things We Do for No Reason” topics by emailing TWDFNR@hospitalmedicine.org

Inspired by the ABIM Foundation’s Choosing Wisely® campaign, the “Things We Do for No Reason” (TWDFNR) series reviews practices that have become common parts of hospital care but may provide little value to our patients. Practices reviewed in the TWDFNR series do not represent clear-cut conclusions or clinical practice standards but are meant as a starting place for research and active discussions among hospitalists and patients. We invite you to be part of that discussion.

Clinical Scenario  

A 70-year-old woman presented to the emergency department (ED) with diffuse abdominal pain, nausea, and vomiting with normal liver function tests and lipase. Computed tomography (CT) of the abdomen and pelvis with intravenous contrast revealed no acute intraabdominal pathology except for an incidentally noted, mildly enlarged but nondistended gallbladder without evident cholelithiasis, pericholecystic fluid, or gallbladder wall edema. The hospitalist orders an abdominal ultrasound to evaluate for acute biliary pathology potentially missed by CT. 

Why You Might Consider Ordering an Abdominal Ultrasound After a Negative CT

Guidelines and expert opinion recommend an “ultrasound-first” approach when patients present with right upper quadrant (RUQ) abdominal pain or pelvic pain of suspected gynecologic origin.1-3 When evaluating suspected biliary disease, experts recommend beginning with ultrasonography based on the speed of obtaining results, absence of radiation exposure, reduced cost, and good diagnostic accuracy.1 Ultrasound has superior sensitivity, of 98%,4 in identifying radiolucent gallstones, compared to CT’s 79% sensitivity.5 Ultrasonography also differentiates gallbladder sludge from cholelithiasis, evaluates the extrahepatic and intrahepatic bile ducts, and can identify alternate causes of RUQ pain.1,3 Since ultrasound has important advantages, a negative initial CT may lead the clinician to consider an ultrasound to evaluate for gallbladder diseases.

Additionally, ultrasound provides improved anatomic detail of pelvic structures when diagnosing endometrial or ovarian pathology2 and improves diagnostic accuracy when the initial CT reveals an abnormal pelvic finding (eg, defining an enlarged ovary on CT as ovarian torsion, a cyst, or an adnexal mass).6 While CT excludes emergent surgical diagnoses, ultrasound may add value in elucidating a cause of the pain, even when urgent surgical management is not necessary.7

Many providers believe that a CT lacks sensitivity for acute biliary or pelvic pathology and will order an ultrasound to avoid missing an important diagnosis.7 Within 6 months at a single center, clinicians ordered 614 abdominal ultrasounds within 72 hours of an abdominal CT; 227 of these orders were to evaluate the gallbladder. Clinicians documented a discussion with a radiologist in only 19% of cases.8

Why Ordering an Ultrasound After a Negative CT Is Unnecessary

While ultrasound is more sensitive for detecting gallstones, the data do not indicate that it is more sensitive than CT for detecting acute cholecystitis. Abdominal ultrasound has a sensitivity for the diagnosis of acute cholecystitis of 81%, with a specificity of 83%,9 while CT has a comparable 85% to 94%9,10 sensitivity and specificity ranging from 59% to 99%.9,11 A recent study using more stringent radiographic criteria (two or more abnormal features) for diagnosing acute cholecystitis found ultrasound and CT had near equivalent sensitivities of 61% and 55%, respectively.12 Even with these stringent criteria, CT had a negative predictive value of 90% and approached 95% when applying a less strict (one feature) criterion.12 As a result, an abdominal ultrasound will rarely diagnose cholecystitis after a normal CT.

A 2020 study evaluated the diagnostic yield and clinical impact of ordering an abdominal or pelvic ultrasound within 24 hours of a negative abdominal CT.7It found that only 3/132 (2.3%) of abdominal ultrasounds ordered after a negative CT revealed acute pathology potentially requiring surgery. Only one of these three patients (1/132) required surgical intervention for confirmed acute cholecystitis.7 The follow-up abdominal ultrasound identified asymptomatic gallstones in 9/132 (6.8%) and gallbladder polyps in 4/132 (3.0%) of cases.7 Selective use of ultrasound after CT for patients with clinically worsening or progressive RUQ pain will avoid missing a “can’t miss” diagnosis and reduce low-yield testing for a majority of patients.

As with abdominal CT and ultrasound, the recommendation for an initial pelvic ultrasound when evaluating female pelvic pain also stems from the reduced cost, absence of radiation exposure, and superior anatomic visualization of the pelvic organs when compared with pelvic CT.2,13 However, as with the results of studies investigating the use of abdominal ultrasound after negative CT, a study of pelvic ultrasound after a negative CT revealed that only 4/126 (3.2%) follow-up ultrasounds had an abnormal finding not identified on CT.13 Pelvic ultrasound found four endometrial abnormalities that did not alter acute management.13 Notably, in 58% of the cases, the indication for ordering the subsequent ultrasound was “rule out ovarian torsion.” However, CT almost always finds a morphologically abnormal ovary in the case of torsion.6 One study and literature review found that all 28 patients studied and all 85 patients from previous studies with proven ovarian torsion had either an adnexal mass or an enlarged ovary on pelvic CT.6 Harfouch et al found that 0 out of 199 pelvic ultrasounds ordered after a negative CT revealed acute surgical pathology, but pelvic ultrasound did identify nonsurgical uterine and ovarian abnormalities.7 In conclusion, when clinicians order CT as the first study to diagnose acute, surgical biliary or gynecologic causes of pain, follow-up ultrasound has a low probability of affecting diagnosis or management if the CT is normal.

When You Should Consider Ultrasound After CT

The previous discussion only applies if hospitalists order an ultrasound within 24 to 48 hours of the initial CT. Time and clinical course are critical diagnostic tools during an admission for abdominal pain. Consider pelvic or abdominal ultrasound based on guideline recommendations if a patient develops new or evolving RUQ or pelvic pain.1,2 The rationale for obtaining the initial negative CT may no longer apply, and the clinician must consider the changing characteristics of the patient’s symptoms. For example, initial CT imaging may miss cholelithiasis in a patient presenting for biliary colic. Under observation, the patient may develop acute cholecystitis, potentially requiring an abdominal ultrasound. Also, the data for pelvic ultrasound apply to a normal CT of the abdomen and pelvis. Ultrasound may help to further evaluate indeterminate findings present on initial CT or if recommended by radiology.

What You Should Do Instead

When the hospitalist assumes care for a patient with abdominal pain and a negative CT, appropriate next steps include taking time to reexamine the differential diagnosis, repeating the history and physical, and communicating directly with a radiologist. These steps ensure the highest diagnostic yield and the lowest cost and help prevent diagnostic error arising from anchoring on the initial negative ED evaluation. Prior research demonstrates that the initial history alone can lead to the correct diagnosis in up to 76% of cases of abdominal pain.14 If repeat evaluation determines that additional imaging is necessary, the American College of Radiology provides evidence-based guidelines to help clinicians determine the correct imaging test based on the clinical situation (Appendix Table).1,2 For example, an equivocal ultrasound or CT exam with continued suspicion for acute cholecystitis or an alternate diagnosis, such as acalculous cholecystitis or choledocholithiasis, merits alternative tests with improved sensitivity and specificity profiles (Tc 99 m hepatobiliary iminodiacetic acid scan, also known as cholescintigraphy, for cholecystitis and acalculous cholecystitis, or magnetic resonance cholangiopancreatography for choledocholithiasis).1

Remember to communicate with the radiologist to rule out “can’t miss” diagnoses, increase mutual understanding of the radiographic test characteristics for specific disease processes, and improve the radiologist’s understanding of the patient’s history and clinical question.15 Collaboration with the radiologist can also determine the need for follow-up imaging and its timing. One single-center study found that surgeons’ diagnostic impression and management changed in 35/100 (35%) cases after an in-person review with the radiologist.15 Observing patients in the hospital with a nondiagnostic initial evaluation but concerning clinical features often allows for either a trial of cure or for the disease process to “declare itself.”14 This allows clinicians to target additional testing to a specific diagnosis and avoid reflexive ordering of additional radiographic studies.

Recommendations

  • Order an ultrasound for initial imaging of RUQ and female pelvic pain.
  • Do not reflexively order an ultrasound within 24 to 48 hours of a negative CT scan to pursue biliary or pelvic pathology.
  • Only order repeat abdominal imaging if clinical circumstances evolve or discussions with a radiologist conclude it will answer a more specific diagnostic question.

Conclusion

In our clinical scenario involving a patient with diffuse abdominal pain and a negative CT, the hospitalist should reevaluate the history, exam, and differential diagnosis before pursuing further diagnostic imaging. Based on the evidence presented, CT has similar diagnostic accuracy to ultrasound for biliary and gynecologic pathologies necessitating urgent surgical management (eg, acute cholecystitis, ovarian torsion), and a follow-up ultrasound adds little. If the utility of imaging remains in question, hospitalist consultation with a radiologist can clarify whether prior imaging answered the clinical question and the diagnostic utility of repeat abdominal imaging. With thoughtful reevaluation of the history and physical, and communication with radiology, hospitalists can reduce unnecessary, low-yield imaging and reduce healthcare costs when evaluating patients with abdominal pain.

Do you think this is a low-value practice? Is this truly a “Thing We Do for No Reason”? Share what you do in your practice and join in the conversation online by retweeting it on Twitter (#TWDFNR) and liking it on Facebook. We invite you to propose ideas for other “Things We Do for No Reason” topics by emailing TWDFNR@hospitalmedicine.org

References

1. Expert Panel on Gastrointestinal Imaging; Peterson CM, McNamara MM, Kamel IR, et al. ACR Appropriateness Criteria® Right Upper Quadrant Pain. J Am Coll Radiol. 2019;16(5S):S235-S243. https://doi.org/10.1016/j.jacr.2019.02.013
2. Bhosale PR, Javitt MC, Atri M, et al. ACR Appropriateness Criteria® Acute Pelvic Pain in the Reproductive Age Group. Ultrasound Q. 2016;32(2):108-115. https://doi.org/10.1097/RUQ.0000000000000200
3. Revzin MV, Scoutt LM, Garner JG, Moore CL. Right upper quadrant pain: ultrasound first! J Ultrasound Med. 2017;36(10):1975-1985. https://doi.org/10.1002/jum.14274
4. Cooperberg PL, Burhenne HJ. Real-time ultrasonography. Diagnostic technique of choice in calculous gallbladder disease. N Engl J Med. 1980;302(23):1277-1279. https://doi.org/10.1056/NEJM198006053022303
5. Barakos JA, Ralls PW, Lapin SA, et al. Cholelithiasis: evaluation with CT. Radiology. 1987;162(2):415-418. https://doi.org/10.1148/radiology.162.2.3797654
6. Moore C, Meyers AB, Capotasto J, Bokhari J. Prevalence of abnormal CT findings in patients with proven ovarian torsion and a proposed triage schema. Emerg Radiol. 2009;16(2):115-120. https://doi.org/10.1007/s10140-008-0754-x
7. Harfouch N, Stern J, Chowdhary V, et al. Utility of ultrasound after a negative CT abdomen and pelvis in the emergency department. Clin Imaging. 2020;68:29-35. https://doi.org/10.1016/j.clinimag.2020.06.007
8. Adenaw N, Wen J, Pahwa AK, Sheth S, Johnson PT. Decreasing duplicative imaging: inpatient and emergency medicine abdominal ultrasound within 72 hours of abdominal CT. J Am Coll Radiol. 2020;17(5):590-596. https://doi.org/10.1016/j.jacr.2020.03.010
9. Kiewiet JJ, Leeuwenburgh MM, Bipat S, Bossuyt PM, Stoker J, Boermeester MA. A systematic review and meta-analysis of diagnostic performance of imaging in acute cholecystitis. Radiology. 2012;264(3):708-720. https://doi.org/10.1148/radiol.12111561
10. Wertz JR, Lopez JM, Olson D, Thompson WM. Comparing the diagnostic accuracy of ultrasound and CT in evaluating acute cholecystitis. AJR Am J Roentgenol. 2018;211(2):W92-W97. https://doi.org/10.2214/AJR.17.18884
11. Bennett GL, Rusinek H, Lisi V, et al. CT findings in acute gangrenous cholecystitis. AJR Am J Roentgenol. 2002;178(2):275-281. https://doi.org/10.2214/ajr.178.2.1780275
12. Hiatt KD, Ou JJ, Childs DD. Role of ultrasound and CT in the workup of right upper quadrant pain in adults in the emergency department: a retrospective review of more than 2800 cases. AJR Am J Roentgenol. 2020;214(6):1305-1310. https://doi.org/10.2214/AJR.19.22188
13. Gao Y, Lee K, Camacho M. Utility of pelvic ultrasound following negative abdominal and pelvic CT in the emergency room. Clin Radiol. 2013;68(11):e586-e592. https://doi.org/10.1016/j.crad.2013.05.101
14. Natesan S, Lee J, Volkamer H, Thoureen T. Evidence-based medicine approach to abdominal pain. Emerg Med Clin North Am. 2016;34(2):165-190. https://doi.org/10.1016/j.emc.2015.12.008.
15. Dickerson EC, Alam HB, Brown RK, Stojanovska J, Davenport MS; Michigan Radiology Quality Collaborative. In-person communication between radiologists and acute care surgeons leads to significant alterations in surgical decision making. J Am Coll Radiol. 2016;13(8):943-949. https://doi.org/10.1016/j.jacr.2016.02.005

References

1. Expert Panel on Gastrointestinal Imaging; Peterson CM, McNamara MM, Kamel IR, et al. ACR Appropriateness Criteria® Right Upper Quadrant Pain. J Am Coll Radiol. 2019;16(5S):S235-S243. https://doi.org/10.1016/j.jacr.2019.02.013
2. Bhosale PR, Javitt MC, Atri M, et al. ACR Appropriateness Criteria® Acute Pelvic Pain in the Reproductive Age Group. Ultrasound Q. 2016;32(2):108-115. https://doi.org/10.1097/RUQ.0000000000000200
3. Revzin MV, Scoutt LM, Garner JG, Moore CL. Right upper quadrant pain: ultrasound first! J Ultrasound Med. 2017;36(10):1975-1985. https://doi.org/10.1002/jum.14274
4. Cooperberg PL, Burhenne HJ. Real-time ultrasonography. Diagnostic technique of choice in calculous gallbladder disease. N Engl J Med. 1980;302(23):1277-1279. https://doi.org/10.1056/NEJM198006053022303
5. Barakos JA, Ralls PW, Lapin SA, et al. Cholelithiasis: evaluation with CT. Radiology. 1987;162(2):415-418. https://doi.org/10.1148/radiology.162.2.3797654
6. Moore C, Meyers AB, Capotasto J, Bokhari J. Prevalence of abnormal CT findings in patients with proven ovarian torsion and a proposed triage schema. Emerg Radiol. 2009;16(2):115-120. https://doi.org/10.1007/s10140-008-0754-x
7. Harfouch N, Stern J, Chowdhary V, et al. Utility of ultrasound after a negative CT abdomen and pelvis in the emergency department. Clin Imaging. 2020;68:29-35. https://doi.org/10.1016/j.clinimag.2020.06.007
8. Adenaw N, Wen J, Pahwa AK, Sheth S, Johnson PT. Decreasing duplicative imaging: inpatient and emergency medicine abdominal ultrasound within 72 hours of abdominal CT. J Am Coll Radiol. 2020;17(5):590-596. https://doi.org/10.1016/j.jacr.2020.03.010
9. Kiewiet JJ, Leeuwenburgh MM, Bipat S, Bossuyt PM, Stoker J, Boermeester MA. A systematic review and meta-analysis of diagnostic performance of imaging in acute cholecystitis. Radiology. 2012;264(3):708-720. https://doi.org/10.1148/radiol.12111561
10. Wertz JR, Lopez JM, Olson D, Thompson WM. Comparing the diagnostic accuracy of ultrasound and CT in evaluating acute cholecystitis. AJR Am J Roentgenol. 2018;211(2):W92-W97. https://doi.org/10.2214/AJR.17.18884
11. Bennett GL, Rusinek H, Lisi V, et al. CT findings in acute gangrenous cholecystitis. AJR Am J Roentgenol. 2002;178(2):275-281. https://doi.org/10.2214/ajr.178.2.1780275
12. Hiatt KD, Ou JJ, Childs DD. Role of ultrasound and CT in the workup of right upper quadrant pain in adults in the emergency department: a retrospective review of more than 2800 cases. AJR Am J Roentgenol. 2020;214(6):1305-1310. https://doi.org/10.2214/AJR.19.22188
13. Gao Y, Lee K, Camacho M. Utility of pelvic ultrasound following negative abdominal and pelvic CT in the emergency room. Clin Radiol. 2013;68(11):e586-e592. https://doi.org/10.1016/j.crad.2013.05.101
14. Natesan S, Lee J, Volkamer H, Thoureen T. Evidence-based medicine approach to abdominal pain. Emerg Med Clin North Am. 2016;34(2):165-190. https://doi.org/10.1016/j.emc.2015.12.008.
15. Dickerson EC, Alam HB, Brown RK, Stojanovska J, Davenport MS; Michigan Radiology Quality Collaborative. In-person communication between radiologists and acute care surgeons leads to significant alterations in surgical decision making. J Am Coll Radiol. 2016;13(8):943-949. https://doi.org/10.1016/j.jacr.2016.02.005

Publications
Publications
Topics
Article Type
Display Headline
Things We Do For No Reason™: Ultrasonography After an Initial Negative CT in Patients Presenting With Acute Abdominal or Pelvic Pain
Display Headline
Things We Do For No Reason™: Ultrasonography After an Initial Negative CT in Patients Presenting With Acute Abdominal or Pelvic Pain
Sections
Article Source

© 2021 Society of Hospital Medicine

Citation Override
J Hosp Med. Published Online Only November 17, 2021. DOI: 10.12788/jhm.3704
Disallow All Ads
Correspondence Location
John M Cunningham, MD; Email: John.cunningham@dhha.org; Telephone: 651-269-1449.
Content Gating
Gated (full article locked unless allowed per User)
Alternative CME
Disqus Comments
Default
Use ProPublica
Hide sidebar & use full width
render the right sidebar.
Conference Recap Checkbox
Not Conference Recap
Clinical Edge
Display the Slideshow in this Article
Gating Strategy
First Page Free
Medscape Article
Display survey writer
Reuters content
Disable Inline Native ads
WebMD Article
Article PDF Media
Image
Teambase ID
18001F11.SIG
Disable zoom
Off
Media Files

Improving Healthcare Access for Patients With Limited English Proficiency

Article Type
Changed
Wed, 11/17/2021 - 01:15
Display Headline
Improving Healthcare Access for Patients With Limited English Proficiency

Patients whose primary language is not English and who have a limited ability to read, speak, write, or understand English experience worse healthcare access than their English-speaking counterparts, highlighted by fewer healthcare visits and filled prescription medications.1 These patients with limited English proficiency (LEP) face additional barriers to quality healthcare during the COVID-19 pandemic, including lower rates of telehealth use,2 lower rates of COVID-19 testing,3 and challenges with implementing high-quality interpretation.4 As a result of such long-standing disparities, healthcare policy has focused on improving access to language-concordant care.

The Civil Rights Act of 1964 and Department of Health and Human Services (HHS) regulations require recipients of federal financial assistance to provide reasonable access to programs, services, and activities to persons with LEP. Section 1557 of the Affordable Care Act extends Title VI nondiscrimination standards to the entire healthcare system, including insurers and health plans. In 2016, the Obama administration implemented Section 1557 through regulations that clarified and expanded language accessibility standards, although several years later the Trump administration sought to weaken the rule’s protections.

Although the requirement to make healthcare accessible to patients with LEP is unequivocal, “reasonable access” provides clinicians who accept federal funds with flexibility in how they deliver language access services. Differing interpretations of what is considered “reasonable” drives variation in how and when medical facilities provide interpreter services. This results in inconsistency of services provided across care settings and decreased availability of language-concordant care. For example, less than one-third of outpatient providers regularly use qualified interpreters when seeing patients with LEP.5 Furthermore, only about 69% of hospitals offer language access services.6 Clinician underutilization of interpreters for patients with LEP results in poor patient satisfaction and worse health outcomes.7 In light of the Biden administration’s commitment to civil rights and healthcare access, we outline a roadmap of actions that this administration must take to ensure access to basic communication needs and improve health equity.

IDENTIFY CURRENT OPPORTUNITIES FOR IMPROVING REGULATIONS

The Trump-era rules loosened the requirement that care providers notify patients with LEP of their rights to language services and provide instructions on how to access these services. These rules also allowed providers to replace video-based interpreter services with audio-based services, which disproportionately impacts patients in rural areas, who rely on high-quality video interpretation to facilitate telehealth visits, especially during the ongoing COVID-19 pandemic, which has increased patient reliance on telehealth infrastructure for primary healthcare access. The Trump administration weakened both the standards for ensuring adequate access to language assistance services and the compliance tests used to assess whether healthcare organizations have met those standards. The revised regulations deem certain healthcare services effectively exempt from interpreter standards if the projected number of encountered patients with LEP falls below preset minimums and a healthcare entity considers the cost of compliance onerous.8 The Trump administration justified these changes as a cost-savings matter, but the suboptimal care resulting from these changes will likely offset any savings.

RESTORE AND IMPROVE LANGUAGE ACCESS PROVISIONS

To restore a strong commitment to language access, the HHS Office for Civil Rights, which the Biden administration has targeted for new investments in fiscal year 2022, should reestablish the HHS Language Access Steering Committee. This committee maintains criteria that guide covered health entities in developing language access compliance plans. Maintaining such plans should become a basic element of the revised Section 1557 compliance rules and should also become a core feature of the standards applicable to Joint Commission–accredited healthcare organizations. In addition, the Center for Medicare and Medicaid Innovation, whose mission is to identify, test, and implement major improvements in healthcare quality and efficiency, could undertake a special project to identify and incentivize adoption of the most effective language access innovations for incorporation into language access plans.

RESTRUCTURE AND STRENGTHEN COMPLIANCE FOR LANGUAGE ACCESS

Section 1557, as well as federal standards governing the conditions of participation in federal healthcare programs, should ensure that covered entities report on interpreter use and associated patient health outcomes for patients with LEP. Overall compliance measurement and reporting in connection with language access is a matter of basic health equity. Currently, any individual who believes they have experienced discrimination based on language can report a potential violation for federal investigation. But an individual remedy is insufficient because it cannot ensure the types of systemic changes essential to overcome decades of structural exclusion and achieve broader health equity. Further, barriers from digital literacy gaps and fear of legal repercussions, such as deportation, hamper individual reporting efforts. Any policy focused on improving language access use should apply to all patients, regardless of immigration status.

INCENTIVIZE LANGUAGE-CONCORDANT CARE

Ultimately, there is little benefit to imposing standards without a concomitant assurance of the resources needed to achieve full adoption and ongoing compliance. For this reason, a commitment to language access must be accompanied by payment reforms that enable Medicare and Medicaid providers to embrace this vital feature of accessible healthcare by recognizing interpreter costs as part of the clinical encounter and care management. Covered entities could use these resources either to strengthen their own staffing or contract with third-party interpreter services organizations. Currently, the Centers for Medicare & Medicaid Services (CMS) allow states to claim federal matching funds for language assistance services provided to Medicaid enrollees, though rates are dependent on how service claims are categorized. State Medicaid programs can facilitate the provision of such services by optimizing reimbursements for provider organizations under CMS policy.

The Merit-based Incentive Payment System (MIPS) provides an opportunity to incorporate the provision of interpreter services into quality measure reporting. Such efforts could improve health equity and address long-standing needs for research into how language barriers affect healthcare outcomes. Given that analyses of inaugural MIPS data revealed that safety-net practices were more likely to receive lower composite scores, additional scoring flexibility under pay-for-performance schemes (rather than strict penalties) may be necessary to ensure the solvency of safety net practices that disproportionately care for patients with LEP.9 Here, CMMI can play a critical role in expanding the use of patient-facing resources by designing new alternative payment models that reward participants for providing high-quality language concordant care.

The COVID-19 pandemic has exacerbated disparities in care for patients with LEP and even starker disparities among immigrant communities and patients of color. These disparities will only worsen if regulations aimed at improving access to language access services are not reinstated and improved. Failing to focus on healthcare access for patients with LEP hurts individual patient health and public health, as we have seen through lower rates of testing and vaccination in communities of color during this pandemic. The Biden administration can put healthcare on a more equitable pathway by expanding and strengthening language access as a core feature of healthcare, as a matter of both civil rights and health care quality.

Acknowledgments

The authors thank Jocelyn Samuels, JD, and Sara Rosenbaum, JD, for comments and guidance on an earlier draft of this article.

References

1. Himmelstein J, Himmelstein DU, Woolhandler S, et al. Health care spending and use among Hispanic adults with and without limited English proficiency, 1999–2018. Health Aff (Millwood). 2021;40(7):1126-1134. https://doi.org/10.1377/hlthaff.2020.02510
2. Rodriguez JA, Saadi A, Schwamm LH, Bates DW, Samal L. Disparities in telehealth use among California patients with limited English proficiency. Health Aff (Millwood). 2021;40(3):487-495. https://doi.org/10.1377/hlthaff.2020.00823
3. Kim HN, Lan KF, Nkyekyer E, et al. Assessment of disparities in COVID-19 testing and infection across language groups in Seattle, Washington. JAMA Netw Open. 2020;3(9):e2021213. https://doi.org/10.1001/jamanetworkopen.2020.21213
4. Page KR, Flores-Miller A. Lessons we’ve learned - Covid-19 and the undocumented Latinx community. N Engl J Med. 2021;384(1):5-7. https://doi.org/10.1056/NEJMp2024897
5. Schulson LB, Anderson TS. National estimates of professional interpreter use in the ambulatory setting. J Gen Intern Med. Published online November 2, 2020. https://doi.org/10.1007/s11606-020-06336-6
6. Schiaffino MK, Nara A, Mao L. Language services in hospitals vary by ownership and location. Health Aff (Millwood). 2016;35(8):1399-1403. https://doi.org/10.1377/hlthaff.2015.0955
7. Taira BR, Kim K, Mody N. Hospital and health system–level interventions to improve care for limited English proficiency patients: a systematic review. Jt Comm J Qual Patient Saf. 2019;45(6):446-458. https://doi.org/10.1016/j.jcjq.2019.02.005
8. Musumeci M, Kates J, Dawson L, Salganicoff A, Sobel L, Artiga S. The Trump administration’s final rule on Section 1557 non-discrimination regulations under the ACA and current status. Kaiser Family Foundation. September 18, 2020. Accessed September 2, 2021. https://www.kff.org/racial-equity-and-health-policy/issue-brief/the-trump-administrations-final-rule-on-section-1557-non-discrimination-regulations-under-the-aca-and-current-status/
9. Liao JM, Navathe AS. Does the Merit-Based Incentive Payment System disproportionately affect safety-net practices? JAMA Health Forum. 2020;1(5):e200452. https://doi.org/10.1001/jamahealthforum.2020.0452

Article PDF
Author and Disclosure Information

1Harvard Medical School, Boston, Massachusetts; 2Harvard Business School, Boston, Massachusetts; 3Harvard Kennedy School of Government, Cambridge, Massachusetts; 4Center for Surgery and Public Health, Department of Surgery, Brigham and Women’s Hospital, Harvard Medical School, Boston, Massachusetts.

Disclosures
Mr Uppal reports income from Quantified Ventures and Ironwood Medical Information Technologies. The other authors have no disclosures or conflicts of interest.

Publications
Topics
Sections
Author and Disclosure Information

1Harvard Medical School, Boston, Massachusetts; 2Harvard Business School, Boston, Massachusetts; 3Harvard Kennedy School of Government, Cambridge, Massachusetts; 4Center for Surgery and Public Health, Department of Surgery, Brigham and Women’s Hospital, Harvard Medical School, Boston, Massachusetts.

Disclosures
Mr Uppal reports income from Quantified Ventures and Ironwood Medical Information Technologies. The other authors have no disclosures or conflicts of interest.

Author and Disclosure Information

1Harvard Medical School, Boston, Massachusetts; 2Harvard Business School, Boston, Massachusetts; 3Harvard Kennedy School of Government, Cambridge, Massachusetts; 4Center for Surgery and Public Health, Department of Surgery, Brigham and Women’s Hospital, Harvard Medical School, Boston, Massachusetts.

Disclosures
Mr Uppal reports income from Quantified Ventures and Ironwood Medical Information Technologies. The other authors have no disclosures or conflicts of interest.

Article PDF
Article PDF
Related Articles

Patients whose primary language is not English and who have a limited ability to read, speak, write, or understand English experience worse healthcare access than their English-speaking counterparts, highlighted by fewer healthcare visits and filled prescription medications.1 These patients with limited English proficiency (LEP) face additional barriers to quality healthcare during the COVID-19 pandemic, including lower rates of telehealth use,2 lower rates of COVID-19 testing,3 and challenges with implementing high-quality interpretation.4 As a result of such long-standing disparities, healthcare policy has focused on improving access to language-concordant care.

The Civil Rights Act of 1964 and Department of Health and Human Services (HHS) regulations require recipients of federal financial assistance to provide reasonable access to programs, services, and activities to persons with LEP. Section 1557 of the Affordable Care Act extends Title VI nondiscrimination standards to the entire healthcare system, including insurers and health plans. In 2016, the Obama administration implemented Section 1557 through regulations that clarified and expanded language accessibility standards, although several years later the Trump administration sought to weaken the rule’s protections.

Although the requirement to make healthcare accessible to patients with LEP is unequivocal, “reasonable access” provides clinicians who accept federal funds with flexibility in how they deliver language access services. Differing interpretations of what is considered “reasonable” drives variation in how and when medical facilities provide interpreter services. This results in inconsistency of services provided across care settings and decreased availability of language-concordant care. For example, less than one-third of outpatient providers regularly use qualified interpreters when seeing patients with LEP.5 Furthermore, only about 69% of hospitals offer language access services.6 Clinician underutilization of interpreters for patients with LEP results in poor patient satisfaction and worse health outcomes.7 In light of the Biden administration’s commitment to civil rights and healthcare access, we outline a roadmap of actions that this administration must take to ensure access to basic communication needs and improve health equity.

IDENTIFY CURRENT OPPORTUNITIES FOR IMPROVING REGULATIONS

The Trump-era rules loosened the requirement that care providers notify patients with LEP of their rights to language services and provide instructions on how to access these services. These rules also allowed providers to replace video-based interpreter services with audio-based services, which disproportionately impacts patients in rural areas, who rely on high-quality video interpretation to facilitate telehealth visits, especially during the ongoing COVID-19 pandemic, which has increased patient reliance on telehealth infrastructure for primary healthcare access. The Trump administration weakened both the standards for ensuring adequate access to language assistance services and the compliance tests used to assess whether healthcare organizations have met those standards. The revised regulations deem certain healthcare services effectively exempt from interpreter standards if the projected number of encountered patients with LEP falls below preset minimums and a healthcare entity considers the cost of compliance onerous.8 The Trump administration justified these changes as a cost-savings matter, but the suboptimal care resulting from these changes will likely offset any savings.

RESTORE AND IMPROVE LANGUAGE ACCESS PROVISIONS

To restore a strong commitment to language access, the HHS Office for Civil Rights, which the Biden administration has targeted for new investments in fiscal year 2022, should reestablish the HHS Language Access Steering Committee. This committee maintains criteria that guide covered health entities in developing language access compliance plans. Maintaining such plans should become a basic element of the revised Section 1557 compliance rules and should also become a core feature of the standards applicable to Joint Commission–accredited healthcare organizations. In addition, the Center for Medicare and Medicaid Innovation, whose mission is to identify, test, and implement major improvements in healthcare quality and efficiency, could undertake a special project to identify and incentivize adoption of the most effective language access innovations for incorporation into language access plans.

RESTRUCTURE AND STRENGTHEN COMPLIANCE FOR LANGUAGE ACCESS

Section 1557, as well as federal standards governing the conditions of participation in federal healthcare programs, should ensure that covered entities report on interpreter use and associated patient health outcomes for patients with LEP. Overall compliance measurement and reporting in connection with language access is a matter of basic health equity. Currently, any individual who believes they have experienced discrimination based on language can report a potential violation for federal investigation. But an individual remedy is insufficient because it cannot ensure the types of systemic changes essential to overcome decades of structural exclusion and achieve broader health equity. Further, barriers from digital literacy gaps and fear of legal repercussions, such as deportation, hamper individual reporting efforts. Any policy focused on improving language access use should apply to all patients, regardless of immigration status.

INCENTIVIZE LANGUAGE-CONCORDANT CARE

Ultimately, there is little benefit to imposing standards without a concomitant assurance of the resources needed to achieve full adoption and ongoing compliance. For this reason, a commitment to language access must be accompanied by payment reforms that enable Medicare and Medicaid providers to embrace this vital feature of accessible healthcare by recognizing interpreter costs as part of the clinical encounter and care management. Covered entities could use these resources either to strengthen their own staffing or contract with third-party interpreter services organizations. Currently, the Centers for Medicare & Medicaid Services (CMS) allow states to claim federal matching funds for language assistance services provided to Medicaid enrollees, though rates are dependent on how service claims are categorized. State Medicaid programs can facilitate the provision of such services by optimizing reimbursements for provider organizations under CMS policy.

The Merit-based Incentive Payment System (MIPS) provides an opportunity to incorporate the provision of interpreter services into quality measure reporting. Such efforts could improve health equity and address long-standing needs for research into how language barriers affect healthcare outcomes. Given that analyses of inaugural MIPS data revealed that safety-net practices were more likely to receive lower composite scores, additional scoring flexibility under pay-for-performance schemes (rather than strict penalties) may be necessary to ensure the solvency of safety net practices that disproportionately care for patients with LEP.9 Here, CMMI can play a critical role in expanding the use of patient-facing resources by designing new alternative payment models that reward participants for providing high-quality language concordant care.

The COVID-19 pandemic has exacerbated disparities in care for patients with LEP and even starker disparities among immigrant communities and patients of color. These disparities will only worsen if regulations aimed at improving access to language access services are not reinstated and improved. Failing to focus on healthcare access for patients with LEP hurts individual patient health and public health, as we have seen through lower rates of testing and vaccination in communities of color during this pandemic. The Biden administration can put healthcare on a more equitable pathway by expanding and strengthening language access as a core feature of healthcare, as a matter of both civil rights and health care quality.

Acknowledgments

The authors thank Jocelyn Samuels, JD, and Sara Rosenbaum, JD, for comments and guidance on an earlier draft of this article.

Patients whose primary language is not English and who have a limited ability to read, speak, write, or understand English experience worse healthcare access than their English-speaking counterparts, highlighted by fewer healthcare visits and filled prescription medications.1 These patients with limited English proficiency (LEP) face additional barriers to quality healthcare during the COVID-19 pandemic, including lower rates of telehealth use,2 lower rates of COVID-19 testing,3 and challenges with implementing high-quality interpretation.4 As a result of such long-standing disparities, healthcare policy has focused on improving access to language-concordant care.

The Civil Rights Act of 1964 and Department of Health and Human Services (HHS) regulations require recipients of federal financial assistance to provide reasonable access to programs, services, and activities to persons with LEP. Section 1557 of the Affordable Care Act extends Title VI nondiscrimination standards to the entire healthcare system, including insurers and health plans. In 2016, the Obama administration implemented Section 1557 through regulations that clarified and expanded language accessibility standards, although several years later the Trump administration sought to weaken the rule’s protections.

Although the requirement to make healthcare accessible to patients with LEP is unequivocal, “reasonable access” provides clinicians who accept federal funds with flexibility in how they deliver language access services. Differing interpretations of what is considered “reasonable” drives variation in how and when medical facilities provide interpreter services. This results in inconsistency of services provided across care settings and decreased availability of language-concordant care. For example, less than one-third of outpatient providers regularly use qualified interpreters when seeing patients with LEP.5 Furthermore, only about 69% of hospitals offer language access services.6 Clinician underutilization of interpreters for patients with LEP results in poor patient satisfaction and worse health outcomes.7 In light of the Biden administration’s commitment to civil rights and healthcare access, we outline a roadmap of actions that this administration must take to ensure access to basic communication needs and improve health equity.

IDENTIFY CURRENT OPPORTUNITIES FOR IMPROVING REGULATIONS

The Trump-era rules loosened the requirement that care providers notify patients with LEP of their rights to language services and provide instructions on how to access these services. These rules also allowed providers to replace video-based interpreter services with audio-based services, which disproportionately impacts patients in rural areas, who rely on high-quality video interpretation to facilitate telehealth visits, especially during the ongoing COVID-19 pandemic, which has increased patient reliance on telehealth infrastructure for primary healthcare access. The Trump administration weakened both the standards for ensuring adequate access to language assistance services and the compliance tests used to assess whether healthcare organizations have met those standards. The revised regulations deem certain healthcare services effectively exempt from interpreter standards if the projected number of encountered patients with LEP falls below preset minimums and a healthcare entity considers the cost of compliance onerous.8 The Trump administration justified these changes as a cost-savings matter, but the suboptimal care resulting from these changes will likely offset any savings.

RESTORE AND IMPROVE LANGUAGE ACCESS PROVISIONS

To restore a strong commitment to language access, the HHS Office for Civil Rights, which the Biden administration has targeted for new investments in fiscal year 2022, should reestablish the HHS Language Access Steering Committee. This committee maintains criteria that guide covered health entities in developing language access compliance plans. Maintaining such plans should become a basic element of the revised Section 1557 compliance rules and should also become a core feature of the standards applicable to Joint Commission–accredited healthcare organizations. In addition, the Center for Medicare and Medicaid Innovation, whose mission is to identify, test, and implement major improvements in healthcare quality and efficiency, could undertake a special project to identify and incentivize adoption of the most effective language access innovations for incorporation into language access plans.

RESTRUCTURE AND STRENGTHEN COMPLIANCE FOR LANGUAGE ACCESS

Section 1557, as well as federal standards governing the conditions of participation in federal healthcare programs, should ensure that covered entities report on interpreter use and associated patient health outcomes for patients with LEP. Overall compliance measurement and reporting in connection with language access is a matter of basic health equity. Currently, any individual who believes they have experienced discrimination based on language can report a potential violation for federal investigation. But an individual remedy is insufficient because it cannot ensure the types of systemic changes essential to overcome decades of structural exclusion and achieve broader health equity. Further, barriers from digital literacy gaps and fear of legal repercussions, such as deportation, hamper individual reporting efforts. Any policy focused on improving language access use should apply to all patients, regardless of immigration status.

INCENTIVIZE LANGUAGE-CONCORDANT CARE

Ultimately, there is little benefit to imposing standards without a concomitant assurance of the resources needed to achieve full adoption and ongoing compliance. For this reason, a commitment to language access must be accompanied by payment reforms that enable Medicare and Medicaid providers to embrace this vital feature of accessible healthcare by recognizing interpreter costs as part of the clinical encounter and care management. Covered entities could use these resources either to strengthen their own staffing or contract with third-party interpreter services organizations. Currently, the Centers for Medicare & Medicaid Services (CMS) allow states to claim federal matching funds for language assistance services provided to Medicaid enrollees, though rates are dependent on how service claims are categorized. State Medicaid programs can facilitate the provision of such services by optimizing reimbursements for provider organizations under CMS policy.

The Merit-based Incentive Payment System (MIPS) provides an opportunity to incorporate the provision of interpreter services into quality measure reporting. Such efforts could improve health equity and address long-standing needs for research into how language barriers affect healthcare outcomes. Given that analyses of inaugural MIPS data revealed that safety-net practices were more likely to receive lower composite scores, additional scoring flexibility under pay-for-performance schemes (rather than strict penalties) may be necessary to ensure the solvency of safety net practices that disproportionately care for patients with LEP.9 Here, CMMI can play a critical role in expanding the use of patient-facing resources by designing new alternative payment models that reward participants for providing high-quality language concordant care.

The COVID-19 pandemic has exacerbated disparities in care for patients with LEP and even starker disparities among immigrant communities and patients of color. These disparities will only worsen if regulations aimed at improving access to language access services are not reinstated and improved. Failing to focus on healthcare access for patients with LEP hurts individual patient health and public health, as we have seen through lower rates of testing and vaccination in communities of color during this pandemic. The Biden administration can put healthcare on a more equitable pathway by expanding and strengthening language access as a core feature of healthcare, as a matter of both civil rights and health care quality.

Acknowledgments

The authors thank Jocelyn Samuels, JD, and Sara Rosenbaum, JD, for comments and guidance on an earlier draft of this article.

References

1. Himmelstein J, Himmelstein DU, Woolhandler S, et al. Health care spending and use among Hispanic adults with and without limited English proficiency, 1999–2018. Health Aff (Millwood). 2021;40(7):1126-1134. https://doi.org/10.1377/hlthaff.2020.02510
2. Rodriguez JA, Saadi A, Schwamm LH, Bates DW, Samal L. Disparities in telehealth use among California patients with limited English proficiency. Health Aff (Millwood). 2021;40(3):487-495. https://doi.org/10.1377/hlthaff.2020.00823
3. Kim HN, Lan KF, Nkyekyer E, et al. Assessment of disparities in COVID-19 testing and infection across language groups in Seattle, Washington. JAMA Netw Open. 2020;3(9):e2021213. https://doi.org/10.1001/jamanetworkopen.2020.21213
4. Page KR, Flores-Miller A. Lessons we’ve learned - Covid-19 and the undocumented Latinx community. N Engl J Med. 2021;384(1):5-7. https://doi.org/10.1056/NEJMp2024897
5. Schulson LB, Anderson TS. National estimates of professional interpreter use in the ambulatory setting. J Gen Intern Med. Published online November 2, 2020. https://doi.org/10.1007/s11606-020-06336-6
6. Schiaffino MK, Nara A, Mao L. Language services in hospitals vary by ownership and location. Health Aff (Millwood). 2016;35(8):1399-1403. https://doi.org/10.1377/hlthaff.2015.0955
7. Taira BR, Kim K, Mody N. Hospital and health system–level interventions to improve care for limited English proficiency patients: a systematic review. Jt Comm J Qual Patient Saf. 2019;45(6):446-458. https://doi.org/10.1016/j.jcjq.2019.02.005
8. Musumeci M, Kates J, Dawson L, Salganicoff A, Sobel L, Artiga S. The Trump administration’s final rule on Section 1557 non-discrimination regulations under the ACA and current status. Kaiser Family Foundation. September 18, 2020. Accessed September 2, 2021. https://www.kff.org/racial-equity-and-health-policy/issue-brief/the-trump-administrations-final-rule-on-section-1557-non-discrimination-regulations-under-the-aca-and-current-status/
9. Liao JM, Navathe AS. Does the Merit-Based Incentive Payment System disproportionately affect safety-net practices? JAMA Health Forum. 2020;1(5):e200452. https://doi.org/10.1001/jamahealthforum.2020.0452

References

1. Himmelstein J, Himmelstein DU, Woolhandler S, et al. Health care spending and use among Hispanic adults with and without limited English proficiency, 1999–2018. Health Aff (Millwood). 2021;40(7):1126-1134. https://doi.org/10.1377/hlthaff.2020.02510
2. Rodriguez JA, Saadi A, Schwamm LH, Bates DW, Samal L. Disparities in telehealth use among California patients with limited English proficiency. Health Aff (Millwood). 2021;40(3):487-495. https://doi.org/10.1377/hlthaff.2020.00823
3. Kim HN, Lan KF, Nkyekyer E, et al. Assessment of disparities in COVID-19 testing and infection across language groups in Seattle, Washington. JAMA Netw Open. 2020;3(9):e2021213. https://doi.org/10.1001/jamanetworkopen.2020.21213
4. Page KR, Flores-Miller A. Lessons we’ve learned - Covid-19 and the undocumented Latinx community. N Engl J Med. 2021;384(1):5-7. https://doi.org/10.1056/NEJMp2024897
5. Schulson LB, Anderson TS. National estimates of professional interpreter use in the ambulatory setting. J Gen Intern Med. Published online November 2, 2020. https://doi.org/10.1007/s11606-020-06336-6
6. Schiaffino MK, Nara A, Mao L. Language services in hospitals vary by ownership and location. Health Aff (Millwood). 2016;35(8):1399-1403. https://doi.org/10.1377/hlthaff.2015.0955
7. Taira BR, Kim K, Mody N. Hospital and health system–level interventions to improve care for limited English proficiency patients: a systematic review. Jt Comm J Qual Patient Saf. 2019;45(6):446-458. https://doi.org/10.1016/j.jcjq.2019.02.005
8. Musumeci M, Kates J, Dawson L, Salganicoff A, Sobel L, Artiga S. The Trump administration’s final rule on Section 1557 non-discrimination regulations under the ACA and current status. Kaiser Family Foundation. September 18, 2020. Accessed September 2, 2021. https://www.kff.org/racial-equity-and-health-policy/issue-brief/the-trump-administrations-final-rule-on-section-1557-non-discrimination-regulations-under-the-aca-and-current-status/
9. Liao JM, Navathe AS. Does the Merit-Based Incentive Payment System disproportionately affect safety-net practices? JAMA Health Forum. 2020;1(5):e200452. https://doi.org/10.1001/jamahealthforum.2020.0452

Publications
Publications
Topics
Article Type
Display Headline
Improving Healthcare Access for Patients With Limited English Proficiency
Display Headline
Improving Healthcare Access for Patients With Limited English Proficiency
Sections
Article Source

© 2021 Society of Hospital Medicine

Citation Override
J Hosp Med. Published Online Only November 17, 2021. DOI: 10.12788/jhm.3701
Disallow All Ads
Correspondence Location
Gezzer Ortega, MD, MPH; Email: gortega1@bwh.harvard.edu; Telephone: (617) 732-4082; Twitter: @DrGezzer.
Content Gating
Gated (full article locked unless allowed per User)
Alternative CME
Disqus Comments
Default
Use ProPublica
Hide sidebar & use full width
render the right sidebar.
Conference Recap Checkbox
Not Conference Recap
Clinical Edge
Display the Slideshow in this Article
Gating Strategy
First Page Free
Medscape Article
Display survey writer
Reuters content
Disable Inline Native ads
WebMD Article
Article PDF Media
Image
Teambase ID
18001F09.SIG
Disable zoom
Off

The No Judgment Zone: Building Trust Through Trustworthiness

Article Type
Changed
Mon, 11/08/2021 - 11:49
Display Headline
The No Judgment Zone: Building Trust Through Trustworthiness

The collective struggle felt by healthcare workers simultaneously learning about and caring for patients impacted by SARS-CoV2 infections throughout 2020 was physically and emotionally exhausting. The majority of us had never experienced a global pandemic. Beyond our work in the professional arena of ambulatory practices and hospitals, we also felt the soul-crushing impact of the pandemic in every other aspect of our lives. Preexisting health disparities were amplified by COVID-19. Some of the most affected communities also bore the weight of an additional tsunami of ongoing racial injustice.1 And as healthcare workers, we did our best to process and navigate it all while trying to avoid burnout—as well as being infected with COVID-19 ourselves. When the news of the highly effective vaccines against SARS-CoV2 receiving emergency use authorization broke late in 2020, it felt like a light at the end of a very dark tunnel.

In the weeks preceding wide availability of the vaccines, it became apparent that significant numbers of people lacked confidence in the vaccines. Given the disproportionate impact of COVID-19 on racial minorities, much of the discussion centered around “vaccine hesitancy” in these communities. Reasons such as historical mistrust, belief in conspiracy theories, and misinformation emerged as the leading explanations.2 Campaigns and educational programs targeting Black Americans were quickly developed to counter this widely distributed narrative.

Vaccine uptake also became politicized, which created additional challenges. As schools and businesses reopened, the voices of those opposing pandemic mitigation mandates such as masking and vaccination were highlighted by media outlets. And though a large movement of individuals who had opted against vaccines existed well before the pandemic, with few exceptions, that number had never been great enough to impact public health to this extent.3 This primarily nonminority group of unvaccinated individuals also morphed into another monolithic identity: the “anti-vaxxer.”

The lion’s share of discussions around vaccine uptake centered on these two groups: the “vaccine hesitant” minority and the “anti-vaxxer.” The perspectives and frustration around these two stereotypical unvaccinated groups were underscored in journals and the lay press. But those working in communities and in direct care came into contact with countless COVID-19-positive patients who were unvaccinated and fell into neither of these categories. There was a large swath of vulnerable people who still had unanswered questions and mistrust in the medical system standing in their way. Awareness of health disparities among racial minorities is something that was discussed among providers, but it was something experienced and felt by patients daily in regard to so much more than just COVID-19.

With broader access to vaccines through retail, community-based, and clinical facilities, more patients who desired vaccination had the opportunity. After an initial rise in vaccine uptake, the numbers plateaued. But what remained was the repetitive messaging and sustained focus directed toward Black people and their “vaccine hesitancy.”

Grady Memorial Hospital, a public safety net hospital in Atlanta, serves a predominantly Black and uninsured patient population. We found that a “FAQ” approach with a narrow range of hypothetical ideas about unvaccinated minorities clashed with the reality of what we encountered in clinical environments and the community. While misinformation did appear to be prevalent, we appreciated that the context and level of conviction were heterogenous. We appreciated that each individual conversation could reveal something new to us about that unique patient and their personal concerns about vaccination. As time moved forward, it became clear that there was no playbook for any group, especially for historically disadvantaged communities. Importantly, it was recognized that attempts to anticipate what may be a person’s barrier to vaccination often worked to further erode trust. However, when we focused on creating a space for dialogue, we found we were able to move beyond information-sharing and instead were able to co-construct interpretations of information and co-create solutions that matched patients’ values and lived experiences.4 Through dialogue, we were better able to be transparent about our own experiences, which ultimately facilitated authentic conversations with patients.

In September 2021, we approached our hospital leadership with a patient-centered strategy aimed at providing our patients, staff, and visitors a psychologically safe place to discuss vaccine-related concerns without judgment. With their support, we set up a table in the busiest part of our hospital atrium between the information desk and vaccine-administration site. Beside it was a folding board sign with an image and these words:

“Still unsure about being vaccinated? Let’s talk about it.”

We aptly called the area the “No Judgment Zone.”

The No Judgment Zone is collaboratively staffed in 1- to 2-hour voluntary increments by physician faculty and resident physicians at Emory University School of Medicine and Morehouse School of Medicine. Our goal is to increase patient trust by honoring individual vaccine-related concerns without shame or ridicule. We also work to increase patient trust by being transparent around our own experiences with COVID-19; by sharing our own journeys, concerns, and challenges, we are better able to engage in meaningful dialogue. Also, recognizing the power of logistical barriers, in addition to answering questions, we offer physical assistance with check-in, forms, and escorts to our administration areas. The numbers of unique visits have varied from day to day, but the impact of each individual encounter cannot be overstated.

Here, we describe our approach to interactions at the No Judgment Zone. These are the instructions offered to our volunteers. Though we offer some explicit examples, each talking point is designed to open the door to a patient-centered individual dialogue. We believe that these strategies can be applied to clinical settings as well as any conversation surrounding vaccination with those who have not yet decided to be vaccinated.

THE GRADY “NO JUDGMENT ZONE” INTERACTION APPROACH

No Labels

Try to think of all who are not yet vaccinated as “on a spectrum of deliberation” about their decision—not “hesitant” or “anti-vaxxer.”

Step 1: Gratitude

  • “Thank you for stopping to talk to us today.”
  • “I appreciate you taking the time.”
  • “Before we start—I’m glad you’re here. Thanks.”

Step 2: Determine Where They Are

  • Has the person you’re speaking with been vaccinated yet?
  • If no, ask: “On a scale of 0 to 10—zero being “I will never get vaccinated under any circumstances” and 10 being ‘I will definitely get vaccinated’—what number would you give yourself?”
  • If the person is a firm zero: “Is there anything I might be able to share with you or tell you about that might move you away from that perspective?”
  • If the answer is NO: “It sounds like you’ve thought a lot about this and are no longer deliberating about whether you will be vaccinated. If you find yourself considering it, come back to talk with us, okay?” We are not here to debate or argue. We also need to avail ourselves to those who are open to changing their mind.
  • If they say anything other than zero, move to an open-ended question about #WhatsYourWhy.

Step 3: #WhatsYourWhy

  • “What would you say has been your main reason for not getting vaccinated yet?”
  • “Tell me what has stood in the way of you getting vaccinated.”
  • Remember: Assume nothing. It may have nothing to do with misinformation, fear, or perceived threat. It could be logistics or many other things. You will not know unless you ask.
  • Providers should feel encouraged to also share their why as well and the reasons they encouraged their parents/kids/loved ones to get vaccinated. Making it personal can help establish connection and be more compelling.

Step 4: Listen Completely

  • Give full eye contact. Slow all body movements. Use facilitative gestures to let the person know you are listening.
  • Do not plan what you wish to say next.
  • Limit reactions to misinformation. Shame and judgment can be subtle. Be mindful.
  • Repeat the concern back if you are not sure or want to confirm that you’ve heard correctly.
  • Ask questions for clarity if you aren’t sure.

Step 5: Affirm All Concerns and Find Common Ground

  • “I can only imagine how scary it must be to take a shot that you believe could cause you to not be able to have babies.”
  • “You aren’t alone. That’s a concern that many of my patients have had, too. May I share some information about that with you?”
  • “When I first heard about the vaccine, I worried it was too new, too. Can I share what I learned?”

Step 6: Provide Factual Information

  • Without excessive medical jargon, offer factual information aimed at each concern or question. Probe to be certain your patient understands through a teach-back or question.
  • If you are unsure about the answer to their question, admit that you don’t know. You can also ask a colleague or the attending with you. Another option is to call someone or say “Let’s pull this up together.” Then share your answer.
  • It is okay to acknowledge that the healthcare system has not and does not always do right by minority populations, especially Black people. Use that as a pivot to why these truths make vaccination that much more important
  • Have FAQ information sheets available. Confirm that the patient is comfortable with the information sheet by asking.

Step 7: Offer to Help Them Get Vaccinated Today

  • “Would you like me to help you get vaccinated today?”
  • “What can I do to assist you with getting vaccinated? Is today a good day?”
  • Escort those who agree to the registration area.
  • Affirm those plans to get vaccinated or those who feel closer to getting vaccinated after speaking with you.

Step 8: Gratitude

  • Close with gratitude and an affirmation.
  • “I’m so glad you took the time to talk with us today. You didn’t have to stop.”
  • “Feel free to come back to talk to us if you think of any more questions. I’m grateful that you stopped.”
  • We are planting seeds. Do not feel pressure to get a person to say yes. Our secret sauce is kindness, respect, and empathy.
  • We do not think of our unvaccinated community members as “hesitant.” We approach all as if they are on a spectrum of deliberation.

Step 9: Reflect

  • Understand the importance of your service and the potential impact each encounter has.
  • Recognize the unique lived experiences of individual patients and how this may impact their deliberation process. While there is urgency and we may feel frustrated, the ultimate goal is to engender trust through respectful interactions.
  • Pause for moments of quiet gratitude and self-check-ins.

Conclusion

Just as SARS-CoV2 spreads from one person to many, we recognize that information—factual and otherwise—has the potential to move quickly as well. It is important to realize that providing an opportunity for people to ask questions or receive clarification and confirmation in a safe space is critical. The No Judgement Zone, as the name indicates, offers this opportunity. The conversations that we have in this space are valuable to those who are still considering the vaccine as an option for themselves. The trust required for such conversations is less about the transmission of information and more about the social act of engaging in bidirectional dialogue. The foundation upon which trust is built is consistent trustworthy actions. One such action is respectful communication without shame or ridicule. Another is our willingness to be transparent about our own concerns, experiences, and journeys. Assumptions based upon single-story narratives of the unvaccinated—particularly those from historically marginalized groups—fracture an already fragile confidence in medical authorities.

While we understand that mitigating the ongoing spread of the virus and getting more people vaccinated will call for more than just individual conversations, we believe that respecting the unique perspectives of community members is an equally critical piece to moving forward. Throughout a healthcare worker’s typical day, we work to create personal moments of connection with patients among the immense bustle of other work that has to be done. Initiatives like this one have a focused intentionality behind creating space for patients to feel heard that is not only helpful for vaccine uptake and addressing mistrust, but can also be restorative for providers as well.

References

1. Manning KD. When grief and crises intersect: perspectives of a Black physician in the time of two pandemics. J Hosp Med. 2020;15(9):566-567. https://doi.org/10.12788/jhm.3481
2. Young S. Black vaccine hesitancy rooted in mistrust, doubts. WebMD. February 2, 2021. Accessed November 1, 2021. https://www.webmd.com/vaccines/covid-19-vaccine/news/20210202/black-vaccine-hesitancy-rooted-in-mistrust-doubts
3. Sanyaolu A, Okorie C, Marinkovic A, et al. Measles outbreak in unvaccinated and partially vaccinated children and adults in the United States and Canada (2018-2019): a narrative review of cases. Inquiry. 2019;56:46958019894098. https://doi.org/10.1177/0046958019894098
4. O’Brien BC. Do you see what I see? Reflections on the relationship between transparency and trust. Acad Med. 2019;94(6):757-759. https://doi.org/10.1097/ACM.0000000000002710

Article PDF
Author and Disclosure Information

1Department of Medicine, Emory University School of Medicine, Atlanta, Georgia; 2Department of Pediatrics, Morehouse School of Medicine, Atlanta, Georgia; 3Chief Health Equity Officer, Grady Health System, Atlanta, Georgia.

Disclosures
The authors reported no conflicts of interest.

Publications
Topics
Sections
Author and Disclosure Information

1Department of Medicine, Emory University School of Medicine, Atlanta, Georgia; 2Department of Pediatrics, Morehouse School of Medicine, Atlanta, Georgia; 3Chief Health Equity Officer, Grady Health System, Atlanta, Georgia.

Disclosures
The authors reported no conflicts of interest.

Author and Disclosure Information

1Department of Medicine, Emory University School of Medicine, Atlanta, Georgia; 2Department of Pediatrics, Morehouse School of Medicine, Atlanta, Georgia; 3Chief Health Equity Officer, Grady Health System, Atlanta, Georgia.

Disclosures
The authors reported no conflicts of interest.

Article PDF
Article PDF
Related Articles

The collective struggle felt by healthcare workers simultaneously learning about and caring for patients impacted by SARS-CoV2 infections throughout 2020 was physically and emotionally exhausting. The majority of us had never experienced a global pandemic. Beyond our work in the professional arena of ambulatory practices and hospitals, we also felt the soul-crushing impact of the pandemic in every other aspect of our lives. Preexisting health disparities were amplified by COVID-19. Some of the most affected communities also bore the weight of an additional tsunami of ongoing racial injustice.1 And as healthcare workers, we did our best to process and navigate it all while trying to avoid burnout—as well as being infected with COVID-19 ourselves. When the news of the highly effective vaccines against SARS-CoV2 receiving emergency use authorization broke late in 2020, it felt like a light at the end of a very dark tunnel.

In the weeks preceding wide availability of the vaccines, it became apparent that significant numbers of people lacked confidence in the vaccines. Given the disproportionate impact of COVID-19 on racial minorities, much of the discussion centered around “vaccine hesitancy” in these communities. Reasons such as historical mistrust, belief in conspiracy theories, and misinformation emerged as the leading explanations.2 Campaigns and educational programs targeting Black Americans were quickly developed to counter this widely distributed narrative.

Vaccine uptake also became politicized, which created additional challenges. As schools and businesses reopened, the voices of those opposing pandemic mitigation mandates such as masking and vaccination were highlighted by media outlets. And though a large movement of individuals who had opted against vaccines existed well before the pandemic, with few exceptions, that number had never been great enough to impact public health to this extent.3 This primarily nonminority group of unvaccinated individuals also morphed into another monolithic identity: the “anti-vaxxer.”

The lion’s share of discussions around vaccine uptake centered on these two groups: the “vaccine hesitant” minority and the “anti-vaxxer.” The perspectives and frustration around these two stereotypical unvaccinated groups were underscored in journals and the lay press. But those working in communities and in direct care came into contact with countless COVID-19-positive patients who were unvaccinated and fell into neither of these categories. There was a large swath of vulnerable people who still had unanswered questions and mistrust in the medical system standing in their way. Awareness of health disparities among racial minorities is something that was discussed among providers, but it was something experienced and felt by patients daily in regard to so much more than just COVID-19.

With broader access to vaccines through retail, community-based, and clinical facilities, more patients who desired vaccination had the opportunity. After an initial rise in vaccine uptake, the numbers plateaued. But what remained was the repetitive messaging and sustained focus directed toward Black people and their “vaccine hesitancy.”

Grady Memorial Hospital, a public safety net hospital in Atlanta, serves a predominantly Black and uninsured patient population. We found that a “FAQ” approach with a narrow range of hypothetical ideas about unvaccinated minorities clashed with the reality of what we encountered in clinical environments and the community. While misinformation did appear to be prevalent, we appreciated that the context and level of conviction were heterogenous. We appreciated that each individual conversation could reveal something new to us about that unique patient and their personal concerns about vaccination. As time moved forward, it became clear that there was no playbook for any group, especially for historically disadvantaged communities. Importantly, it was recognized that attempts to anticipate what may be a person’s barrier to vaccination often worked to further erode trust. However, when we focused on creating a space for dialogue, we found we were able to move beyond information-sharing and instead were able to co-construct interpretations of information and co-create solutions that matched patients’ values and lived experiences.4 Through dialogue, we were better able to be transparent about our own experiences, which ultimately facilitated authentic conversations with patients.

In September 2021, we approached our hospital leadership with a patient-centered strategy aimed at providing our patients, staff, and visitors a psychologically safe place to discuss vaccine-related concerns without judgment. With their support, we set up a table in the busiest part of our hospital atrium between the information desk and vaccine-administration site. Beside it was a folding board sign with an image and these words:

“Still unsure about being vaccinated? Let’s talk about it.”

We aptly called the area the “No Judgment Zone.”

The No Judgment Zone is collaboratively staffed in 1- to 2-hour voluntary increments by physician faculty and resident physicians at Emory University School of Medicine and Morehouse School of Medicine. Our goal is to increase patient trust by honoring individual vaccine-related concerns without shame or ridicule. We also work to increase patient trust by being transparent around our own experiences with COVID-19; by sharing our own journeys, concerns, and challenges, we are better able to engage in meaningful dialogue. Also, recognizing the power of logistical barriers, in addition to answering questions, we offer physical assistance with check-in, forms, and escorts to our administration areas. The numbers of unique visits have varied from day to day, but the impact of each individual encounter cannot be overstated.

Here, we describe our approach to interactions at the No Judgment Zone. These are the instructions offered to our volunteers. Though we offer some explicit examples, each talking point is designed to open the door to a patient-centered individual dialogue. We believe that these strategies can be applied to clinical settings as well as any conversation surrounding vaccination with those who have not yet decided to be vaccinated.

THE GRADY “NO JUDGMENT ZONE” INTERACTION APPROACH

No Labels

Try to think of all who are not yet vaccinated as “on a spectrum of deliberation” about their decision—not “hesitant” or “anti-vaxxer.”

Step 1: Gratitude

  • “Thank you for stopping to talk to us today.”
  • “I appreciate you taking the time.”
  • “Before we start—I’m glad you’re here. Thanks.”

Step 2: Determine Where They Are

  • Has the person you’re speaking with been vaccinated yet?
  • If no, ask: “On a scale of 0 to 10—zero being “I will never get vaccinated under any circumstances” and 10 being ‘I will definitely get vaccinated’—what number would you give yourself?”
  • If the person is a firm zero: “Is there anything I might be able to share with you or tell you about that might move you away from that perspective?”
  • If the answer is NO: “It sounds like you’ve thought a lot about this and are no longer deliberating about whether you will be vaccinated. If you find yourself considering it, come back to talk with us, okay?” We are not here to debate or argue. We also need to avail ourselves to those who are open to changing their mind.
  • If they say anything other than zero, move to an open-ended question about #WhatsYourWhy.

Step 3: #WhatsYourWhy

  • “What would you say has been your main reason for not getting vaccinated yet?”
  • “Tell me what has stood in the way of you getting vaccinated.”
  • Remember: Assume nothing. It may have nothing to do with misinformation, fear, or perceived threat. It could be logistics or many other things. You will not know unless you ask.
  • Providers should feel encouraged to also share their why as well and the reasons they encouraged their parents/kids/loved ones to get vaccinated. Making it personal can help establish connection and be more compelling.

Step 4: Listen Completely

  • Give full eye contact. Slow all body movements. Use facilitative gestures to let the person know you are listening.
  • Do not plan what you wish to say next.
  • Limit reactions to misinformation. Shame and judgment can be subtle. Be mindful.
  • Repeat the concern back if you are not sure or want to confirm that you’ve heard correctly.
  • Ask questions for clarity if you aren’t sure.

Step 5: Affirm All Concerns and Find Common Ground

  • “I can only imagine how scary it must be to take a shot that you believe could cause you to not be able to have babies.”
  • “You aren’t alone. That’s a concern that many of my patients have had, too. May I share some information about that with you?”
  • “When I first heard about the vaccine, I worried it was too new, too. Can I share what I learned?”

Step 6: Provide Factual Information

  • Without excessive medical jargon, offer factual information aimed at each concern or question. Probe to be certain your patient understands through a teach-back or question.
  • If you are unsure about the answer to their question, admit that you don’t know. You can also ask a colleague or the attending with you. Another option is to call someone or say “Let’s pull this up together.” Then share your answer.
  • It is okay to acknowledge that the healthcare system has not and does not always do right by minority populations, especially Black people. Use that as a pivot to why these truths make vaccination that much more important
  • Have FAQ information sheets available. Confirm that the patient is comfortable with the information sheet by asking.

Step 7: Offer to Help Them Get Vaccinated Today

  • “Would you like me to help you get vaccinated today?”
  • “What can I do to assist you with getting vaccinated? Is today a good day?”
  • Escort those who agree to the registration area.
  • Affirm those plans to get vaccinated or those who feel closer to getting vaccinated after speaking with you.

Step 8: Gratitude

  • Close with gratitude and an affirmation.
  • “I’m so glad you took the time to talk with us today. You didn’t have to stop.”
  • “Feel free to come back to talk to us if you think of any more questions. I’m grateful that you stopped.”
  • We are planting seeds. Do not feel pressure to get a person to say yes. Our secret sauce is kindness, respect, and empathy.
  • We do not think of our unvaccinated community members as “hesitant.” We approach all as if they are on a spectrum of deliberation.

Step 9: Reflect

  • Understand the importance of your service and the potential impact each encounter has.
  • Recognize the unique lived experiences of individual patients and how this may impact their deliberation process. While there is urgency and we may feel frustrated, the ultimate goal is to engender trust through respectful interactions.
  • Pause for moments of quiet gratitude and self-check-ins.

Conclusion

Just as SARS-CoV2 spreads from one person to many, we recognize that information—factual and otherwise—has the potential to move quickly as well. It is important to realize that providing an opportunity for people to ask questions or receive clarification and confirmation in a safe space is critical. The No Judgement Zone, as the name indicates, offers this opportunity. The conversations that we have in this space are valuable to those who are still considering the vaccine as an option for themselves. The trust required for such conversations is less about the transmission of information and more about the social act of engaging in bidirectional dialogue. The foundation upon which trust is built is consistent trustworthy actions. One such action is respectful communication without shame or ridicule. Another is our willingness to be transparent about our own concerns, experiences, and journeys. Assumptions based upon single-story narratives of the unvaccinated—particularly those from historically marginalized groups—fracture an already fragile confidence in medical authorities.

While we understand that mitigating the ongoing spread of the virus and getting more people vaccinated will call for more than just individual conversations, we believe that respecting the unique perspectives of community members is an equally critical piece to moving forward. Throughout a healthcare worker’s typical day, we work to create personal moments of connection with patients among the immense bustle of other work that has to be done. Initiatives like this one have a focused intentionality behind creating space for patients to feel heard that is not only helpful for vaccine uptake and addressing mistrust, but can also be restorative for providers as well.

The collective struggle felt by healthcare workers simultaneously learning about and caring for patients impacted by SARS-CoV2 infections throughout 2020 was physically and emotionally exhausting. The majority of us had never experienced a global pandemic. Beyond our work in the professional arena of ambulatory practices and hospitals, we also felt the soul-crushing impact of the pandemic in every other aspect of our lives. Preexisting health disparities were amplified by COVID-19. Some of the most affected communities also bore the weight of an additional tsunami of ongoing racial injustice.1 And as healthcare workers, we did our best to process and navigate it all while trying to avoid burnout—as well as being infected with COVID-19 ourselves. When the news of the highly effective vaccines against SARS-CoV2 receiving emergency use authorization broke late in 2020, it felt like a light at the end of a very dark tunnel.

In the weeks preceding wide availability of the vaccines, it became apparent that significant numbers of people lacked confidence in the vaccines. Given the disproportionate impact of COVID-19 on racial minorities, much of the discussion centered around “vaccine hesitancy” in these communities. Reasons such as historical mistrust, belief in conspiracy theories, and misinformation emerged as the leading explanations.2 Campaigns and educational programs targeting Black Americans were quickly developed to counter this widely distributed narrative.

Vaccine uptake also became politicized, which created additional challenges. As schools and businesses reopened, the voices of those opposing pandemic mitigation mandates such as masking and vaccination were highlighted by media outlets. And though a large movement of individuals who had opted against vaccines existed well before the pandemic, with few exceptions, that number had never been great enough to impact public health to this extent.3 This primarily nonminority group of unvaccinated individuals also morphed into another monolithic identity: the “anti-vaxxer.”

The lion’s share of discussions around vaccine uptake centered on these two groups: the “vaccine hesitant” minority and the “anti-vaxxer.” The perspectives and frustration around these two stereotypical unvaccinated groups were underscored in journals and the lay press. But those working in communities and in direct care came into contact with countless COVID-19-positive patients who were unvaccinated and fell into neither of these categories. There was a large swath of vulnerable people who still had unanswered questions and mistrust in the medical system standing in their way. Awareness of health disparities among racial minorities is something that was discussed among providers, but it was something experienced and felt by patients daily in regard to so much more than just COVID-19.

With broader access to vaccines through retail, community-based, and clinical facilities, more patients who desired vaccination had the opportunity. After an initial rise in vaccine uptake, the numbers plateaued. But what remained was the repetitive messaging and sustained focus directed toward Black people and their “vaccine hesitancy.”

Grady Memorial Hospital, a public safety net hospital in Atlanta, serves a predominantly Black and uninsured patient population. We found that a “FAQ” approach with a narrow range of hypothetical ideas about unvaccinated minorities clashed with the reality of what we encountered in clinical environments and the community. While misinformation did appear to be prevalent, we appreciated that the context and level of conviction were heterogenous. We appreciated that each individual conversation could reveal something new to us about that unique patient and their personal concerns about vaccination. As time moved forward, it became clear that there was no playbook for any group, especially for historically disadvantaged communities. Importantly, it was recognized that attempts to anticipate what may be a person’s barrier to vaccination often worked to further erode trust. However, when we focused on creating a space for dialogue, we found we were able to move beyond information-sharing and instead were able to co-construct interpretations of information and co-create solutions that matched patients’ values and lived experiences.4 Through dialogue, we were better able to be transparent about our own experiences, which ultimately facilitated authentic conversations with patients.

In September 2021, we approached our hospital leadership with a patient-centered strategy aimed at providing our patients, staff, and visitors a psychologically safe place to discuss vaccine-related concerns without judgment. With their support, we set up a table in the busiest part of our hospital atrium between the information desk and vaccine-administration site. Beside it was a folding board sign with an image and these words:

“Still unsure about being vaccinated? Let’s talk about it.”

We aptly called the area the “No Judgment Zone.”

The No Judgment Zone is collaboratively staffed in 1- to 2-hour voluntary increments by physician faculty and resident physicians at Emory University School of Medicine and Morehouse School of Medicine. Our goal is to increase patient trust by honoring individual vaccine-related concerns without shame or ridicule. We also work to increase patient trust by being transparent around our own experiences with COVID-19; by sharing our own journeys, concerns, and challenges, we are better able to engage in meaningful dialogue. Also, recognizing the power of logistical barriers, in addition to answering questions, we offer physical assistance with check-in, forms, and escorts to our administration areas. The numbers of unique visits have varied from day to day, but the impact of each individual encounter cannot be overstated.

Here, we describe our approach to interactions at the No Judgment Zone. These are the instructions offered to our volunteers. Though we offer some explicit examples, each talking point is designed to open the door to a patient-centered individual dialogue. We believe that these strategies can be applied to clinical settings as well as any conversation surrounding vaccination with those who have not yet decided to be vaccinated.

THE GRADY “NO JUDGMENT ZONE” INTERACTION APPROACH

No Labels

Try to think of all who are not yet vaccinated as “on a spectrum of deliberation” about their decision—not “hesitant” or “anti-vaxxer.”

Step 1: Gratitude

  • “Thank you for stopping to talk to us today.”
  • “I appreciate you taking the time.”
  • “Before we start—I’m glad you’re here. Thanks.”

Step 2: Determine Where They Are

  • Has the person you’re speaking with been vaccinated yet?
  • If no, ask: “On a scale of 0 to 10—zero being “I will never get vaccinated under any circumstances” and 10 being ‘I will definitely get vaccinated’—what number would you give yourself?”
  • If the person is a firm zero: “Is there anything I might be able to share with you or tell you about that might move you away from that perspective?”
  • If the answer is NO: “It sounds like you’ve thought a lot about this and are no longer deliberating about whether you will be vaccinated. If you find yourself considering it, come back to talk with us, okay?” We are not here to debate or argue. We also need to avail ourselves to those who are open to changing their mind.
  • If they say anything other than zero, move to an open-ended question about #WhatsYourWhy.

Step 3: #WhatsYourWhy

  • “What would you say has been your main reason for not getting vaccinated yet?”
  • “Tell me what has stood in the way of you getting vaccinated.”
  • Remember: Assume nothing. It may have nothing to do with misinformation, fear, or perceived threat. It could be logistics or many other things. You will not know unless you ask.
  • Providers should feel encouraged to also share their why as well and the reasons they encouraged their parents/kids/loved ones to get vaccinated. Making it personal can help establish connection and be more compelling.

Step 4: Listen Completely

  • Give full eye contact. Slow all body movements. Use facilitative gestures to let the person know you are listening.
  • Do not plan what you wish to say next.
  • Limit reactions to misinformation. Shame and judgment can be subtle. Be mindful.
  • Repeat the concern back if you are not sure or want to confirm that you’ve heard correctly.
  • Ask questions for clarity if you aren’t sure.

Step 5: Affirm All Concerns and Find Common Ground

  • “I can only imagine how scary it must be to take a shot that you believe could cause you to not be able to have babies.”
  • “You aren’t alone. That’s a concern that many of my patients have had, too. May I share some information about that with you?”
  • “When I first heard about the vaccine, I worried it was too new, too. Can I share what I learned?”

Step 6: Provide Factual Information

  • Without excessive medical jargon, offer factual information aimed at each concern or question. Probe to be certain your patient understands through a teach-back or question.
  • If you are unsure about the answer to their question, admit that you don’t know. You can also ask a colleague or the attending with you. Another option is to call someone or say “Let’s pull this up together.” Then share your answer.
  • It is okay to acknowledge that the healthcare system has not and does not always do right by minority populations, especially Black people. Use that as a pivot to why these truths make vaccination that much more important
  • Have FAQ information sheets available. Confirm that the patient is comfortable with the information sheet by asking.

Step 7: Offer to Help Them Get Vaccinated Today

  • “Would you like me to help you get vaccinated today?”
  • “What can I do to assist you with getting vaccinated? Is today a good day?”
  • Escort those who agree to the registration area.
  • Affirm those plans to get vaccinated or those who feel closer to getting vaccinated after speaking with you.

Step 8: Gratitude

  • Close with gratitude and an affirmation.
  • “I’m so glad you took the time to talk with us today. You didn’t have to stop.”
  • “Feel free to come back to talk to us if you think of any more questions. I’m grateful that you stopped.”
  • We are planting seeds. Do not feel pressure to get a person to say yes. Our secret sauce is kindness, respect, and empathy.
  • We do not think of our unvaccinated community members as “hesitant.” We approach all as if they are on a spectrum of deliberation.

Step 9: Reflect

  • Understand the importance of your service and the potential impact each encounter has.
  • Recognize the unique lived experiences of individual patients and how this may impact their deliberation process. While there is urgency and we may feel frustrated, the ultimate goal is to engender trust through respectful interactions.
  • Pause for moments of quiet gratitude and self-check-ins.

Conclusion

Just as SARS-CoV2 spreads from one person to many, we recognize that information—factual and otherwise—has the potential to move quickly as well. It is important to realize that providing an opportunity for people to ask questions or receive clarification and confirmation in a safe space is critical. The No Judgement Zone, as the name indicates, offers this opportunity. The conversations that we have in this space are valuable to those who are still considering the vaccine as an option for themselves. The trust required for such conversations is less about the transmission of information and more about the social act of engaging in bidirectional dialogue. The foundation upon which trust is built is consistent trustworthy actions. One such action is respectful communication without shame or ridicule. Another is our willingness to be transparent about our own concerns, experiences, and journeys. Assumptions based upon single-story narratives of the unvaccinated—particularly those from historically marginalized groups—fracture an already fragile confidence in medical authorities.

While we understand that mitigating the ongoing spread of the virus and getting more people vaccinated will call for more than just individual conversations, we believe that respecting the unique perspectives of community members is an equally critical piece to moving forward. Throughout a healthcare worker’s typical day, we work to create personal moments of connection with patients among the immense bustle of other work that has to be done. Initiatives like this one have a focused intentionality behind creating space for patients to feel heard that is not only helpful for vaccine uptake and addressing mistrust, but can also be restorative for providers as well.

References

1. Manning KD. When grief and crises intersect: perspectives of a Black physician in the time of two pandemics. J Hosp Med. 2020;15(9):566-567. https://doi.org/10.12788/jhm.3481
2. Young S. Black vaccine hesitancy rooted in mistrust, doubts. WebMD. February 2, 2021. Accessed November 1, 2021. https://www.webmd.com/vaccines/covid-19-vaccine/news/20210202/black-vaccine-hesitancy-rooted-in-mistrust-doubts
3. Sanyaolu A, Okorie C, Marinkovic A, et al. Measles outbreak in unvaccinated and partially vaccinated children and adults in the United States and Canada (2018-2019): a narrative review of cases. Inquiry. 2019;56:46958019894098. https://doi.org/10.1177/0046958019894098
4. O’Brien BC. Do you see what I see? Reflections on the relationship between transparency and trust. Acad Med. 2019;94(6):757-759. https://doi.org/10.1097/ACM.0000000000002710

References

1. Manning KD. When grief and crises intersect: perspectives of a Black physician in the time of two pandemics. J Hosp Med. 2020;15(9):566-567. https://doi.org/10.12788/jhm.3481
2. Young S. Black vaccine hesitancy rooted in mistrust, doubts. WebMD. February 2, 2021. Accessed November 1, 2021. https://www.webmd.com/vaccines/covid-19-vaccine/news/20210202/black-vaccine-hesitancy-rooted-in-mistrust-doubts
3. Sanyaolu A, Okorie C, Marinkovic A, et al. Measles outbreak in unvaccinated and partially vaccinated children and adults in the United States and Canada (2018-2019): a narrative review of cases. Inquiry. 2019;56:46958019894098. https://doi.org/10.1177/0046958019894098
4. O’Brien BC. Do you see what I see? Reflections on the relationship between transparency and trust. Acad Med. 2019;94(6):757-759. https://doi.org/10.1097/ACM.0000000000002710

Publications
Publications
Topics
Article Type
Display Headline
The No Judgment Zone: Building Trust Through Trustworthiness
Display Headline
The No Judgment Zone: Building Trust Through Trustworthiness
Sections
Article Source

© 2021 Society of Hospital Medicine

Citation Override
J Hosp Med. Published Online Only November 8, 2021.
Disallow All Ads
Correspondence Location
Kimberly D Manning, MD; Email: kdmanni@emory.edu; Twitter: @gradydoctor.
Content Gating
Open Access (article Unlocked/Open Access)
Alternative CME
Disqus Comments
Default
Use ProPublica
Hide sidebar & use full width
render the right sidebar.
Conference Recap Checkbox
Not Conference Recap
Clinical Edge
Display the Slideshow in this Article
Gating Strategy
No Gating
Medscape Article
Display survey writer
Reuters content
Disable Inline Native ads
WebMD Article
Article PDF Media
Image
Teambase ID
18001EDF.SIG
Disable zoom
Off

Improving Healthcare Value: Reducing Overuse in Hospital Pediatrics

Article Type
Changed
Wed, 10/20/2021 - 01:15
Display Headline
Improving Healthcare Value: Reducing Overuse in Hospital Pediatrics

Most hospital pediatricians can recall cases where an abnormal result in one unnecessary test led to a cascade of multiple further unnecessary treatments, procedures, and tests. These cases are well described in the literature and written off as a side effect of delivering high-quality, comprehensive pediatric care.1 Unfortunately, however, these frequent events are not without consequence and can cause significant harm to patients, as well as stress and fear for parents and families, and indirectly waste valuable resources.

As we look forward to recovering from the COVID-19 pandemic, there are calls to prioritize high-value and more equitable care in the postpandemic world.2 Choosing Wisely is a global movement comprised of clinician-led campaigns that partner with national specialty societies to develop lists of evidence-based recommendations of tests, treatments, and procedures that offer no added clinical value and may cause harm.3

In pediatrics, there is a growing recognition and published literature on the harms of overdiagnosis and unnecessary care in children.4-6 Choosing Wisely recommendations are being used as a resource to drive healthcare prioritization and ensure low-value care is avoided so that greater focus can be placed on areas of need exacerbated by the pandemic. Using a Choosing Wisely perspective can drive quality and help inform a shift in practice, creating a roadmap for reducing testing or treatment cascades that harm patients and waste resources as we move toward the goal of high-value pediatric care. However, adoption of Choosing Wisely recommendations in pediatrics has been slow. For example, the pediatric working group of the Society of Hospital Medicine released a Choosing Wisely® recommendation in 2013 against the use of continuous pulse oximetry monitoring in children with acute respiratory illness who are not on supplementary oxygen.7 Data from a cross-sectional study across 56 hospitals 6 years later found significant variation in this practice for infants hospitalized with bronchiolitis and not receiving supplemental oxygen; 46% were continuously monitored with pulse oximetry (range, 2%-92%).8

WHY HAS CHOOSING WISELY LAGGED IN PEDIATRICS?

Traditionally, attention in children’s healthcare has focused on underuse (eg, immunizations or mental health) rather than overuse. Further, the weakness of the evidence base, with very few randomized controlled trials in children, limits our ability to provide sufficient confidence in the evidence supporting some of our recommendations.9

Second, there is also tremendous anxiety for both parents and frontline clinicians around diagnostic uncertainty of any kind when it comes to children. We endeavour to reassure ourselves and patients’ families by leaving no stone unturned. This approach can lead to unnecessary care, including false-positive test results, “incidentalomas,” and adverse effects from unnecessary medications. Despite the best intentions of assuaging caregivers’ anxiety, overuse of invasive and uncomfortable tests can have the opposite effect of increasing stress and trauma for both children and parents.

Third, there is compelling evidence that practice habits, once established, are difficult to break.10 Particularly in the high-stakes practice of hospital pediatric medicine, where we are conditioned to expect the worst and anticipate the unexpected. This “do everything to everyone” approach, however, can lead to significant harms for pediatric patients. For example, the exposure to ionizing radiation through unnecessary computed tomography (CT) scans can increase a child’s lifetime cancer risk.11

The perpetuation of unnecessary care needs to change in pediatrics, especially for the most vulnerable young patients seeking hospital care. Implementation is a necessary next step to introduce recommendations into practice, and the Choosing Wisely efforts of the Hospital for Sick Children in Toronto, Canada, can offer insights into opportunities to embed this approach across similar quaternary care teaching hospitals, as well as general hospitals and the systems they support.

STEPS TO IMPLEMENTING CHOOSING WISELY HOSPITAL-WIDE

Creating Lists of Recommendations Aligned With Quality Metrics

The Hospital for Sick Children developed a hospital-specific Choosing Wisely list in 2016 to address a gap in existing Choosing Wisely Canada campaign recommendations related to pediatric hospitals.12 Choosing Wisely Canada was initially focused on adult medicine, and a list of recommendations developed by the Canadian Paediatric Society relates mostly to overuse in pediatric outpatient settings and is not applicable to hospital-based practice.13 The Society of Hospital Medicine-Pediatric Hospital Medicine Choosing Wisely® list predominantly pertained to unnecessary care of infants with bronchiolitis (eg, not to order chest radiographs in uncomplicated asthma and bronchiolitis). We had measured our compliance with this recommendation and found it was already well below the achievable benchmark of care in the United States,14 so we preferred to create a list that would resonate with our clinicians. Since the original list was created at the Hospital for Sick Children,12 we have developed two subsequent lists of recommendations, which were released in 2018 and 2021 (Table).

friedman0306-1020e-t1.jpg

The approach to list development used by staff pediatricians and trainees, with input from hospital staff and family advisors, has been described elsewhere.12 The goal was to self-identify five local practices that we felt would help us reduce unnecessary care. This list served as the foundation of an organization-wide quality initiative driven by a steering committee that consisted of the clinician champions as well as representation from various groups at the hospital, including decision support, information services, the family advisory committee, and public affairs.

Each recommendation needed to be evidence-based and measurable, have a clinician champion to implement the recommendation, and have the potential to improve the quality and safety of the care we provided. “Balancing” measures needed to be carefully monitored to ensure that no diagnoses were being missed or negative effects resulted from decreasing these interventions. In order for a recommendation to be considered, a subgroup of the pediatric department’s clinical advisory committee reviewed the references provided to ensure that what was being suggested was based on published evidence and part of current national guidelines. The clinician champion needed to agree to lead the implementation project, and specific outcomes, including appropriate balancing measures, needed to be identified a priori, in addition to an appropriate mechanism to collect the data. Hospital executive leaders were supportive of the initiative and facilitated access to “in-kind” hospital resources as required, although no financial budget was provided. After some early success, the Department of Paediatrics provided part-time project management support to help coordinate the growth and administration of the initiative.

Measuring and Supporting Practice Change

The main implementation principles included targeted education/awareness, transparent measurement with audit/feedback, and, most importantly, embedding changes in the ordering process, essentially making it easy for frontline clinicians to do the right thing (and trickier to do the “wrong” thing). Audit and feedback have been used at both the individual provider level (eg, respiratory viral testing–ordering practices) and the divisional level (eg, ordering of postoperative antibiotics). These quality improvement initiatives have had a compelling impact. Scorecards have been developed and results shared internally using local divisional as well as hospital-wide tools, varying from staff meetings to screensavers across hospital computers and television screens and the hospital intranet. Evaluation is ongoing, but many of the initial results have been encouraging.15-17

For example, the 2016 list includes recommendations related to emergency department (ED) test ordering. Implementation efforts to address unnecessary nasopharyngeal swabs for viral testing in bronchiolitis reduced this practice by 80%,15 and there has been a 50% reduction in ankle X-rays in children with acute ankle injuries who meet criteria for a low-risk examination.16 The 2016 list also included a recommendation related to inappropriate intravenous immunoglobulin (IVIG) use in children with typical acute immune thrombocytopenic purpura (ITP), and a targeted quality improvement initiative reduced inappropriate IVIG use by 50%, with no detectable increase in bleeding complications or readmission to hospital.17 These results have been sustained over a period of 3 or more years. Examples from the 2018 list include a 40% decrease in inappropriate urinary tract infection diagnosis and treatment in the ED and a four-fold decrease in the CT abdomen/pelvis imaging rate for low-risk trauma.18

The steering committee meets every 2 months and includes all of the clinician champions as well as representatives from strategic hospital resources and two family advisors (NGS). These meetings are chaired by the Associate Pediatrician-in-Chief (JNF) and the project manager. The progress of the active projects is discussed, and the experience of the group is used to problem-solve, plan ahead, and encourage academic presentation and publication of the various projects. Patient partnership and participation in committees has ensured that improvements to patient experience, satisfaction, and education are considered in the outcomes of implementation. Moreover, it has safeguarded that this effort is not misperceived as limiting care and remains focused on advancing quality, safety, and the patient experience.

SOME LESSONS LEARNED

While most projects have surpassed expectations, not all have proceeded as anticipated. The biggest challenge is finding a reliable and practical source for data collection. For example, at the time of initiation of the voiding cystourethrography (VCUG) recommendation, practice had presumably changed over the recent years, and compliance already exceeded the goal, illustrating the importance of current accurate data. The oxygen saturation–monitoring recommendation highlighted the challenge presented by data collection that requires manual audits; the inability to find staff to do this regularly significantly hampered this project. The critical role of the clinician champion was highlighted in a few projects when a lead was absent for a prolonged period of time (eg, due to a parental leave or change in job), with no willing replacement. There does seem to be a strong correlation between the commitment and passion of the clinician lead and the success of the project. We have incorporated the lessons learned into the development and rollout of the 2018 and 2021 lists.

SPREAD AND SCALE

The challenge is to scale up these successes to impact and change practice across the hospital pediatrics community. After 5 years, awareness of and engagement with this process are still not uniform across our hospital campus. Nevertheless, anecdotally, at the Hospital for Sick Children, there is a shift in culture where clinicians have processed the imperative to reduce overuse and unnecessary tests and treatments, with phrases such as “this is not very Choosing Wisely” entering the vernacular. It is becoming part of the culture. Second, the new generation of medical school trainees and residents has displayed a tremendous appetite and passion for stewardship and a sense that practice can change from the ground up. The SickKids Choosing Wisely efforts have been a hub for resident-led quality improvement projects and leadership for implementation of recommendations.19 As we continue to engage all providers at our hospital, we are also reaching out to the other community hospitals in our region, and all children’s hospitals in Canada, to share the principles and lessons learned from our program through a national community of practice.

CONCLUSION

Practicing pediatric medicine in a well-resourced hospital setting should not drive us to overuse in practice “just because we can.” The harms of this approach to our patients and health systems, coupled with the pressures of the pandemic, are compelling reasons to be responsible stewards. There are opportunities to reshape and rethink practice patterns and habits.20 Overuse and overdiagnosis harm our patients and families physically and emotionally and indirectly waste resources urgently needed for investment upstream. Providing safe, quality, high-value care to our young patients requires constant critical thinking. The time is here to advance Choosing Wisely into pediatric hospital practice.

References

1. Elliott DK, Rose SR, Ronan JC. Changing the culture around cultures. Hosp Pediatr. 2014;4(6):405-407. https://doi.org/10.1542/hpeds.2014-0064
2. Gupta R, Simpson LA, Morgan DJ. Prioritizing high-value, equitable care after the COVID-19 shutdown: an opportunity for a healthcare renaissance. J Hosp Med. 2021;16(2):114-116. https://doi.org/10.12788/jhm.3526
3. Born K, Kool T, Levinson W. Reducing overuse in healthcare: advancing Choosing Wisely. BMJ. 2019;367:l6317. https://doi.org/10.1136/bmj.l6317
4. Coon ER, Young PC, Quinonez RA, Morgan DJ, Dhruva SS, Schroeder AR. Update on pediatric overuse. Pediatrics. 2017;139(2):e20162797. https://doi.org/10.1542/peds.2016-2797
5. Coon ER, Quinonez RA, Moyer VA, Schroeder AR. Overdiagnosis: how our compulsion for diagnosis may be harming children. Pediatrics. 2014;134(5):1013-1023. https://doi.org/10.1542/peds.2014-1778
6. Wolf ER, Krist AH, Schroeder AR. Deimplementation in pediatrics: past, present, and future. JAMA Pediatr. 2021;175(3):230-232. https://doi.org/10.1001/jamapediatrics.2020.4681
7. Quinonez RA, Garber MD, Schroeder AR, et al. Choosing Wisely in pediatric hospital medicine: five opportunities for improved healthcare value. J Hosp Med. 2013;8(9):479-485. https://doi.org/10.1002/jhm.2064
8. Bonafide CP, Xiao R, Brady PW, et al. Prevalence of continuous pulse oximetry monitoring in hospitalized children with bronchiolitis not requiring supplemental oxygen. JAMA. 2020;323(15):1467-1477. https://doi.org/10.1001/jama.2020.2998
9. Ralston SL, Schroeder AR. Why is it so hard to talk about overuse in pediatrics and why it matters. JAMA Pediatr. 2017;17(10):931-932. https://doi.org/10.1001/jamapediatrics.2017.2239
10. Stammen LA, Stalmeijer RE, Paternotte E, et al. Training physicians to provide high-value, cost-conscious care: a systematic review. JAMA. 2015;314(22):2384-2400. https://doi.org/10.1001/jama.2015.16353
11. Mathews JD, Forsythe AV, Brady Z, et al. Cancer risk in 680,000 people exposed to computed tomography scans in childhood or adolescence: data linkage study of 11 million Australians. BMJ. 2013;346:f2360. https://doi.org/10.1136/bmj.f2360
12. Friedman JN. Saying yes to the less: making it easier to choose wisely [editorial]. J Pediatr. 2017;145:4-5. https://doi.org/10.1016/j.jpeds.2017.01.062
13. Canadian Paediatric Society. Five things physicians and patients should question. Choosing Wisely Canada. Updated July 2019. Accessed June 17, 2021. https://choosingwiselycanada.org/wp-content/uploads/2020/07/Paediatrics_EN.pdf
14. Parikh K, Hall M, Montalbano A, et al. Establishing benchmarks for the hospitalized care of children with asthma, bronciolitis, and pneumonia. Pediatrics. 2014;134(3):555-562. https://doi.org/10.1542/peds.2014-1052
15. Ostrow O, Richardson S, Savlov D, Friedman JN. Reducing unnecessary respiratory viral testing to promote high value care. Pediatrics. In press.
16. Al-Sani F, Ben-Yakov M, Harvey G, et al. P016: Low risk ankle rule, high reward—a quality improvement initiative to reduce ankle x-rays in the pediatric emergency department [poster]. CJEM. 2017;19(S1):S83. https://doi.org/10.1017/cem.2017.218
17. Beck CE, Carcao M, Cada M, Porter S, Blanchette VS, Parkin PC. A quality improvement bundle to improve informed choice for children with typical, newly diagnosed immune thrombocytopenia. J Pediatr Hematol Oncol. 2018;40(8):e537-e543. https://doi.org/10.1097/MPH.0000000000001247
18. Beno S, Lenton-Brym T, Rosenfield D, McDowall D, Wales P, Principi T. Safe reduction of abdominal CT imaging in pediatric trauma patients: a quality-improvement initiative [abstract]. Can J Surg. 2019;62(3 Suppl 2):S29-S30.
19. Bal C, Tesch M, Blair G, Ostrow O, Premji L. Engaging medical trainees in resource stewardship through resident-led teaching sessions: a choosing wisely educational initiative. Can Med Educ J. 2021;12(1):e98-e100. https://doi.org/10.36834/cmej.70563
20. Berwick DM. Choices for the “new normal.” JAMA. 2020;323(21):2125-2126. https://doi.org/10.1001/jama.2020.6949

Article PDF
Author and Disclosure Information

1Hospital for Sick Children, Toronto, Ontario, Canada; 2Department of Paediatrics, University of Toronto, Toronto, Ontario, Canada; 3Institute for Health Policy, Management & Evaluation, University of Toronto, Toronto, Ontario, Canada.

Disclosures
The authors reported no conflicts of interest.

Publications
Topics
Sections
Author and Disclosure Information

1Hospital for Sick Children, Toronto, Ontario, Canada; 2Department of Paediatrics, University of Toronto, Toronto, Ontario, Canada; 3Institute for Health Policy, Management & Evaluation, University of Toronto, Toronto, Ontario, Canada.

Disclosures
The authors reported no conflicts of interest.

Author and Disclosure Information

1Hospital for Sick Children, Toronto, Ontario, Canada; 2Department of Paediatrics, University of Toronto, Toronto, Ontario, Canada; 3Institute for Health Policy, Management & Evaluation, University of Toronto, Toronto, Ontario, Canada.

Disclosures
The authors reported no conflicts of interest.

Article PDF
Article PDF
Related Articles

Most hospital pediatricians can recall cases where an abnormal result in one unnecessary test led to a cascade of multiple further unnecessary treatments, procedures, and tests. These cases are well described in the literature and written off as a side effect of delivering high-quality, comprehensive pediatric care.1 Unfortunately, however, these frequent events are not without consequence and can cause significant harm to patients, as well as stress and fear for parents and families, and indirectly waste valuable resources.

As we look forward to recovering from the COVID-19 pandemic, there are calls to prioritize high-value and more equitable care in the postpandemic world.2 Choosing Wisely is a global movement comprised of clinician-led campaigns that partner with national specialty societies to develop lists of evidence-based recommendations of tests, treatments, and procedures that offer no added clinical value and may cause harm.3

In pediatrics, there is a growing recognition and published literature on the harms of overdiagnosis and unnecessary care in children.4-6 Choosing Wisely recommendations are being used as a resource to drive healthcare prioritization and ensure low-value care is avoided so that greater focus can be placed on areas of need exacerbated by the pandemic. Using a Choosing Wisely perspective can drive quality and help inform a shift in practice, creating a roadmap for reducing testing or treatment cascades that harm patients and waste resources as we move toward the goal of high-value pediatric care. However, adoption of Choosing Wisely recommendations in pediatrics has been slow. For example, the pediatric working group of the Society of Hospital Medicine released a Choosing Wisely® recommendation in 2013 against the use of continuous pulse oximetry monitoring in children with acute respiratory illness who are not on supplementary oxygen.7 Data from a cross-sectional study across 56 hospitals 6 years later found significant variation in this practice for infants hospitalized with bronchiolitis and not receiving supplemental oxygen; 46% were continuously monitored with pulse oximetry (range, 2%-92%).8

WHY HAS CHOOSING WISELY LAGGED IN PEDIATRICS?

Traditionally, attention in children’s healthcare has focused on underuse (eg, immunizations or mental health) rather than overuse. Further, the weakness of the evidence base, with very few randomized controlled trials in children, limits our ability to provide sufficient confidence in the evidence supporting some of our recommendations.9

Second, there is also tremendous anxiety for both parents and frontline clinicians around diagnostic uncertainty of any kind when it comes to children. We endeavour to reassure ourselves and patients’ families by leaving no stone unturned. This approach can lead to unnecessary care, including false-positive test results, “incidentalomas,” and adverse effects from unnecessary medications. Despite the best intentions of assuaging caregivers’ anxiety, overuse of invasive and uncomfortable tests can have the opposite effect of increasing stress and trauma for both children and parents.

Third, there is compelling evidence that practice habits, once established, are difficult to break.10 Particularly in the high-stakes practice of hospital pediatric medicine, where we are conditioned to expect the worst and anticipate the unexpected. This “do everything to everyone” approach, however, can lead to significant harms for pediatric patients. For example, the exposure to ionizing radiation through unnecessary computed tomography (CT) scans can increase a child’s lifetime cancer risk.11

The perpetuation of unnecessary care needs to change in pediatrics, especially for the most vulnerable young patients seeking hospital care. Implementation is a necessary next step to introduce recommendations into practice, and the Choosing Wisely efforts of the Hospital for Sick Children in Toronto, Canada, can offer insights into opportunities to embed this approach across similar quaternary care teaching hospitals, as well as general hospitals and the systems they support.

STEPS TO IMPLEMENTING CHOOSING WISELY HOSPITAL-WIDE

Creating Lists of Recommendations Aligned With Quality Metrics

The Hospital for Sick Children developed a hospital-specific Choosing Wisely list in 2016 to address a gap in existing Choosing Wisely Canada campaign recommendations related to pediatric hospitals.12 Choosing Wisely Canada was initially focused on adult medicine, and a list of recommendations developed by the Canadian Paediatric Society relates mostly to overuse in pediatric outpatient settings and is not applicable to hospital-based practice.13 The Society of Hospital Medicine-Pediatric Hospital Medicine Choosing Wisely® list predominantly pertained to unnecessary care of infants with bronchiolitis (eg, not to order chest radiographs in uncomplicated asthma and bronchiolitis). We had measured our compliance with this recommendation and found it was already well below the achievable benchmark of care in the United States,14 so we preferred to create a list that would resonate with our clinicians. Since the original list was created at the Hospital for Sick Children,12 we have developed two subsequent lists of recommendations, which were released in 2018 and 2021 (Table).

friedman0306-1020e-t1.jpg

The approach to list development used by staff pediatricians and trainees, with input from hospital staff and family advisors, has been described elsewhere.12 The goal was to self-identify five local practices that we felt would help us reduce unnecessary care. This list served as the foundation of an organization-wide quality initiative driven by a steering committee that consisted of the clinician champions as well as representation from various groups at the hospital, including decision support, information services, the family advisory committee, and public affairs.

Each recommendation needed to be evidence-based and measurable, have a clinician champion to implement the recommendation, and have the potential to improve the quality and safety of the care we provided. “Balancing” measures needed to be carefully monitored to ensure that no diagnoses were being missed or negative effects resulted from decreasing these interventions. In order for a recommendation to be considered, a subgroup of the pediatric department’s clinical advisory committee reviewed the references provided to ensure that what was being suggested was based on published evidence and part of current national guidelines. The clinician champion needed to agree to lead the implementation project, and specific outcomes, including appropriate balancing measures, needed to be identified a priori, in addition to an appropriate mechanism to collect the data. Hospital executive leaders were supportive of the initiative and facilitated access to “in-kind” hospital resources as required, although no financial budget was provided. After some early success, the Department of Paediatrics provided part-time project management support to help coordinate the growth and administration of the initiative.

Measuring and Supporting Practice Change

The main implementation principles included targeted education/awareness, transparent measurement with audit/feedback, and, most importantly, embedding changes in the ordering process, essentially making it easy for frontline clinicians to do the right thing (and trickier to do the “wrong” thing). Audit and feedback have been used at both the individual provider level (eg, respiratory viral testing–ordering practices) and the divisional level (eg, ordering of postoperative antibiotics). These quality improvement initiatives have had a compelling impact. Scorecards have been developed and results shared internally using local divisional as well as hospital-wide tools, varying from staff meetings to screensavers across hospital computers and television screens and the hospital intranet. Evaluation is ongoing, but many of the initial results have been encouraging.15-17

For example, the 2016 list includes recommendations related to emergency department (ED) test ordering. Implementation efforts to address unnecessary nasopharyngeal swabs for viral testing in bronchiolitis reduced this practice by 80%,15 and there has been a 50% reduction in ankle X-rays in children with acute ankle injuries who meet criteria for a low-risk examination.16 The 2016 list also included a recommendation related to inappropriate intravenous immunoglobulin (IVIG) use in children with typical acute immune thrombocytopenic purpura (ITP), and a targeted quality improvement initiative reduced inappropriate IVIG use by 50%, with no detectable increase in bleeding complications or readmission to hospital.17 These results have been sustained over a period of 3 or more years. Examples from the 2018 list include a 40% decrease in inappropriate urinary tract infection diagnosis and treatment in the ED and a four-fold decrease in the CT abdomen/pelvis imaging rate for low-risk trauma.18

The steering committee meets every 2 months and includes all of the clinician champions as well as representatives from strategic hospital resources and two family advisors (NGS). These meetings are chaired by the Associate Pediatrician-in-Chief (JNF) and the project manager. The progress of the active projects is discussed, and the experience of the group is used to problem-solve, plan ahead, and encourage academic presentation and publication of the various projects. Patient partnership and participation in committees has ensured that improvements to patient experience, satisfaction, and education are considered in the outcomes of implementation. Moreover, it has safeguarded that this effort is not misperceived as limiting care and remains focused on advancing quality, safety, and the patient experience.

SOME LESSONS LEARNED

While most projects have surpassed expectations, not all have proceeded as anticipated. The biggest challenge is finding a reliable and practical source for data collection. For example, at the time of initiation of the voiding cystourethrography (VCUG) recommendation, practice had presumably changed over the recent years, and compliance already exceeded the goal, illustrating the importance of current accurate data. The oxygen saturation–monitoring recommendation highlighted the challenge presented by data collection that requires manual audits; the inability to find staff to do this regularly significantly hampered this project. The critical role of the clinician champion was highlighted in a few projects when a lead was absent for a prolonged period of time (eg, due to a parental leave or change in job), with no willing replacement. There does seem to be a strong correlation between the commitment and passion of the clinician lead and the success of the project. We have incorporated the lessons learned into the development and rollout of the 2018 and 2021 lists.

SPREAD AND SCALE

The challenge is to scale up these successes to impact and change practice across the hospital pediatrics community. After 5 years, awareness of and engagement with this process are still not uniform across our hospital campus. Nevertheless, anecdotally, at the Hospital for Sick Children, there is a shift in culture where clinicians have processed the imperative to reduce overuse and unnecessary tests and treatments, with phrases such as “this is not very Choosing Wisely” entering the vernacular. It is becoming part of the culture. Second, the new generation of medical school trainees and residents has displayed a tremendous appetite and passion for stewardship and a sense that practice can change from the ground up. The SickKids Choosing Wisely efforts have been a hub for resident-led quality improvement projects and leadership for implementation of recommendations.19 As we continue to engage all providers at our hospital, we are also reaching out to the other community hospitals in our region, and all children’s hospitals in Canada, to share the principles and lessons learned from our program through a national community of practice.

CONCLUSION

Practicing pediatric medicine in a well-resourced hospital setting should not drive us to overuse in practice “just because we can.” The harms of this approach to our patients and health systems, coupled with the pressures of the pandemic, are compelling reasons to be responsible stewards. There are opportunities to reshape and rethink practice patterns and habits.20 Overuse and overdiagnosis harm our patients and families physically and emotionally and indirectly waste resources urgently needed for investment upstream. Providing safe, quality, high-value care to our young patients requires constant critical thinking. The time is here to advance Choosing Wisely into pediatric hospital practice.

Most hospital pediatricians can recall cases where an abnormal result in one unnecessary test led to a cascade of multiple further unnecessary treatments, procedures, and tests. These cases are well described in the literature and written off as a side effect of delivering high-quality, comprehensive pediatric care.1 Unfortunately, however, these frequent events are not without consequence and can cause significant harm to patients, as well as stress and fear for parents and families, and indirectly waste valuable resources.

As we look forward to recovering from the COVID-19 pandemic, there are calls to prioritize high-value and more equitable care in the postpandemic world.2 Choosing Wisely is a global movement comprised of clinician-led campaigns that partner with national specialty societies to develop lists of evidence-based recommendations of tests, treatments, and procedures that offer no added clinical value and may cause harm.3

In pediatrics, there is a growing recognition and published literature on the harms of overdiagnosis and unnecessary care in children.4-6 Choosing Wisely recommendations are being used as a resource to drive healthcare prioritization and ensure low-value care is avoided so that greater focus can be placed on areas of need exacerbated by the pandemic. Using a Choosing Wisely perspective can drive quality and help inform a shift in practice, creating a roadmap for reducing testing or treatment cascades that harm patients and waste resources as we move toward the goal of high-value pediatric care. However, adoption of Choosing Wisely recommendations in pediatrics has been slow. For example, the pediatric working group of the Society of Hospital Medicine released a Choosing Wisely® recommendation in 2013 against the use of continuous pulse oximetry monitoring in children with acute respiratory illness who are not on supplementary oxygen.7 Data from a cross-sectional study across 56 hospitals 6 years later found significant variation in this practice for infants hospitalized with bronchiolitis and not receiving supplemental oxygen; 46% were continuously monitored with pulse oximetry (range, 2%-92%).8

WHY HAS CHOOSING WISELY LAGGED IN PEDIATRICS?

Traditionally, attention in children’s healthcare has focused on underuse (eg, immunizations or mental health) rather than overuse. Further, the weakness of the evidence base, with very few randomized controlled trials in children, limits our ability to provide sufficient confidence in the evidence supporting some of our recommendations.9

Second, there is also tremendous anxiety for both parents and frontline clinicians around diagnostic uncertainty of any kind when it comes to children. We endeavour to reassure ourselves and patients’ families by leaving no stone unturned. This approach can lead to unnecessary care, including false-positive test results, “incidentalomas,” and adverse effects from unnecessary medications. Despite the best intentions of assuaging caregivers’ anxiety, overuse of invasive and uncomfortable tests can have the opposite effect of increasing stress and trauma for both children and parents.

Third, there is compelling evidence that practice habits, once established, are difficult to break.10 Particularly in the high-stakes practice of hospital pediatric medicine, where we are conditioned to expect the worst and anticipate the unexpected. This “do everything to everyone” approach, however, can lead to significant harms for pediatric patients. For example, the exposure to ionizing radiation through unnecessary computed tomography (CT) scans can increase a child’s lifetime cancer risk.11

The perpetuation of unnecessary care needs to change in pediatrics, especially for the most vulnerable young patients seeking hospital care. Implementation is a necessary next step to introduce recommendations into practice, and the Choosing Wisely efforts of the Hospital for Sick Children in Toronto, Canada, can offer insights into opportunities to embed this approach across similar quaternary care teaching hospitals, as well as general hospitals and the systems they support.

STEPS TO IMPLEMENTING CHOOSING WISELY HOSPITAL-WIDE

Creating Lists of Recommendations Aligned With Quality Metrics

The Hospital for Sick Children developed a hospital-specific Choosing Wisely list in 2016 to address a gap in existing Choosing Wisely Canada campaign recommendations related to pediatric hospitals.12 Choosing Wisely Canada was initially focused on adult medicine, and a list of recommendations developed by the Canadian Paediatric Society relates mostly to overuse in pediatric outpatient settings and is not applicable to hospital-based practice.13 The Society of Hospital Medicine-Pediatric Hospital Medicine Choosing Wisely® list predominantly pertained to unnecessary care of infants with bronchiolitis (eg, not to order chest radiographs in uncomplicated asthma and bronchiolitis). We had measured our compliance with this recommendation and found it was already well below the achievable benchmark of care in the United States,14 so we preferred to create a list that would resonate with our clinicians. Since the original list was created at the Hospital for Sick Children,12 we have developed two subsequent lists of recommendations, which were released in 2018 and 2021 (Table).

friedman0306-1020e-t1.jpg

The approach to list development used by staff pediatricians and trainees, with input from hospital staff and family advisors, has been described elsewhere.12 The goal was to self-identify five local practices that we felt would help us reduce unnecessary care. This list served as the foundation of an organization-wide quality initiative driven by a steering committee that consisted of the clinician champions as well as representation from various groups at the hospital, including decision support, information services, the family advisory committee, and public affairs.

Each recommendation needed to be evidence-based and measurable, have a clinician champion to implement the recommendation, and have the potential to improve the quality and safety of the care we provided. “Balancing” measures needed to be carefully monitored to ensure that no diagnoses were being missed or negative effects resulted from decreasing these interventions. In order for a recommendation to be considered, a subgroup of the pediatric department’s clinical advisory committee reviewed the references provided to ensure that what was being suggested was based on published evidence and part of current national guidelines. The clinician champion needed to agree to lead the implementation project, and specific outcomes, including appropriate balancing measures, needed to be identified a priori, in addition to an appropriate mechanism to collect the data. Hospital executive leaders were supportive of the initiative and facilitated access to “in-kind” hospital resources as required, although no financial budget was provided. After some early success, the Department of Paediatrics provided part-time project management support to help coordinate the growth and administration of the initiative.

Measuring and Supporting Practice Change

The main implementation principles included targeted education/awareness, transparent measurement with audit/feedback, and, most importantly, embedding changes in the ordering process, essentially making it easy for frontline clinicians to do the right thing (and trickier to do the “wrong” thing). Audit and feedback have been used at both the individual provider level (eg, respiratory viral testing–ordering practices) and the divisional level (eg, ordering of postoperative antibiotics). These quality improvement initiatives have had a compelling impact. Scorecards have been developed and results shared internally using local divisional as well as hospital-wide tools, varying from staff meetings to screensavers across hospital computers and television screens and the hospital intranet. Evaluation is ongoing, but many of the initial results have been encouraging.15-17

For example, the 2016 list includes recommendations related to emergency department (ED) test ordering. Implementation efforts to address unnecessary nasopharyngeal swabs for viral testing in bronchiolitis reduced this practice by 80%,15 and there has been a 50% reduction in ankle X-rays in children with acute ankle injuries who meet criteria for a low-risk examination.16 The 2016 list also included a recommendation related to inappropriate intravenous immunoglobulin (IVIG) use in children with typical acute immune thrombocytopenic purpura (ITP), and a targeted quality improvement initiative reduced inappropriate IVIG use by 50%, with no detectable increase in bleeding complications or readmission to hospital.17 These results have been sustained over a period of 3 or more years. Examples from the 2018 list include a 40% decrease in inappropriate urinary tract infection diagnosis and treatment in the ED and a four-fold decrease in the CT abdomen/pelvis imaging rate for low-risk trauma.18

The steering committee meets every 2 months and includes all of the clinician champions as well as representatives from strategic hospital resources and two family advisors (NGS). These meetings are chaired by the Associate Pediatrician-in-Chief (JNF) and the project manager. The progress of the active projects is discussed, and the experience of the group is used to problem-solve, plan ahead, and encourage academic presentation and publication of the various projects. Patient partnership and participation in committees has ensured that improvements to patient experience, satisfaction, and education are considered in the outcomes of implementation. Moreover, it has safeguarded that this effort is not misperceived as limiting care and remains focused on advancing quality, safety, and the patient experience.

SOME LESSONS LEARNED

While most projects have surpassed expectations, not all have proceeded as anticipated. The biggest challenge is finding a reliable and practical source for data collection. For example, at the time of initiation of the voiding cystourethrography (VCUG) recommendation, practice had presumably changed over the recent years, and compliance already exceeded the goal, illustrating the importance of current accurate data. The oxygen saturation–monitoring recommendation highlighted the challenge presented by data collection that requires manual audits; the inability to find staff to do this regularly significantly hampered this project. The critical role of the clinician champion was highlighted in a few projects when a lead was absent for a prolonged period of time (eg, due to a parental leave or change in job), with no willing replacement. There does seem to be a strong correlation between the commitment and passion of the clinician lead and the success of the project. We have incorporated the lessons learned into the development and rollout of the 2018 and 2021 lists.

SPREAD AND SCALE

The challenge is to scale up these successes to impact and change practice across the hospital pediatrics community. After 5 years, awareness of and engagement with this process are still not uniform across our hospital campus. Nevertheless, anecdotally, at the Hospital for Sick Children, there is a shift in culture where clinicians have processed the imperative to reduce overuse and unnecessary tests and treatments, with phrases such as “this is not very Choosing Wisely” entering the vernacular. It is becoming part of the culture. Second, the new generation of medical school trainees and residents has displayed a tremendous appetite and passion for stewardship and a sense that practice can change from the ground up. The SickKids Choosing Wisely efforts have been a hub for resident-led quality improvement projects and leadership for implementation of recommendations.19 As we continue to engage all providers at our hospital, we are also reaching out to the other community hospitals in our region, and all children’s hospitals in Canada, to share the principles and lessons learned from our program through a national community of practice.

CONCLUSION

Practicing pediatric medicine in a well-resourced hospital setting should not drive us to overuse in practice “just because we can.” The harms of this approach to our patients and health systems, coupled with the pressures of the pandemic, are compelling reasons to be responsible stewards. There are opportunities to reshape and rethink practice patterns and habits.20 Overuse and overdiagnosis harm our patients and families physically and emotionally and indirectly waste resources urgently needed for investment upstream. Providing safe, quality, high-value care to our young patients requires constant critical thinking. The time is here to advance Choosing Wisely into pediatric hospital practice.

References

1. Elliott DK, Rose SR, Ronan JC. Changing the culture around cultures. Hosp Pediatr. 2014;4(6):405-407. https://doi.org/10.1542/hpeds.2014-0064
2. Gupta R, Simpson LA, Morgan DJ. Prioritizing high-value, equitable care after the COVID-19 shutdown: an opportunity for a healthcare renaissance. J Hosp Med. 2021;16(2):114-116. https://doi.org/10.12788/jhm.3526
3. Born K, Kool T, Levinson W. Reducing overuse in healthcare: advancing Choosing Wisely. BMJ. 2019;367:l6317. https://doi.org/10.1136/bmj.l6317
4. Coon ER, Young PC, Quinonez RA, Morgan DJ, Dhruva SS, Schroeder AR. Update on pediatric overuse. Pediatrics. 2017;139(2):e20162797. https://doi.org/10.1542/peds.2016-2797
5. Coon ER, Quinonez RA, Moyer VA, Schroeder AR. Overdiagnosis: how our compulsion for diagnosis may be harming children. Pediatrics. 2014;134(5):1013-1023. https://doi.org/10.1542/peds.2014-1778
6. Wolf ER, Krist AH, Schroeder AR. Deimplementation in pediatrics: past, present, and future. JAMA Pediatr. 2021;175(3):230-232. https://doi.org/10.1001/jamapediatrics.2020.4681
7. Quinonez RA, Garber MD, Schroeder AR, et al. Choosing Wisely in pediatric hospital medicine: five opportunities for improved healthcare value. J Hosp Med. 2013;8(9):479-485. https://doi.org/10.1002/jhm.2064
8. Bonafide CP, Xiao R, Brady PW, et al. Prevalence of continuous pulse oximetry monitoring in hospitalized children with bronchiolitis not requiring supplemental oxygen. JAMA. 2020;323(15):1467-1477. https://doi.org/10.1001/jama.2020.2998
9. Ralston SL, Schroeder AR. Why is it so hard to talk about overuse in pediatrics and why it matters. JAMA Pediatr. 2017;17(10):931-932. https://doi.org/10.1001/jamapediatrics.2017.2239
10. Stammen LA, Stalmeijer RE, Paternotte E, et al. Training physicians to provide high-value, cost-conscious care: a systematic review. JAMA. 2015;314(22):2384-2400. https://doi.org/10.1001/jama.2015.16353
11. Mathews JD, Forsythe AV, Brady Z, et al. Cancer risk in 680,000 people exposed to computed tomography scans in childhood or adolescence: data linkage study of 11 million Australians. BMJ. 2013;346:f2360. https://doi.org/10.1136/bmj.f2360
12. Friedman JN. Saying yes to the less: making it easier to choose wisely [editorial]. J Pediatr. 2017;145:4-5. https://doi.org/10.1016/j.jpeds.2017.01.062
13. Canadian Paediatric Society. Five things physicians and patients should question. Choosing Wisely Canada. Updated July 2019. Accessed June 17, 2021. https://choosingwiselycanada.org/wp-content/uploads/2020/07/Paediatrics_EN.pdf
14. Parikh K, Hall M, Montalbano A, et al. Establishing benchmarks for the hospitalized care of children with asthma, bronciolitis, and pneumonia. Pediatrics. 2014;134(3):555-562. https://doi.org/10.1542/peds.2014-1052
15. Ostrow O, Richardson S, Savlov D, Friedman JN. Reducing unnecessary respiratory viral testing to promote high value care. Pediatrics. In press.
16. Al-Sani F, Ben-Yakov M, Harvey G, et al. P016: Low risk ankle rule, high reward—a quality improvement initiative to reduce ankle x-rays in the pediatric emergency department [poster]. CJEM. 2017;19(S1):S83. https://doi.org/10.1017/cem.2017.218
17. Beck CE, Carcao M, Cada M, Porter S, Blanchette VS, Parkin PC. A quality improvement bundle to improve informed choice for children with typical, newly diagnosed immune thrombocytopenia. J Pediatr Hematol Oncol. 2018;40(8):e537-e543. https://doi.org/10.1097/MPH.0000000000001247
18. Beno S, Lenton-Brym T, Rosenfield D, McDowall D, Wales P, Principi T. Safe reduction of abdominal CT imaging in pediatric trauma patients: a quality-improvement initiative [abstract]. Can J Surg. 2019;62(3 Suppl 2):S29-S30.
19. Bal C, Tesch M, Blair G, Ostrow O, Premji L. Engaging medical trainees in resource stewardship through resident-led teaching sessions: a choosing wisely educational initiative. Can Med Educ J. 2021;12(1):e98-e100. https://doi.org/10.36834/cmej.70563
20. Berwick DM. Choices for the “new normal.” JAMA. 2020;323(21):2125-2126. https://doi.org/10.1001/jama.2020.6949

References

1. Elliott DK, Rose SR, Ronan JC. Changing the culture around cultures. Hosp Pediatr. 2014;4(6):405-407. https://doi.org/10.1542/hpeds.2014-0064
2. Gupta R, Simpson LA, Morgan DJ. Prioritizing high-value, equitable care after the COVID-19 shutdown: an opportunity for a healthcare renaissance. J Hosp Med. 2021;16(2):114-116. https://doi.org/10.12788/jhm.3526
3. Born K, Kool T, Levinson W. Reducing overuse in healthcare: advancing Choosing Wisely. BMJ. 2019;367:l6317. https://doi.org/10.1136/bmj.l6317
4. Coon ER, Young PC, Quinonez RA, Morgan DJ, Dhruva SS, Schroeder AR. Update on pediatric overuse. Pediatrics. 2017;139(2):e20162797. https://doi.org/10.1542/peds.2016-2797
5. Coon ER, Quinonez RA, Moyer VA, Schroeder AR. Overdiagnosis: how our compulsion for diagnosis may be harming children. Pediatrics. 2014;134(5):1013-1023. https://doi.org/10.1542/peds.2014-1778
6. Wolf ER, Krist AH, Schroeder AR. Deimplementation in pediatrics: past, present, and future. JAMA Pediatr. 2021;175(3):230-232. https://doi.org/10.1001/jamapediatrics.2020.4681
7. Quinonez RA, Garber MD, Schroeder AR, et al. Choosing Wisely in pediatric hospital medicine: five opportunities for improved healthcare value. J Hosp Med. 2013;8(9):479-485. https://doi.org/10.1002/jhm.2064
8. Bonafide CP, Xiao R, Brady PW, et al. Prevalence of continuous pulse oximetry monitoring in hospitalized children with bronchiolitis not requiring supplemental oxygen. JAMA. 2020;323(15):1467-1477. https://doi.org/10.1001/jama.2020.2998
9. Ralston SL, Schroeder AR. Why is it so hard to talk about overuse in pediatrics and why it matters. JAMA Pediatr. 2017;17(10):931-932. https://doi.org/10.1001/jamapediatrics.2017.2239
10. Stammen LA, Stalmeijer RE, Paternotte E, et al. Training physicians to provide high-value, cost-conscious care: a systematic review. JAMA. 2015;314(22):2384-2400. https://doi.org/10.1001/jama.2015.16353
11. Mathews JD, Forsythe AV, Brady Z, et al. Cancer risk in 680,000 people exposed to computed tomography scans in childhood or adolescence: data linkage study of 11 million Australians. BMJ. 2013;346:f2360. https://doi.org/10.1136/bmj.f2360
12. Friedman JN. Saying yes to the less: making it easier to choose wisely [editorial]. J Pediatr. 2017;145:4-5. https://doi.org/10.1016/j.jpeds.2017.01.062
13. Canadian Paediatric Society. Five things physicians and patients should question. Choosing Wisely Canada. Updated July 2019. Accessed June 17, 2021. https://choosingwiselycanada.org/wp-content/uploads/2020/07/Paediatrics_EN.pdf
14. Parikh K, Hall M, Montalbano A, et al. Establishing benchmarks for the hospitalized care of children with asthma, bronciolitis, and pneumonia. Pediatrics. 2014;134(3):555-562. https://doi.org/10.1542/peds.2014-1052
15. Ostrow O, Richardson S, Savlov D, Friedman JN. Reducing unnecessary respiratory viral testing to promote high value care. Pediatrics. In press.
16. Al-Sani F, Ben-Yakov M, Harvey G, et al. P016: Low risk ankle rule, high reward—a quality improvement initiative to reduce ankle x-rays in the pediatric emergency department [poster]. CJEM. 2017;19(S1):S83. https://doi.org/10.1017/cem.2017.218
17. Beck CE, Carcao M, Cada M, Porter S, Blanchette VS, Parkin PC. A quality improvement bundle to improve informed choice for children with typical, newly diagnosed immune thrombocytopenia. J Pediatr Hematol Oncol. 2018;40(8):e537-e543. https://doi.org/10.1097/MPH.0000000000001247
18. Beno S, Lenton-Brym T, Rosenfield D, McDowall D, Wales P, Principi T. Safe reduction of abdominal CT imaging in pediatric trauma patients: a quality-improvement initiative [abstract]. Can J Surg. 2019;62(3 Suppl 2):S29-S30.
19. Bal C, Tesch M, Blair G, Ostrow O, Premji L. Engaging medical trainees in resource stewardship through resident-led teaching sessions: a choosing wisely educational initiative. Can Med Educ J. 2021;12(1):e98-e100. https://doi.org/10.36834/cmej.70563
20. Berwick DM. Choices for the “new normal.” JAMA. 2020;323(21):2125-2126. https://doi.org/10.1001/jama.2020.6949

Publications
Publications
Topics
Article Type
Display Headline
Improving Healthcare Value: Reducing Overuse in Hospital Pediatrics
Display Headline
Improving Healthcare Value: Reducing Overuse in Hospital Pediatrics
Sections
Article Source

© 2021 Society of Hospital Medicine

Citation Override
J Hosp Med. Published Online Only October 20, 2021. DOI: 10.12788/jhm.3686
Disallow All Ads
Correspondence Location
Karen Born, MSc, PhD; Email: Karen.born@utoronto.ca; Telephone: 416-254-7505; Twitter: @bornk.
Content Gating
Gated (full article locked unless allowed per User)
Alternative CME
Disqus Comments
Default
Use ProPublica
Hide sidebar & use full width
render the right sidebar.
Conference Recap Checkbox
Not Conference Recap
Clinical Edge
Display the Slideshow in this Article
Gating Strategy
First Page Free
Medscape Article
Display survey writer
Reuters content
Disable Inline Native ads
WebMD Article
Article PDF Media

Things We Do for No Reason™: Routine Use of Corticosteroids for the Treatment of Anaphylaxis

Article Type
Changed
Wed, 10/13/2021 - 01:15
Display Headline
Things We Do for No Reason™: Routine Use of Corticosteroids for the Treatment of Anaphylaxis

Inspired by the ABIM Foundation’s Choosing Wisely® campaign, the “Things We Do for No Reason” (TWDFNR) series reviews practices that have become common parts of hospital care but may provide little value to our patients. Practices reviewed in the TWDFNR series do not represent clear-cut conclusions or clinical practice standards but are meant as a starting place for research and active discussions among hospitalists and patients. We invite you to be part of that discussion.

CLINICAL SCENARIO

A 56-year-old man with coronary artery disease (CAD) undergoes hospital treatment for diverticulitis. He receives ketorolac for abdominal pain upon arrival to the medical ward despite his known allergy to nonsteroidal anti-inflammatory drugs. Fifteen minutes after administration, he develops lightheadedness and experiences swelling of his lips. On exam, he has tachycardia and a diffuse urticarial rash across his torso. The admitting physician prescribes methylprednisolone, diphenhydramine, and a liter bolus of normal saline for suspected anaphylaxis. Epinephrine is not administered for fear of precipitating an adverse cardiovascular event given the patient’s history of CAD.

BACKGROUND

Anaphylaxis, a rapid-onset generalized immunoglobulin E (IgE)–mediated hypersensitivity reaction, can lead to significant morbidity and mortality when not managed properly. Patients can present with anaphylaxis in heterogeneous ways. Fulfilling any one of three criteria establishes the diagnosis of anaphylaxis: (1) rapid onset of skin or mucosal symptoms complicated by either respiratory compromise or hypotension; (2) two or more symptoms involving the respiratory, mucosal, cardiovascular, or gastrointestinal systems following exposure to a likely allergen; and (3) reduced blood pressure in response to a known allergen.1 Up to 5% of the population experiences anaphylaxis in a lifetime. Medication and stinging insects account for the majority of anaphylactic reactions in adults, while food and insect stings commonly trigger it in children and adolescents.2

The majority of anaphylactic reactions, known as uniphasic or monophasic, occur rapidly as single episodes following exposure to a specific trigger and resolve within minutes to hours after treatment. Meanwhile, biphasic, or delayed-phase, anaphylaxis occurs when symptoms recur after an apparent resolution and in the absence of reexposure to the trigger. Symptoms restart within 1 to 72 hours after resolution of an initial anaphylaxis episode, with a median time to onset of 11 hours. Biphasic reactions occur in roughly 5% of patients with anaphylaxis.3

Epinephrine is the only recommended first-line medication for the treatment of anaphylaxis in all age groups.4 Epinephrine counteracts the cardiovascular and respiratory compromise induced by anaphylaxis through its α- and β-adrenergic activity and stabilizes mast cells.4 Early administration of intramuscular epinephrine decreases the need for additional interventions, reduces the likelihood of hospitalization, and is associated with reduced biphasic reactions.5-7 Paradoxically, patients receive corticosteroids more often than epinephrine for suspected anaphylaxis, despite no robust evidence for their efficacy.4,8,9

WHY YOU MIGHT THINK STEROIDS aRE HELPFUL FOR ANAPHYLAXIS

Corticosteroids act as potent anti-inflammatory medications that modulate mast-cell maturation, activation, and degranulation. Known to work primarily through downregulation of gene transcription responsible for cytokine, chemokine, and arachidonic acid production, their maximal anti-inflammatory effects manifest 2 to 6 hours after administration. Demonstrated efficacy in treating and preventing relapse of other inflammatory conditions, such as asthma and croup, may, in part, explain the widespread glucocorticoid use in anaphylaxis. Some believe that administration of corticosteroids may also help reduce the risk of biphasic or delayed-phase anaphylaxis.10

WHY THERE IS NO REASON TO PRESCRIBE CORTICOSTEROIDS FOR ANAPHYLAXIS

Based on their mechanism of action, corticosteroids do not exert any anti-inflammatory effects for several hours, regardless of their route of administration.10 In contrast, epinephrine exerts an almost immediate effect to increase cardiac output and vascular resistance, to reverse edema and bronchoconstriction, and to stabilize mast cells, preventing release of harmful chemokines and cytokines.4

The American Academy of Allergy, Asthma & Immunology (AAAAI) recommends early administration of epinephrine as the first-line treatment of anaphylaxis and emphasizes that evidence does not support routine corticosteroid use in the management of acute anaphylaxis or for prevention of biphasic reactions.9 To date, no randomized controlled trials have explored the role of corticosteroids in the treatment of acute anaphylaxis, although one is currently under way looking at whether dexamethasone has an impact on preventing biphasic reactions (Table).11

jhmvol16no10_auth02881013e_t1.jpg

The AAAAI Joint Task Force on Practice Parameters (JTFPP) conducted a pooled analysis of observational studies that did not find a reduction in biphasic reactions in adult patients receiving corticosteroids (odds ratio [OR], 0.87; 95% CI, 0.74-1.02).9 Further, their analysis suggests an association with administration of corticosteroids and an increased likelihood of biphasic reactions in children (OR, 1.55; 95% CI, 1.01-2.38).9

An observational study in children across 35 hospitals demonstrated an association with corticosteroid administration and a reduced length of hospital stay for anaphylaxis, but the same study found no reduction in repeat emergency department (ED) visits within 72 hours.12 Similarly, a retrospective cohort study in adults did not find that corticosteroid administration reduced the 7-day risk of returning to the hospital.13 These studies highlight the importance of anticipatory guidance in both ED and hospital discharges for anaphylaxis since the literature does not provide data that corticosteroid administration reduces the likelihood of a biphasic course.

Long-term corticosteroids have well-known deleterious health effects. Recent evidence highlights the possible adverse events associated with even short courses of corticosteroids. A large case series from Taiwan containing 2,623,327 adults administered brief courses (<14 days) of corticosteroids demonstrated increased incidence of gastrointestinal bleeding, sepsis, and heart failure beginning 5 to 30 days after starting corticosteroid treatments for common medical conditions, with respective absolute risk increases of 10.3, 0.1, and 1 per 1000 patient-years for each condition.14 The same group of researchers found a nearly two-fold increased risk of sepsis, gastrointestinal bleeding, and pneumonia in a nearly 1 million children who had received corticosteroids within the previous year.15 Other common side effects of short-term corticosteroids include insomnia, agitation, mood disturbances, and hyperglycemia.

A growing body of evidence demonstrates that corticosteroids likely do not alter the natural disease course of anaphylaxis and carry increased risks of significant adverse events. The AAAAI recommends against the use of glucocorticoids as a first-line agent for anaphylaxis and suggests against the use of glucocorticoids to prevent biphasic reactions.9

WHEN TREATING WITH CORTICOSTEROIDS MAY BE INDICATED

The recent JTFPP analysis of observational studies demonstrated reduced hypersensitivity reactions to chemotherapeutics with corticosteroid premedication (OR, 0.49; 95% CI, 0.37-0.66). The AAAAI favors administration of corticosteroids to reduce the risk of anaphylactoid reactions—non–IgE-mediated mast cell activation—for some chemotherapeutic protocols.9

There is robust evidence regarding the benefits of corticosteroids in the treatment of asthma and upper-airway edema.16,17 Allergen exposures can precipitate significant bronchospasm in individuals with asthma and trigger an exacerbation. Although routine corticosteroid use for anaphylaxis in these populations has not been directly studied, their use as an adjunctive therapy may be beneficial if there is clinical evidence of bronchospasm or significant upper-airway edema.

WHAT YOU SHOULD DO INSTEAD

Rapid administration of epinephrine saves lives, reduces need for adjuvant treatments and hospitalization, and is associated with decreased risk of developing biphasic anaphylactic reactions (OR, 0.2; 95% CI, 0-0.6).5-7 Some clinicians are apprehensive about using epinephrine owing to fears related to negative side effects, particularly adverse cardiovascular events. Kawano et al18 performed a retrospective evaluation of 492 ED visits for anaphylaxis and found that epinephrine is administered less often in older patients (age >50 years); however, when administered intramuscularly, there was no significant difference in adverse cardiovascular events in this population compared with younger individuals. The study did demonstrate an increased rate of adverse cardiac events in older patients receiving intravenous epinephrine, an observation that the authors attributed partly to dosing errors that were reported more often with intravenous use.18

RECOMMENDATIONS

  • Always promptly administer intramuscular epinephrine when treating anaphylaxis.
  • Routine administration of corticosteroids in the treatment of anaphylaxis is not advised owing to insufficient data supporting their efficacy and potential for adverse events. Some patient populations may derive benefit from corticosteroids, including individuals with history of asthma exhibiting bronchospastic symptoms, individuals with significant upper-airway edema, and those undergoing certain chemotherapy regimens.

CONCLUSIONS

In the clinical vignette, the hospitalist withheld the first-line treatment for anaphylaxis, epinephrine. Without the support of evidence in the literature, patients receive corticosteroids and antihistamines more often than epinephrine for suspected anaphylaxis. No evidence supports the routine use of corticosteroids in the management of anaphylaxis or in the prevention of biphasic reactions. Further, recent research demonstrates significant adverse events are associated with even short courses of corticosteroids.

Do you think this is a low-value practice? Is this truly a “Thing We Do for No Reason”? Share what you do in your practice and join in the conversation online by retweeting it on Twitter (#TWDFNR) and liking it on Facebook. We invite you to propose ideas for other “Things We Do for No Reason” topics by emailing TWDFNR@hospitalmedicine.org.

References

1. Sampson HA, Muñoz-Furlong A, Campbell RL, et al. Second symposium on the definition and management of anaphylaxis: summary report--Second National Institute of Allergy and Infectious Disease/Food Allergy and Anaphylaxis Network symposium. J Allergy Clin Immunol. 2006;117(2):391-397. https://doi.org/10.1016/j.jaci.2005.12.1303
2. Wood RA, Camargo CA Jr, Lieberman P, et al. Anaphylaxis in America: the prevalence and characteristics of anaphylaxis in the United States. J Allergy Clin Immunol. 2014;133(2):461-467. https://doi.org/10.1016/j.jaci.2013.08.016
3. Lee S, Bellolio MF, Hess EP, Erwin P, Murad MH, Campbell RL. Time of onset and predictors of biphasic anaphylactic reactions: a systematic review and meta-analysis. J Allergy Clin Immunol Pract. 2015;3(3):408-16.e162. https://doi.org/10.1016/j.jaip.2014.12.010
4. Simons KJ, Simons FE. Epinephrine and its use in anaphylaxis: current issues. Curr Opin Allergy Clin Immunol. 2010;10(4):354-361. https://doi.org/10.1097/ACI.0b013e32833bc670
5. Fleming JT, Clark S, Camargo CA Jr, Rudders SA. Early treatment of food-induced anaphylaxis with epinephrine is associated with a lower risk of hospitalization. J Allergy Clin Immunol Pract. 2015;3(1):57-62. https://doi.org/10.1016/j.jaip.2014.07.004
6. Sundquist BK, Jose J, Pauze D, Pauze D, Wang H, Järvinen KM. Anaphylaxis risk factors for hospitalization and intensive care: a comparison between adults and children in an upstate New York emergency department. Allergy Asthma Proc. 2019;40(1):41-47. https://doi.org/10.2500/aap.2019.40.4189
7. Hochstadter E, Clarke A, De Schryver S, et al. Increasing visits for anaphylaxis and the benefits of early epinephrine administration: a 4-year study at a pediatric emergency department in Montreal, Canada. J Allergy Clin Immunol. 2016;137(6):1888-1890.e4. https://doi.org/10.1016/j.jaci.2016.02.016
8. Worm M, Moneret-Vautrin A, Scherer K, et al. First European data from the network of severe allergic reactions (NORA). Allergy. 2014;69(10):1397-1404. https://doi.org/10.1111/all.12475
9. Shaker MS, Wallace DV, Golden DBK, et al. Anaphylaxis—a 2020 practice parameter update, systemic review, and Grading of Recommendations, Assessment, Development, and Evaluation (GRADE) analysis. J Allergy Clin Immunol. 2020;145(4):1082-1123. https://doi.org/10.1016/j.jaci.2020.01.017
10. Liyanage CK, Galappatthy P, Seneviratne SL. Corticosteroids in management of anaphylaxis; a systematic review of evidence. Eur Ann Allergy Clin Immunol. 2017;49(5):196-207. https://doi.org/10.23822/EurAnnACI.1764-1489.15
11. Use of dexamethasone in prevention of the second phase of a biphasic reaction of anaphylaxis. ClinicalTrials.gov identifier: NCT03523221. Updated July 29, 2020. Accessed July 16, 2021. https://clinicaltrials.gov/ct2/show/NCT03523221
12. Michelson KA, Monuteaux MC, Neuman MI. Glucocorticoids and hospital length of stay for children with anaphylaxis: a retrospective study. J Pediatr. 2015;167(3):719-24.e243. https://doi.org/10.1016/j.jpeds.2015.05.033
13. Grunau BE, Wiens MO, Rowe BH, et al. Emergency department corticosteroid use for allergy or anaphylaxis is not associated with decreased relapses. Ann Emerg Med. 2015;66(4):381-389. https://doi.org/10.1016/j.annemergmed.2015.03.003
14. Yao TC, Huang YW, Chang SM, Tsai SY, Wu AC, Tsai HJ. Association between oral corticosteroid bursts and severe adverse events: a nationwide population-based cohort study. Ann Intern Med. 2020;173(5):325-330. https://doi.org/10.7326/M20-0432
15. Yao TC, Wang JY, Chang SM, et al. Association of oral corticosteroid bursts with severe adverse events in children. JAMA Pediatr. 2021;175(7):723-729. https://doi.org/10.1001/jamapediatrics.2021.0433
16. Rowe BH, Spooner CH, Ducharme FM, Bretzlaff JA, Bota GW. Corticosteroids for preventing relapse following acute exacerbations of asthma. Cochrane Database Syst Rev. 2007 Jul 18;(3):CD000195. https://doi.org/10.1002/14651858.CD000195.pub2
17. Gates A, Gates M, Vandermeer B, et al. Glucocorticoids for croup in children. Cochrane Database Syst Rev. 2018;8(8):CD001955. https://doi.org/10.1002/14651858.CD001955.pub4
18. Kawano T, Scheuermeyer FX, Stenstrom R, Rowe BH, Grafstein E, Grunau B. Epinephrine use in older patients with anaphylaxis: clinical outcomes and cardiovascular complications. Resuscitation. 2017;112:53-58. https://doi.org/10.1016/j.resuscitation.2016.12.020

Article PDF
Author and Disclosure Information

Lifespan Health System, Providence, Rhode Island.

Disclosures
The authors reported no conflicts of interest.

Publications
Topics
Sections
Author and Disclosure Information

Lifespan Health System, Providence, Rhode Island.

Disclosures
The authors reported no conflicts of interest.

Author and Disclosure Information

Lifespan Health System, Providence, Rhode Island.

Disclosures
The authors reported no conflicts of interest.

Article PDF
Article PDF
Related Articles

Inspired by the ABIM Foundation’s Choosing Wisely® campaign, the “Things We Do for No Reason” (TWDFNR) series reviews practices that have become common parts of hospital care but may provide little value to our patients. Practices reviewed in the TWDFNR series do not represent clear-cut conclusions or clinical practice standards but are meant as a starting place for research and active discussions among hospitalists and patients. We invite you to be part of that discussion.

CLINICAL SCENARIO

A 56-year-old man with coronary artery disease (CAD) undergoes hospital treatment for diverticulitis. He receives ketorolac for abdominal pain upon arrival to the medical ward despite his known allergy to nonsteroidal anti-inflammatory drugs. Fifteen minutes after administration, he develops lightheadedness and experiences swelling of his lips. On exam, he has tachycardia and a diffuse urticarial rash across his torso. The admitting physician prescribes methylprednisolone, diphenhydramine, and a liter bolus of normal saline for suspected anaphylaxis. Epinephrine is not administered for fear of precipitating an adverse cardiovascular event given the patient’s history of CAD.

BACKGROUND

Anaphylaxis, a rapid-onset generalized immunoglobulin E (IgE)–mediated hypersensitivity reaction, can lead to significant morbidity and mortality when not managed properly. Patients can present with anaphylaxis in heterogeneous ways. Fulfilling any one of three criteria establishes the diagnosis of anaphylaxis: (1) rapid onset of skin or mucosal symptoms complicated by either respiratory compromise or hypotension; (2) two or more symptoms involving the respiratory, mucosal, cardiovascular, or gastrointestinal systems following exposure to a likely allergen; and (3) reduced blood pressure in response to a known allergen.1 Up to 5% of the population experiences anaphylaxis in a lifetime. Medication and stinging insects account for the majority of anaphylactic reactions in adults, while food and insect stings commonly trigger it in children and adolescents.2

The majority of anaphylactic reactions, known as uniphasic or monophasic, occur rapidly as single episodes following exposure to a specific trigger and resolve within minutes to hours after treatment. Meanwhile, biphasic, or delayed-phase, anaphylaxis occurs when symptoms recur after an apparent resolution and in the absence of reexposure to the trigger. Symptoms restart within 1 to 72 hours after resolution of an initial anaphylaxis episode, with a median time to onset of 11 hours. Biphasic reactions occur in roughly 5% of patients with anaphylaxis.3

Epinephrine is the only recommended first-line medication for the treatment of anaphylaxis in all age groups.4 Epinephrine counteracts the cardiovascular and respiratory compromise induced by anaphylaxis through its α- and β-adrenergic activity and stabilizes mast cells.4 Early administration of intramuscular epinephrine decreases the need for additional interventions, reduces the likelihood of hospitalization, and is associated with reduced biphasic reactions.5-7 Paradoxically, patients receive corticosteroids more often than epinephrine for suspected anaphylaxis, despite no robust evidence for their efficacy.4,8,9

WHY YOU MIGHT THINK STEROIDS aRE HELPFUL FOR ANAPHYLAXIS

Corticosteroids act as potent anti-inflammatory medications that modulate mast-cell maturation, activation, and degranulation. Known to work primarily through downregulation of gene transcription responsible for cytokine, chemokine, and arachidonic acid production, their maximal anti-inflammatory effects manifest 2 to 6 hours after administration. Demonstrated efficacy in treating and preventing relapse of other inflammatory conditions, such as asthma and croup, may, in part, explain the widespread glucocorticoid use in anaphylaxis. Some believe that administration of corticosteroids may also help reduce the risk of biphasic or delayed-phase anaphylaxis.10

WHY THERE IS NO REASON TO PRESCRIBE CORTICOSTEROIDS FOR ANAPHYLAXIS

Based on their mechanism of action, corticosteroids do not exert any anti-inflammatory effects for several hours, regardless of their route of administration.10 In contrast, epinephrine exerts an almost immediate effect to increase cardiac output and vascular resistance, to reverse edema and bronchoconstriction, and to stabilize mast cells, preventing release of harmful chemokines and cytokines.4

The American Academy of Allergy, Asthma & Immunology (AAAAI) recommends early administration of epinephrine as the first-line treatment of anaphylaxis and emphasizes that evidence does not support routine corticosteroid use in the management of acute anaphylaxis or for prevention of biphasic reactions.9 To date, no randomized controlled trials have explored the role of corticosteroids in the treatment of acute anaphylaxis, although one is currently under way looking at whether dexamethasone has an impact on preventing biphasic reactions (Table).11

jhmvol16no10_auth02881013e_t1.jpg

The AAAAI Joint Task Force on Practice Parameters (JTFPP) conducted a pooled analysis of observational studies that did not find a reduction in biphasic reactions in adult patients receiving corticosteroids (odds ratio [OR], 0.87; 95% CI, 0.74-1.02).9 Further, their analysis suggests an association with administration of corticosteroids and an increased likelihood of biphasic reactions in children (OR, 1.55; 95% CI, 1.01-2.38).9

An observational study in children across 35 hospitals demonstrated an association with corticosteroid administration and a reduced length of hospital stay for anaphylaxis, but the same study found no reduction in repeat emergency department (ED) visits within 72 hours.12 Similarly, a retrospective cohort study in adults did not find that corticosteroid administration reduced the 7-day risk of returning to the hospital.13 These studies highlight the importance of anticipatory guidance in both ED and hospital discharges for anaphylaxis since the literature does not provide data that corticosteroid administration reduces the likelihood of a biphasic course.

Long-term corticosteroids have well-known deleterious health effects. Recent evidence highlights the possible adverse events associated with even short courses of corticosteroids. A large case series from Taiwan containing 2,623,327 adults administered brief courses (<14 days) of corticosteroids demonstrated increased incidence of gastrointestinal bleeding, sepsis, and heart failure beginning 5 to 30 days after starting corticosteroid treatments for common medical conditions, with respective absolute risk increases of 10.3, 0.1, and 1 per 1000 patient-years for each condition.14 The same group of researchers found a nearly two-fold increased risk of sepsis, gastrointestinal bleeding, and pneumonia in a nearly 1 million children who had received corticosteroids within the previous year.15 Other common side effects of short-term corticosteroids include insomnia, agitation, mood disturbances, and hyperglycemia.

A growing body of evidence demonstrates that corticosteroids likely do not alter the natural disease course of anaphylaxis and carry increased risks of significant adverse events. The AAAAI recommends against the use of glucocorticoids as a first-line agent for anaphylaxis and suggests against the use of glucocorticoids to prevent biphasic reactions.9

WHEN TREATING WITH CORTICOSTEROIDS MAY BE INDICATED

The recent JTFPP analysis of observational studies demonstrated reduced hypersensitivity reactions to chemotherapeutics with corticosteroid premedication (OR, 0.49; 95% CI, 0.37-0.66). The AAAAI favors administration of corticosteroids to reduce the risk of anaphylactoid reactions—non–IgE-mediated mast cell activation—for some chemotherapeutic protocols.9

There is robust evidence regarding the benefits of corticosteroids in the treatment of asthma and upper-airway edema.16,17 Allergen exposures can precipitate significant bronchospasm in individuals with asthma and trigger an exacerbation. Although routine corticosteroid use for anaphylaxis in these populations has not been directly studied, their use as an adjunctive therapy may be beneficial if there is clinical evidence of bronchospasm or significant upper-airway edema.

WHAT YOU SHOULD DO INSTEAD

Rapid administration of epinephrine saves lives, reduces need for adjuvant treatments and hospitalization, and is associated with decreased risk of developing biphasic anaphylactic reactions (OR, 0.2; 95% CI, 0-0.6).5-7 Some clinicians are apprehensive about using epinephrine owing to fears related to negative side effects, particularly adverse cardiovascular events. Kawano et al18 performed a retrospective evaluation of 492 ED visits for anaphylaxis and found that epinephrine is administered less often in older patients (age >50 years); however, when administered intramuscularly, there was no significant difference in adverse cardiovascular events in this population compared with younger individuals. The study did demonstrate an increased rate of adverse cardiac events in older patients receiving intravenous epinephrine, an observation that the authors attributed partly to dosing errors that were reported more often with intravenous use.18

RECOMMENDATIONS

  • Always promptly administer intramuscular epinephrine when treating anaphylaxis.
  • Routine administration of corticosteroids in the treatment of anaphylaxis is not advised owing to insufficient data supporting their efficacy and potential for adverse events. Some patient populations may derive benefit from corticosteroids, including individuals with history of asthma exhibiting bronchospastic symptoms, individuals with significant upper-airway edema, and those undergoing certain chemotherapy regimens.

CONCLUSIONS

In the clinical vignette, the hospitalist withheld the first-line treatment for anaphylaxis, epinephrine. Without the support of evidence in the literature, patients receive corticosteroids and antihistamines more often than epinephrine for suspected anaphylaxis. No evidence supports the routine use of corticosteroids in the management of anaphylaxis or in the prevention of biphasic reactions. Further, recent research demonstrates significant adverse events are associated with even short courses of corticosteroids.

Do you think this is a low-value practice? Is this truly a “Thing We Do for No Reason”? Share what you do in your practice and join in the conversation online by retweeting it on Twitter (#TWDFNR) and liking it on Facebook. We invite you to propose ideas for other “Things We Do for No Reason” topics by emailing TWDFNR@hospitalmedicine.org.

Inspired by the ABIM Foundation’s Choosing Wisely® campaign, the “Things We Do for No Reason” (TWDFNR) series reviews practices that have become common parts of hospital care but may provide little value to our patients. Practices reviewed in the TWDFNR series do not represent clear-cut conclusions or clinical practice standards but are meant as a starting place for research and active discussions among hospitalists and patients. We invite you to be part of that discussion.

CLINICAL SCENARIO

A 56-year-old man with coronary artery disease (CAD) undergoes hospital treatment for diverticulitis. He receives ketorolac for abdominal pain upon arrival to the medical ward despite his known allergy to nonsteroidal anti-inflammatory drugs. Fifteen minutes after administration, he develops lightheadedness and experiences swelling of his lips. On exam, he has tachycardia and a diffuse urticarial rash across his torso. The admitting physician prescribes methylprednisolone, diphenhydramine, and a liter bolus of normal saline for suspected anaphylaxis. Epinephrine is not administered for fear of precipitating an adverse cardiovascular event given the patient’s history of CAD.

BACKGROUND

Anaphylaxis, a rapid-onset generalized immunoglobulin E (IgE)–mediated hypersensitivity reaction, can lead to significant morbidity and mortality when not managed properly. Patients can present with anaphylaxis in heterogeneous ways. Fulfilling any one of three criteria establishes the diagnosis of anaphylaxis: (1) rapid onset of skin or mucosal symptoms complicated by either respiratory compromise or hypotension; (2) two or more symptoms involving the respiratory, mucosal, cardiovascular, or gastrointestinal systems following exposure to a likely allergen; and (3) reduced blood pressure in response to a known allergen.1 Up to 5% of the population experiences anaphylaxis in a lifetime. Medication and stinging insects account for the majority of anaphylactic reactions in adults, while food and insect stings commonly trigger it in children and adolescents.2

The majority of anaphylactic reactions, known as uniphasic or monophasic, occur rapidly as single episodes following exposure to a specific trigger and resolve within minutes to hours after treatment. Meanwhile, biphasic, or delayed-phase, anaphylaxis occurs when symptoms recur after an apparent resolution and in the absence of reexposure to the trigger. Symptoms restart within 1 to 72 hours after resolution of an initial anaphylaxis episode, with a median time to onset of 11 hours. Biphasic reactions occur in roughly 5% of patients with anaphylaxis.3

Epinephrine is the only recommended first-line medication for the treatment of anaphylaxis in all age groups.4 Epinephrine counteracts the cardiovascular and respiratory compromise induced by anaphylaxis through its α- and β-adrenergic activity and stabilizes mast cells.4 Early administration of intramuscular epinephrine decreases the need for additional interventions, reduces the likelihood of hospitalization, and is associated with reduced biphasic reactions.5-7 Paradoxically, patients receive corticosteroids more often than epinephrine for suspected anaphylaxis, despite no robust evidence for their efficacy.4,8,9

WHY YOU MIGHT THINK STEROIDS aRE HELPFUL FOR ANAPHYLAXIS

Corticosteroids act as potent anti-inflammatory medications that modulate mast-cell maturation, activation, and degranulation. Known to work primarily through downregulation of gene transcription responsible for cytokine, chemokine, and arachidonic acid production, their maximal anti-inflammatory effects manifest 2 to 6 hours after administration. Demonstrated efficacy in treating and preventing relapse of other inflammatory conditions, such as asthma and croup, may, in part, explain the widespread glucocorticoid use in anaphylaxis. Some believe that administration of corticosteroids may also help reduce the risk of biphasic or delayed-phase anaphylaxis.10

WHY THERE IS NO REASON TO PRESCRIBE CORTICOSTEROIDS FOR ANAPHYLAXIS

Based on their mechanism of action, corticosteroids do not exert any anti-inflammatory effects for several hours, regardless of their route of administration.10 In contrast, epinephrine exerts an almost immediate effect to increase cardiac output and vascular resistance, to reverse edema and bronchoconstriction, and to stabilize mast cells, preventing release of harmful chemokines and cytokines.4

The American Academy of Allergy, Asthma & Immunology (AAAAI) recommends early administration of epinephrine as the first-line treatment of anaphylaxis and emphasizes that evidence does not support routine corticosteroid use in the management of acute anaphylaxis or for prevention of biphasic reactions.9 To date, no randomized controlled trials have explored the role of corticosteroids in the treatment of acute anaphylaxis, although one is currently under way looking at whether dexamethasone has an impact on preventing biphasic reactions (Table).11

jhmvol16no10_auth02881013e_t1.jpg

The AAAAI Joint Task Force on Practice Parameters (JTFPP) conducted a pooled analysis of observational studies that did not find a reduction in biphasic reactions in adult patients receiving corticosteroids (odds ratio [OR], 0.87; 95% CI, 0.74-1.02).9 Further, their analysis suggests an association with administration of corticosteroids and an increased likelihood of biphasic reactions in children (OR, 1.55; 95% CI, 1.01-2.38).9

An observational study in children across 35 hospitals demonstrated an association with corticosteroid administration and a reduced length of hospital stay for anaphylaxis, but the same study found no reduction in repeat emergency department (ED) visits within 72 hours.12 Similarly, a retrospective cohort study in adults did not find that corticosteroid administration reduced the 7-day risk of returning to the hospital.13 These studies highlight the importance of anticipatory guidance in both ED and hospital discharges for anaphylaxis since the literature does not provide data that corticosteroid administration reduces the likelihood of a biphasic course.

Long-term corticosteroids have well-known deleterious health effects. Recent evidence highlights the possible adverse events associated with even short courses of corticosteroids. A large case series from Taiwan containing 2,623,327 adults administered brief courses (<14 days) of corticosteroids demonstrated increased incidence of gastrointestinal bleeding, sepsis, and heart failure beginning 5 to 30 days after starting corticosteroid treatments for common medical conditions, with respective absolute risk increases of 10.3, 0.1, and 1 per 1000 patient-years for each condition.14 The same group of researchers found a nearly two-fold increased risk of sepsis, gastrointestinal bleeding, and pneumonia in a nearly 1 million children who had received corticosteroids within the previous year.15 Other common side effects of short-term corticosteroids include insomnia, agitation, mood disturbances, and hyperglycemia.

A growing body of evidence demonstrates that corticosteroids likely do not alter the natural disease course of anaphylaxis and carry increased risks of significant adverse events. The AAAAI recommends against the use of glucocorticoids as a first-line agent for anaphylaxis and suggests against the use of glucocorticoids to prevent biphasic reactions.9

WHEN TREATING WITH CORTICOSTEROIDS MAY BE INDICATED

The recent JTFPP analysis of observational studies demonstrated reduced hypersensitivity reactions to chemotherapeutics with corticosteroid premedication (OR, 0.49; 95% CI, 0.37-0.66). The AAAAI favors administration of corticosteroids to reduce the risk of anaphylactoid reactions—non–IgE-mediated mast cell activation—for some chemotherapeutic protocols.9

There is robust evidence regarding the benefits of corticosteroids in the treatment of asthma and upper-airway edema.16,17 Allergen exposures can precipitate significant bronchospasm in individuals with asthma and trigger an exacerbation. Although routine corticosteroid use for anaphylaxis in these populations has not been directly studied, their use as an adjunctive therapy may be beneficial if there is clinical evidence of bronchospasm or significant upper-airway edema.

WHAT YOU SHOULD DO INSTEAD

Rapid administration of epinephrine saves lives, reduces need for adjuvant treatments and hospitalization, and is associated with decreased risk of developing biphasic anaphylactic reactions (OR, 0.2; 95% CI, 0-0.6).5-7 Some clinicians are apprehensive about using epinephrine owing to fears related to negative side effects, particularly adverse cardiovascular events. Kawano et al18 performed a retrospective evaluation of 492 ED visits for anaphylaxis and found that epinephrine is administered less often in older patients (age >50 years); however, when administered intramuscularly, there was no significant difference in adverse cardiovascular events in this population compared with younger individuals. The study did demonstrate an increased rate of adverse cardiac events in older patients receiving intravenous epinephrine, an observation that the authors attributed partly to dosing errors that were reported more often with intravenous use.18

RECOMMENDATIONS

  • Always promptly administer intramuscular epinephrine when treating anaphylaxis.
  • Routine administration of corticosteroids in the treatment of anaphylaxis is not advised owing to insufficient data supporting their efficacy and potential for adverse events. Some patient populations may derive benefit from corticosteroids, including individuals with history of asthma exhibiting bronchospastic symptoms, individuals with significant upper-airway edema, and those undergoing certain chemotherapy regimens.

CONCLUSIONS

In the clinical vignette, the hospitalist withheld the first-line treatment for anaphylaxis, epinephrine. Without the support of evidence in the literature, patients receive corticosteroids and antihistamines more often than epinephrine for suspected anaphylaxis. No evidence supports the routine use of corticosteroids in the management of anaphylaxis or in the prevention of biphasic reactions. Further, recent research demonstrates significant adverse events are associated with even short courses of corticosteroids.

Do you think this is a low-value practice? Is this truly a “Thing We Do for No Reason”? Share what you do in your practice and join in the conversation online by retweeting it on Twitter (#TWDFNR) and liking it on Facebook. We invite you to propose ideas for other “Things We Do for No Reason” topics by emailing TWDFNR@hospitalmedicine.org.

References

1. Sampson HA, Muñoz-Furlong A, Campbell RL, et al. Second symposium on the definition and management of anaphylaxis: summary report--Second National Institute of Allergy and Infectious Disease/Food Allergy and Anaphylaxis Network symposium. J Allergy Clin Immunol. 2006;117(2):391-397. https://doi.org/10.1016/j.jaci.2005.12.1303
2. Wood RA, Camargo CA Jr, Lieberman P, et al. Anaphylaxis in America: the prevalence and characteristics of anaphylaxis in the United States. J Allergy Clin Immunol. 2014;133(2):461-467. https://doi.org/10.1016/j.jaci.2013.08.016
3. Lee S, Bellolio MF, Hess EP, Erwin P, Murad MH, Campbell RL. Time of onset and predictors of biphasic anaphylactic reactions: a systematic review and meta-analysis. J Allergy Clin Immunol Pract. 2015;3(3):408-16.e162. https://doi.org/10.1016/j.jaip.2014.12.010
4. Simons KJ, Simons FE. Epinephrine and its use in anaphylaxis: current issues. Curr Opin Allergy Clin Immunol. 2010;10(4):354-361. https://doi.org/10.1097/ACI.0b013e32833bc670
5. Fleming JT, Clark S, Camargo CA Jr, Rudders SA. Early treatment of food-induced anaphylaxis with epinephrine is associated with a lower risk of hospitalization. J Allergy Clin Immunol Pract. 2015;3(1):57-62. https://doi.org/10.1016/j.jaip.2014.07.004
6. Sundquist BK, Jose J, Pauze D, Pauze D, Wang H, Järvinen KM. Anaphylaxis risk factors for hospitalization and intensive care: a comparison between adults and children in an upstate New York emergency department. Allergy Asthma Proc. 2019;40(1):41-47. https://doi.org/10.2500/aap.2019.40.4189
7. Hochstadter E, Clarke A, De Schryver S, et al. Increasing visits for anaphylaxis and the benefits of early epinephrine administration: a 4-year study at a pediatric emergency department in Montreal, Canada. J Allergy Clin Immunol. 2016;137(6):1888-1890.e4. https://doi.org/10.1016/j.jaci.2016.02.016
8. Worm M, Moneret-Vautrin A, Scherer K, et al. First European data from the network of severe allergic reactions (NORA). Allergy. 2014;69(10):1397-1404. https://doi.org/10.1111/all.12475
9. Shaker MS, Wallace DV, Golden DBK, et al. Anaphylaxis—a 2020 practice parameter update, systemic review, and Grading of Recommendations, Assessment, Development, and Evaluation (GRADE) analysis. J Allergy Clin Immunol. 2020;145(4):1082-1123. https://doi.org/10.1016/j.jaci.2020.01.017
10. Liyanage CK, Galappatthy P, Seneviratne SL. Corticosteroids in management of anaphylaxis; a systematic review of evidence. Eur Ann Allergy Clin Immunol. 2017;49(5):196-207. https://doi.org/10.23822/EurAnnACI.1764-1489.15
11. Use of dexamethasone in prevention of the second phase of a biphasic reaction of anaphylaxis. ClinicalTrials.gov identifier: NCT03523221. Updated July 29, 2020. Accessed July 16, 2021. https://clinicaltrials.gov/ct2/show/NCT03523221
12. Michelson KA, Monuteaux MC, Neuman MI. Glucocorticoids and hospital length of stay for children with anaphylaxis: a retrospective study. J Pediatr. 2015;167(3):719-24.e243. https://doi.org/10.1016/j.jpeds.2015.05.033
13. Grunau BE, Wiens MO, Rowe BH, et al. Emergency department corticosteroid use for allergy or anaphylaxis is not associated with decreased relapses. Ann Emerg Med. 2015;66(4):381-389. https://doi.org/10.1016/j.annemergmed.2015.03.003
14. Yao TC, Huang YW, Chang SM, Tsai SY, Wu AC, Tsai HJ. Association between oral corticosteroid bursts and severe adverse events: a nationwide population-based cohort study. Ann Intern Med. 2020;173(5):325-330. https://doi.org/10.7326/M20-0432
15. Yao TC, Wang JY, Chang SM, et al. Association of oral corticosteroid bursts with severe adverse events in children. JAMA Pediatr. 2021;175(7):723-729. https://doi.org/10.1001/jamapediatrics.2021.0433
16. Rowe BH, Spooner CH, Ducharme FM, Bretzlaff JA, Bota GW. Corticosteroids for preventing relapse following acute exacerbations of asthma. Cochrane Database Syst Rev. 2007 Jul 18;(3):CD000195. https://doi.org/10.1002/14651858.CD000195.pub2
17. Gates A, Gates M, Vandermeer B, et al. Glucocorticoids for croup in children. Cochrane Database Syst Rev. 2018;8(8):CD001955. https://doi.org/10.1002/14651858.CD001955.pub4
18. Kawano T, Scheuermeyer FX, Stenstrom R, Rowe BH, Grafstein E, Grunau B. Epinephrine use in older patients with anaphylaxis: clinical outcomes and cardiovascular complications. Resuscitation. 2017;112:53-58. https://doi.org/10.1016/j.resuscitation.2016.12.020

References

1. Sampson HA, Muñoz-Furlong A, Campbell RL, et al. Second symposium on the definition and management of anaphylaxis: summary report--Second National Institute of Allergy and Infectious Disease/Food Allergy and Anaphylaxis Network symposium. J Allergy Clin Immunol. 2006;117(2):391-397. https://doi.org/10.1016/j.jaci.2005.12.1303
2. Wood RA, Camargo CA Jr, Lieberman P, et al. Anaphylaxis in America: the prevalence and characteristics of anaphylaxis in the United States. J Allergy Clin Immunol. 2014;133(2):461-467. https://doi.org/10.1016/j.jaci.2013.08.016
3. Lee S, Bellolio MF, Hess EP, Erwin P, Murad MH, Campbell RL. Time of onset and predictors of biphasic anaphylactic reactions: a systematic review and meta-analysis. J Allergy Clin Immunol Pract. 2015;3(3):408-16.e162. https://doi.org/10.1016/j.jaip.2014.12.010
4. Simons KJ, Simons FE. Epinephrine and its use in anaphylaxis: current issues. Curr Opin Allergy Clin Immunol. 2010;10(4):354-361. https://doi.org/10.1097/ACI.0b013e32833bc670
5. Fleming JT, Clark S, Camargo CA Jr, Rudders SA. Early treatment of food-induced anaphylaxis with epinephrine is associated with a lower risk of hospitalization. J Allergy Clin Immunol Pract. 2015;3(1):57-62. https://doi.org/10.1016/j.jaip.2014.07.004
6. Sundquist BK, Jose J, Pauze D, Pauze D, Wang H, Järvinen KM. Anaphylaxis risk factors for hospitalization and intensive care: a comparison between adults and children in an upstate New York emergency department. Allergy Asthma Proc. 2019;40(1):41-47. https://doi.org/10.2500/aap.2019.40.4189
7. Hochstadter E, Clarke A, De Schryver S, et al. Increasing visits for anaphylaxis and the benefits of early epinephrine administration: a 4-year study at a pediatric emergency department in Montreal, Canada. J Allergy Clin Immunol. 2016;137(6):1888-1890.e4. https://doi.org/10.1016/j.jaci.2016.02.016
8. Worm M, Moneret-Vautrin A, Scherer K, et al. First European data from the network of severe allergic reactions (NORA). Allergy. 2014;69(10):1397-1404. https://doi.org/10.1111/all.12475
9. Shaker MS, Wallace DV, Golden DBK, et al. Anaphylaxis—a 2020 practice parameter update, systemic review, and Grading of Recommendations, Assessment, Development, and Evaluation (GRADE) analysis. J Allergy Clin Immunol. 2020;145(4):1082-1123. https://doi.org/10.1016/j.jaci.2020.01.017
10. Liyanage CK, Galappatthy P, Seneviratne SL. Corticosteroids in management of anaphylaxis; a systematic review of evidence. Eur Ann Allergy Clin Immunol. 2017;49(5):196-207. https://doi.org/10.23822/EurAnnACI.1764-1489.15
11. Use of dexamethasone in prevention of the second phase of a biphasic reaction of anaphylaxis. ClinicalTrials.gov identifier: NCT03523221. Updated July 29, 2020. Accessed July 16, 2021. https://clinicaltrials.gov/ct2/show/NCT03523221
12. Michelson KA, Monuteaux MC, Neuman MI. Glucocorticoids and hospital length of stay for children with anaphylaxis: a retrospective study. J Pediatr. 2015;167(3):719-24.e243. https://doi.org/10.1016/j.jpeds.2015.05.033
13. Grunau BE, Wiens MO, Rowe BH, et al. Emergency department corticosteroid use for allergy or anaphylaxis is not associated with decreased relapses. Ann Emerg Med. 2015;66(4):381-389. https://doi.org/10.1016/j.annemergmed.2015.03.003
14. Yao TC, Huang YW, Chang SM, Tsai SY, Wu AC, Tsai HJ. Association between oral corticosteroid bursts and severe adverse events: a nationwide population-based cohort study. Ann Intern Med. 2020;173(5):325-330. https://doi.org/10.7326/M20-0432
15. Yao TC, Wang JY, Chang SM, et al. Association of oral corticosteroid bursts with severe adverse events in children. JAMA Pediatr. 2021;175(7):723-729. https://doi.org/10.1001/jamapediatrics.2021.0433
16. Rowe BH, Spooner CH, Ducharme FM, Bretzlaff JA, Bota GW. Corticosteroids for preventing relapse following acute exacerbations of asthma. Cochrane Database Syst Rev. 2007 Jul 18;(3):CD000195. https://doi.org/10.1002/14651858.CD000195.pub2
17. Gates A, Gates M, Vandermeer B, et al. Glucocorticoids for croup in children. Cochrane Database Syst Rev. 2018;8(8):CD001955. https://doi.org/10.1002/14651858.CD001955.pub4
18. Kawano T, Scheuermeyer FX, Stenstrom R, Rowe BH, Grafstein E, Grunau B. Epinephrine use in older patients with anaphylaxis: clinical outcomes and cardiovascular complications. Resuscitation. 2017;112:53-58. https://doi.org/10.1016/j.resuscitation.2016.12.020

Publications
Publications
Topics
Article Type
Display Headline
Things We Do for No Reason™: Routine Use of Corticosteroids for the Treatment of Anaphylaxis
Display Headline
Things We Do for No Reason™: Routine Use of Corticosteroids for the Treatment of Anaphylaxis
Sections
Article Source

© 2021 Society of Hospital Medicine

Citation Override
J Hosp Med. Published Online First October 13, 2021. DOI: 10.12788/jhm.3665
Disallow All Ads
Correspondence Location
Roger Auth; Email: roger.auth@lifespan.org.
Content Gating
Gated (full article locked unless allowed per User)
Alternative CME
Disqus Comments
Default
Use ProPublica
Hide sidebar & use full width
render the right sidebar.
Conference Recap Checkbox
Not Conference Recap
Clinical Edge
Display the Slideshow in this Article
Gating Strategy
First Page Free
Medscape Article
Display survey writer
Reuters content
Disable Inline Native ads
WebMD Article
Article PDF Media